From min0220 at postech.ac.kr Wed Jan 2 01:13:05 2008 From: min0220 at postech.ac.kr (Min Seung Kyu) Date: Wed, 2 Jan 2008 09:13:05 +0900 (KST) Subject: [Pw_forum] Information about Hamiltonian matrix Message-ID: Dear users and developers, I'm trying to modify PWSCF code and, first of all, I should get Hamiltonian matrix. As far as I know, however, in PWSCF, the Hamiltonian matrix elements are not stored in output files, even in the codes! ( am I right? ) Then, for example, how can I obtain green's function matrix ( G =[e - H]^(-1) ) without knowing Hamiltonian matrix in PWSCF? My question is: Can I obtain hamiltonian matrix in PWSCF? If not, do you have any recommendations for building hamiltonian matrix? Actually, I saw several papers which calculate quantum conductance using quantum-espresso, and I believe that authors built hamiltonian matrix from PWSCF! Happy new year. Sincerely, Min. -------------------------------Sig.--------------------------- Center for Superfunctional Materials, Department of Chemistry, Pohang University of Science and Technology (POSTECH) Mr. Seung Kyu, Min Ph.D. Candidate E-Mail: min0220 at postech.ac.kr Tel. : 82-54-279-5858 ---------------------------------------------- From akohlmey at cmm.chem.upenn.edu Wed Jan 2 02:07:49 2008 From: akohlmey at cmm.chem.upenn.edu (Axel Kohlmeyer) Date: Tue, 1 Jan 2008 20:07:49 -0500 (EST) Subject: [Pw_forum] Information about Hamiltonian matrix In-Reply-To: References: Message-ID: On Wed, 2 Jan 2008, Min Seung Kyu wrote: MSK> Dear users and developers, MSK> MSK> I'm trying to modify PWSCF code and, first of all, I should get MSK> Hamiltonian matrix. As far as I know, however, in PWSCF, the Hamiltonian MSK> matrix elements are not stored in output files, even in the codes! ( am I MSK> right? ) MSK> MSK> Then, for example, how can I obtain green's function matrix MSK> ( G =[e - H]^(-1) ) without knowing Hamiltonian matrix in PWSCF? MSK> MSK> My question is: MSK> MSK> Can I obtain hamiltonian matrix in PWSCF? If not, do you have any MSK> recommendations for building hamiltonian matrix? Actually, I saw several MSK> papers which calculate quantum conductance using quantum-espresso, and I MSK> believe that authors built hamiltonian matrix from PWSCF! dear min, before going any further on your own, i suggest you have a look at the WanT package ( http://www.wannier-transport.org/ ) which seems (at least to my untrained eye) to implement what you are referring to, and then take it from there. FYI: WanT is a post-processing package for Q-E. cheers, axel. MSK> MSK> Happy new year. MSK> MSK> Sincerely, Min. MSK> MSK> MSK> -------------------------------Sig.--------------------------- MSK> Center for Superfunctional Materials, Department of Chemistry, MSK> Pohang University of Science and Technology (POSTECH) MSK> MSK> Mr. Seung Kyu, Min MSK> MSK> Ph.D. Candidate MSK> MSK> E-Mail: min0220 at postech.ac.kr MSK> Tel. : 82-54-279-5858 MSK> ---------------------------------------------- MSK> MSK> MSK> MSK> MSK> MSK> _______________________________________________ MSK> Pw_forum mailing list MSK> Pw_forum at pwscf.org MSK> http://www.democritos.it/mailman/listinfo/pw_forum MSK> -- ======================================================================= Axel Kohlmeyer akohlmey at cmm.chem.upenn.edu http://www.cmm.upenn.edu Center for Molecular Modeling -- University of Pennsylvania Department of Chemistry, 231 S.34th Street, Philadelphia, PA 19104-6323 tel: 1-215-898-1582, fax: 1-215-573-6233, office-tel: 1-215-898-5425 ======================================================================= If you make something idiot-proof, the universe creates a better idiot. From lanhaiping at gmail.com Wed Jan 2 02:11:55 2008 From: lanhaiping at gmail.com (lan haiping) Date: Wed, 2 Jan 2008 09:11:55 +0800 Subject: [Pw_forum] Information about Hamiltonian matrix In-Reply-To: References: Message-ID: Hi, You can just try 'grep hamiltonian *f90' in $QE/PW directory. it will give you hamiltonian information . if you want to compute quantum conductance, then you can try to have a look at $QE/PWCOND directory . Bests, H.P. On Jan 2, 2008 8:13 AM, Min Seung Kyu wrote: > Dear users and developers, > > I'm trying to modify PWSCF code and, first of all, I should get > Hamiltonian matrix. As far as I know, however, in PWSCF, the Hamiltonian > matrix elements are not stored in output files, even in the codes! ( am I > right? ) > > Then, for example, how can I obtain green's function matrix > ( G =[e - H]^(-1) ) without knowing Hamiltonian matrix in PWSCF? > > My question is: > > Can I obtain hamiltonian matrix in PWSCF? If not, do you have any > recommendations for building hamiltonian matrix? Actually, I saw several > papers which calculate quantum conductance using quantum-espresso, and I > believe that authors built hamiltonian matrix from PWSCF! > > Happy new year. > > Sincerely, Min. > > > -------------------------------Sig.--------------------------- > Center for Superfunctional Materials, Department of Chemistry, > Pohang University of Science and Technology (POSTECH) > > Mr. Seung Kyu, Min > > Ph.D. Candidate > > E-Mail: min0220 at postech.ac.kr > Tel. : 82-54-279-5858 > ---------------------------------------------- > > > > > > _______________________________________________ > Pw_forum mailing list > Pw_forum at pwscf.org > http://www.democritos.it/mailman/listinfo/pw_forum > -- Hai-Ping Lan Department of Electronics , Peking University , Bejing, 100871 lanhaiping at gmail.com, hplan at pku.edu.cn -------------- next part -------------- An HTML attachment was scrubbed... URL: http://www.democritos.it/pipermail/pw_forum/attachments/20080102/8fd26d79/attachment.htm From min0220 at postech.ac.kr Wed Jan 2 06:11:56 2008 From: min0220 at postech.ac.kr (Min Seung Kyu) Date: Wed, 2 Jan 2008 14:11:56 +0900 (KST) Subject: [Pw_forum] Information about Hamiltonian matrix In-Reply-To: References: Message-ID: Thank you all for your quick replies. Actually, quantum conductance is not of my direct interest. It's just an example. :) However, it was very helpful for me to know that WanT program uses MLWFs-based Hamiltonian. I think that subroutines for wannier functions in WanT and WANNIER90 programs are almost same, right? Then, I have several questions which are related to WANNIER90. ( I know another mailing list for WANNIER90 exists, but I want to keep discussing in the same thread. ) I want to calculate MLWFs-based Hamiltonian matrix not by a post-processing tool but by a library, ex) I want to calculate MLWF-based Hamiltonian at each geometry optimization step or each time step in AIMD. I looked at WANNIER90 source codes, and I found that the subroutines in 'hamiltonian.F90' are exactly what I want. However, it seems that it is not implemented in 'wannier_lib.F90'. 'wannier_lib.F90' can compute not hamiltonian matrix but unitary matrix. Am I right? Should I compute MLWFs-based hamiltonian on my own after calling 'wannier_run' subroutine in 'wannier_lib.F90'? Thank you very much. Sincerely, Min. -------------------------------Sig.--------------------------- Center for Superfunctional Materials, Department of Chemistry, Pohang University of Science and Technology (POSTECH) Mr. Seung Kyu, Min Ph.D. Candidate E-Mail: min0220 at postech.ac.kr Tel. : 82-54-279-5858 ---------------------------------------------- From giannozz at nest.sns.it Wed Jan 2 12:29:17 2008 From: giannozz at nest.sns.it (Paolo Giannozzi) Date: Wed, 2 Jan 2008 12:29:17 +0100 Subject: [Pw_forum] Information about Hamiltonian matrix In-Reply-To: References: Message-ID: <200801021229.19167.giannozz@nest.sns.it> On Wednesday 02 January 2008 01:13, Min Seung Kyu wrote: > Can I obtain hamiltonian matrix in PWSCF? not in PWscf nor in any other plane-wave based code designed for serious usage. All you can have is the result, H\psi, of the application of the hamiltonian H operator on a vector \psi. This is done in routine PW/h_psi.f90 P. -- Paolo Giannozzi, Democritos and Udine University From marzari at MIT.EDU Wed Jan 2 14:48:44 2008 From: marzari at MIT.EDU (marzari at MIT.EDU) Date: Wed, 02 Jan 2008 08:48:44 -0500 Subject: [Pw_forum] Information about Hamiltonian matrix In-Reply-To: References: Message-ID: <20080102084844.ytnewcprghdog8sk@webmail.mit.edu> Dear Min, maybe there is too much choice, but the Hamiltonian matrix and the Green's function are also computed in the Wannier code (www.wannier.org), interfaced with PWSCF; both in Wannier90 and in WanT these are calculated in a basis of maximally-localized Wannier functions. Best, nicola marzari Quoting Axel Kohlmeyer : > On Wed, 2 Jan 2008, Min Seung Kyu wrote: > > MSK> Dear users and developers, > MSK> > MSK> I'm trying to modify PWSCF code and, first of all, I should get > MSK> Hamiltonian matrix. As far as I know, however, in PWSCF, the Hamiltonian > MSK> matrix elements are not stored in output files, even in the > codes! ( am I > MSK> right? ) > MSK> > MSK> Then, for example, how can I obtain green's function matrix > MSK> ( G =[e - H]^(-1) ) without knowing Hamiltonian matrix in PWSCF? > MSK> > MSK> My question is: > MSK> > MSK> Can I obtain hamiltonian matrix in PWSCF? If not, do you have any > MSK> recommendations for building hamiltonian matrix? Actually, I saw several > MSK> papers which calculate quantum conductance using quantum-espresso, and I > MSK> believe that authors built hamiltonian matrix from PWSCF! > > dear min, > > before going any further on your own, i suggest you have > a look at the WanT package ( http://www.wannier-transport.org/ ) > which seems (at least to my untrained eye) to implement > what you are referring to, and then take it from there. > FYI: WanT is a post-processing package for Q-E. > > cheers, > axel. > > > MSK> > MSK> Happy new year. > MSK> > MSK> Sincerely, Min. > MSK> > MSK> > MSK> -------------------------------Sig.--------------------------- > MSK> Center for Superfunctional Materials, Department of Chemistry, > MSK> Pohang University of Science and Technology (POSTECH) > MSK> > MSK> Mr. Seung Kyu, Min > MSK> > MSK> Ph.D. Candidate > MSK> > MSK> E-Mail: min0220 at postech.ac.kr > MSK> Tel. : 82-54-279-5858 > MSK> ---------------------------------------------- > MSK> > MSK> > MSK> > MSK> > MSK> > MSK> _______________________________________________ > MSK> Pw_forum mailing list > MSK> Pw_forum at pwscf.org > MSK> http://www.democritos.it/mailman/listinfo/pw_forum > MSK> > > -- > ======================================================================= > Axel Kohlmeyer akohlmey at cmm.chem.upenn.edu http://www.cmm.upenn.edu > Center for Molecular Modeling -- University of Pennsylvania > Department of Chemistry, 231 S.34th Street, Philadelphia, PA 19104-6323 > tel: 1-215-898-1582, fax: 1-215-573-6233, office-tel: 1-215-898-5425 > ======================================================================= > If you make something idiot-proof, the universe creates a better idiot. > _______________________________________________ > Pw_forum mailing list > Pw_forum at pwscf.org > http://www.democritos.it/mailman/listinfo/pw_forum > From marzari at MIT.EDU Wed Jan 2 14:58:04 2008 From: marzari at MIT.EDU (marzari at MIT.EDU) Date: Wed, 02 Jan 2008 08:58:04 -0500 Subject: [Pw_forum] Information about Hamiltonian matrix In-Reply-To: <200801021229.19167.giannozz@nest.sns.it> References: <200801021229.19167.giannozz@nest.sns.it> Message-ID: <20080102085804.1kazurya148ow0sg@webmail.mit.edu> Dear Min, what Paolo means here is the Hamiltonian in a plane wave basis. Of course, all codes give you the Hamiltonian in the basis of the eigenvectors (since these are the eigenvalues) and both Wannier90 and WanT give you the Hamiltonian in a MLWFs basis; this latter has the advantage of being localized (hence the Hamiltonian is sparse and diagonally dominanant), and minimal but exact (e.g. for Si you only need 4 MLWFs to describe exactly the whole valence band manifold). Happy new year, maximally-delocalized nicola Quoting Paolo Giannozzi : > On Wednesday 02 January 2008 01:13, Min Seung Kyu wrote: > >> Can I obtain hamiltonian matrix in PWSCF? > > not in PWscf nor in any other plane-wave based code designed for > serious usage. All you can have is the result, H\psi, of the application > of the hamiltonian H operator on a vector \psi. This is done in routine > PW/h_psi.f90 > > P. From a.mostofi at imperial.ac.uk Wed Jan 2 16:00:01 2008 From: a.mostofi at imperial.ac.uk (Arash Mostofi) Date: Wed, 02 Jan 2008 15:00:01 +0000 Subject: [Pw_forum] Information about Hamiltonian matrix In-Reply-To: References: Message-ID: <477BA6F1.10005@imperial.ac.uk> Dear Min The latest version of Wannier90 (v1.1, http://www.wannier.org) does have the capability to calculate and write the Hamiltonian matrix in the maximally-localised Wannier function basis to a file, including when running it as a library. This file may then be read by whichever ab initio code you happen to be using to do your MD. Of course, if you were feeling adventurous, it would be an easy bit of coding to modify the library call to pass the Hamiltonian matrix back to the ab initio code directly :0) Best regards Arash Mostofi -- :------------------------------------------------------------: : Dr. Arash A. Mostofi :: a.mostofi at imperial.ac.uk : : Lecturer and RCUK Fellow :: : : Depts. of Materials & Physics :: : : Imperial College London :: T +44 (0)207 594 6753 : : London SW7 2AZ, United Kingdom :: F +44 (0)207 594 6757 : :------- http://www.cmth.ph.ic.ac.uk/people/a.mostofi -------: Date: Wed, 2 Jan 2008 14:11:56 +0900 (KST) From: Min Seung Kyu Subject: Re: [Pw_forum] Information about Hamiltonian matrix To: PWSCF Forum Message-ID: Content-Type: TEXT/PLAIN; charset=US-ASCII; format=flowed Thank you all for your quick replies. Actually, quantum conductance is not of my direct interest. It's just an example. :) However, it was very helpful for me to know that WanT program uses MLWFs-based Hamiltonian. I think that subroutines for wannier functions in WanT and WANNIER90 programs are almost same, right? Then, I have several questions which are related to WANNIER90. ( I know another mailing list for WANNIER90 exists, but I want to keep discussing in the same thread. ) I want to calculate MLWFs-based Hamiltonian matrix not by a post-processing tool but by a library, ex) I want to calculate MLWF-based Hamiltonian at each geometry optimization step or each time step in AIMD. I looked at WANNIER90 source codes, and I found that the subroutines in 'hamiltonian.F90' are exactly what I want. However, it seems that it is not implemented in 'wannier_lib.F90'. 'wannier_lib.F90' can compute not hamiltonian matrix but unitary matrix. Am I right? Should I compute MLWFs-based hamiltonian on my own after calling 'wannier_run' subroutine in 'wannier_lib.F90'? Thank you very much. Sincerely, Min. -------------------------------Sig.--------------------------- Center for Superfunctional Materials, Department of Chemistry, Pohang University of Science and Technology (POSTECH) Mr. Seung Kyu, Min Ph.D. Candidate E-Mail: min0220 at postech.ac.kr Tel. : 82-54-279-5858 ---------------------------------------------- From wangxinquan at tju.edu.cn Thu Jan 3 09:24:53 2008 From: wangxinquan at tju.edu.cn (wangxinquan) Date: Thu, 3 Jan 2008 16:24:53 +0800 Subject: [Pw_forum] Ce2O3, LDA+U, Pseudopotential, Semicore, etc. Message-ID: <399348445.01291@tju.edu.cn> Dear users and developers, I'm trying to calculate the DOS for Ce2O3, according to the literature ?S. Fabris, S. Gironcoli, S. Baroni, G. Vicario, and G. Balducci, Phy. Rev. B 71, 041102?.(Very helpful, Thanks a lot) The pseudopotential (PP) of Ce includes(sample1) semicore state(5p6) or not includes (sample2), respectively. The band structures were plotted as following. In my opinion, semicore states(5p6) should be included in the PP generation process for rare earth elements, such as Ce. Is it right? Thanks in advance. Best regards, X.Q. Wang ===================================== X.Q. Wang wangxinquan at tju.edu.cn School of Chemical Engineering and Technology Tianjin University 92 Weijin Road, Tianjin, P. R. China tel:86-22-27890268, fax: 86-22-27892301 ===================================== -------------- next part -------------- An HTML attachment was scrubbed... URL: http://www.democritos.it/pipermail/pw_forum/attachments/20080103/229bbb45/attachment.htm From fabris at democritos.it Thu Jan 3 11:04:47 2008 From: fabris at democritos.it (Stefano Fabris) Date: Thu, 3 Jan 2008 11:04:47 +0100 Subject: [Pw_forum] Ce2O3, LDA+U, Pseudopotential, Semicore, etc. In-Reply-To: <399348445.01291@tju.edu.cn> References: <399348445.01291@tju.edu.cn> Message-ID: <3F2655C0-8112-4FFA-9B5C-22DF10554BEF@democritos.it> On 3 Jan 2008, at 09:24, wangxinquan wrote: > > In my opinion, semicore states(5p6) should be included in the PP > generation process for rare earth elements, such as Ce. Is it right? Dear Wang, absolutely yes. In my experience one should include not only the 5p states but also the 5s. Stefano --- Stefano Fabris Theory at Elettra Group CNR-INFM DEMOCRITOS National Simulation Center c/o Sincrotrone Trieste - SS14, Km 163,5 Basovizza, I-34012 TRIESTE email: fabris at democritos.it tel: +39 040 375-8735 fax: -8776 --- From wchan957 at gmail.com Thu Jan 3 19:52:52 2008 From: wchan957 at gmail.com (wilson chan) Date: Thu, 3 Jan 2008 18:52:52 +0000 Subject: [Pw_forum] Ce PP generation Message-ID: Dear users and developers, I'm having difficulty in generating the ultrasoft pseudopential of Ce, with the 5s5p semicores, etc., using the generation code in atomic. Can anyone help me out please? I attach my input file here, in which lots of parameters probably are not correct, due to my lacking knowledge. Please correct me, and thanks in advance. &input title='Ce', zed=58.0, iswitch=3, nld=5, rlderiv=3.20, eminld=-4.0, emaxld=4.0, deld=0.02, rel=1, config='[Xe] 4f1 5d1 6s2 6p-1', dft='PBE' / &inputp pseudotype=3, lloc=1, file_pseudopw='Ce.UPF', nlcc=.true., rcloc=3.2 / 6 4F 4 3 1.00 0.00 3.2 3.2 5D 3 2 1.00 0.00 3.2 3.2 5S 1 0 2.00 0.00 1.0 3.2 5P 2 1 6.00 0.00 1.0 3.2 6S 2 0 2.00 0.00 3.2 3.2 6P 3 1 -1.00 0.15 3.2 3.2 Yours, wilson -------------- next part -------------- An HTML attachment was scrubbed... URL: http://www.democritos.it/pipermail/pw_forum/attachments/20080103/44e716c3/attachment.htm From kddst2005 at yahoo.com.cn Fri Jan 4 03:16:53 2008 From: kddst2005 at yahoo.com.cn (Dongdong Kang) Date: Fri, 4 Jan 2008 10:16:53 +0800 (CST) Subject: [Pw_forum] about 'md' Message-ID: <925031.10900.qm@web92012.mail.cnb.yahoo.com> Dear all I'm working on vibrational properties of GaSe, I have a problem need your help. When I do 'md' with parallel run, a error information occurs in the output file. I want to know what's wrong and how to solve this problem. thanks dongdong kang the output file: ........................ ........................ Entering Dynamics: iteration = 151 time = 0.1461 pico-seconds ATOMIC_POSITIONS (crystal) Ga -0.010555758 -0.025997987 0.073707594 Se 0.647991277 0.301269214 0.139014245 Se 0.058735718 0.048266323 0.360735776 Ga 0.399531320 0.721859528 0.428022848 Ga 0.316194560 0.701828747 0.569304531 Se -0.013253671 0.045252312 0.636151252 Se 0.641332389 0.283606920 0.863446350 Ga -0.040170969 -0.077639383 0.929703863 kinetic energy (Ekin) = 0.00987857 Ry temperature = 148.54294695 K Ekin + Etot (const) = -94.24971227 Ry Linear momentum : 0.0000000000 0.0000000000 0.0000000000 p0_18818: p4_error: interrupt SIGSEGV: 11 rm_l_1_18840: (3456.863950) net_send: could not write to fd=5, errno = 32 rm_l_2_18861: (3451.681321) net_send: could not write to fd=5, errno = 32 rm_l_3_18882: (3451.460365) net_send: could not write to fd=5, errno = 32 p2_18844: (3457.705481) net_send: could not write to fd=5, errno = 32 Kang National University of Defense Technology, P R China 410073, Hunan, Changsha --------------------------------- ??????????????????? -------------- next part -------------- An HTML attachment was scrubbed... URL: http://www.democritos.it/pipermail/pw_forum/attachments/20080104/d1fa3a00/attachment.htm From giannozz at nest.sns.it Fri Jan 4 15:17:38 2008 From: giannozz at nest.sns.it (Paolo Giannozzi) Date: Fri, 4 Jan 2008 15:17:38 +0100 Subject: [Pw_forum] about 'md' In-Reply-To: <925031.10900.qm@web92012.mail.cnb.yahoo.com> References: <925031.10900.qm@web92012.mail.cnb.yahoo.com> Message-ID: On Jan 4, 2008, at 3:16 , Dongdong Kang wrote: > Entering Dynamics: iteration = 151 > [...] > p0_18818: p4_error: interrupt SIGSEGV: 11 > rm_l_1_18840: (3456.863950) net_send: could not write to fd=5, > errno = 32 > rm_l_2_18861: (3451.681321) net_send: could not write to fd=5, > errno = 32 > rm_l_3_18882: (3451.460365) net_send: could not write to fd=5, > errno = 32 > p2_18844: (3457.705481) net_send: could not write to fd=5, errno = 32 it is very hard to say anything about an error that basically means "you did something very wrong with memory". It might also be a communication or a hardware problem, rather than something related to the code itself. The only code-related reason that could lead to a crash after 151 iterations is a memory leak, but there is no other evidence for it. If you look for "SIGSEGV 11" on google, you will see that it is a quite famous, or infamous, error message... Paolo --- Paolo Giannozzi, Dept of Physics, University of Udine via delle Scienze 208, 33100 Udine, Italy Phone +39-0432-558216, fax +39-0432-558222 From jackygrahamez at gmail.com Sat Jan 5 17:59:41 2008 From: jackygrahamez at gmail.com (Jack Shultz) Date: Sat, 5 Jan 2008 11:59:41 -0500 Subject: [Pw_forum] Newbie Questions Message-ID: <6c65435e0801050859s4868ab06ma1d665b5c069ef60@mail.gmail.com> Hello, I am new to Quantume Espresso and hope I can integrate it into my distributed computing project as I decribe in this post http://www.hydrogenathome.org/forum_thread.php?id=173 As I am formulating my analytical methods, there may be flaws in what I am proposing. I hope to use Climbing-Image Nudged Elastic Band method to identify a transition state structure. Then use that transition state structure in a molecular docking simulation. I then run these simulations on metalic enzymes in hopes of identifing optimal enzymes for water spliting hydrogen generating reactions. Of course before I get that far I will run positve/negative controls to QC my methods. Having resource sharing among volunteers through Berkley's BOINC platform is really benefitial otherwise I would not have the ability to do this kind of analysis on this scale. Anyway, my questions right now: 1) What output formats does QE support? I'm trying to get the command line for open babel to convert xyz to other molecular formats and it is giving me a hard time but if QE can output to PDB or MOL2 or other molecular file format, I would really like to know how. 2) Are there command line tools that generate parameters for these simulations like Exampl17? Where do I find them? I want to be able to just supply a structure representing the substrate and another representing the product of my reaction. I want to make these parameters interchangeable because I am testing controls first. So I would be able to test H2O2 binding to Catalase for instance. 3) Finally, I don't know how to pose this question, perhaps I need read more, how does one integrate DFT+U into models of reaction pathways? Thank you in advance for any assistance. Jack Shultz Project Leader http://hydrogenathome.org -------------- next part -------------- An HTML attachment was scrubbed... URL: http://www.democritos.it/pipermail/pw_forum/attachments/20080105/e9ea60fc/attachment.htm From faridul at email.com Sun Jan 6 10:10:23 2008 From: faridul at email.com (Faridul Isalm) Date: Sun, 6 Jan 2008 04:10:23 -0500 Subject: [Pw_forum] Help to learn Message-ID: <20080106091023.E17631CE7A2@ws1-6.us4.outblaze.com> Dear Sir, I am a novice user of Quantum Espersso (PWscf).I use it on the CYGWIN platform. I did the examples successfully those are given with Quantum Espresso package. I am now trying to learn "band structure calculation" step. But I face the following problems: 1) when I run plotband.x, there need to specify "Efermi", but where I get this value. I look all output files, nowhere that is. such as for Si Efermi = 6.337, from where this value comes. 2)when this method is used for other materials such as SiO2, MnO2, GaAs, etc., then how can i specify their ibrav in the &system namelist. and also how can I know their celldm. 3)when it is used for semiconductor materials, then which variables may need to specify in the &system namelist. 4)how can I determine the proper "ecutwfc" of a particular material. is it =kT? 5)I face a great problem when I use k-points {automatic}, then output of the band does not match with the given examples. But when I use 36 k-points that are given in the examples then it is ok. Now how can I choose the proper k-points for a particular material. 6)When I run xcrysden --pwo si.scf.out, then i get " the hardware does not support the stereo" error massage. My computer is "Intel Core2 Duo, 1.86GHz, 512MB RAM". how can i solve this problem. I will be very greatful to you, if you kindly give me the solutions of the above mentioned problems. I want to learn about this method clearly. I will be very happy if you give me yours further valuable successions that can help me to learn. I am looking forward to hearing from you. Thank you for your valuable time. Sincerely Dr. A.K.Md Farid ul Islam Assistant Professor, Dept. of Physics, Carmichael University College, Rangpur-5004, Bangladesh -- Got No Time? Shop Online for Great Gift Ideas! http://mail.shopping.com/?linkin_id=8033174 From lanhaiping at gmail.com Sun Jan 6 13:01:54 2008 From: lanhaiping at gmail.com (lan haiping) Date: Sun, 6 Jan 2008 13:01:54 +0100 Subject: [Pw_forum] Help to learn In-Reply-To: <20080106091023.E17631CE7A2@ws1-6.us4.outblaze.com> References: <20080106091023.E17631CE7A2@ws1-6.us4.outblaze.com> Message-ID: Hi, Happy new year ~! On Jan 6, 2008 10:10 AM, Faridul Isalm wrote: > Dear Sir, > I am a novice user of Quantum Espersso (PWscf).I use it on the CYGWIN > platform. I did the examples successfully those are given with Quantum > Espresso package. I am now trying to learn "band structure calculation" > step. But I face the following problems: > > 1) when I run plotband.x, there need to specify "Efermi", but where I get > this value. I look all output files, nowhere that is. such as for Si Efermi > = 6.337, from where this value comes. > you can find fermi level at output, if not, i think it is due to your smearing scheme of the calculation. For semiconductor, the fermi level should be at the middle of bandgap. For metal system, i think pwscf should always be able to give you the value of fermi level in you output file. > > 2)when this method is used for other materials such as SiO2, MnO2, GaAs, > etc., then how can i specify their ibrav in the &system namelist. and also > how can I know their celldm. i think you should read the last part of INPUT_PW.. it gives you detail description of crystal system building. of coz, you should know the coordinates of these systems before you starting to handle .. you may obtain coordinates from this http://cst-www.nrl.navy.mil/lattice/ . > > > 3)when it is used for semiconductor materials, then which variables may > need to specify in the &system namelist. > there are several examples for semiconductors in $QE/examples, i think you can benefit from these tests. > > 4)how can I determine the proper "ecutwfc" of a particular material. is it > =kT? hmmm, ecutwfc is one of the most key parameters in periodic boundary calculation, which determines your calculation's accuracy. it should not be equal to kT, i think it is too small to believe. why not to read again INPUT_PW ? > > 5)I face a great problem when I use k-points {automatic}, then output of > the band does not match with the given examples. But when I use 36 k-points > that are given in the examples then it is ok. Now how can I choose the > proper k-points for a particular material. > for this problem, i think you should figure out 2 different calculations , scf and nscf, for scf calculation , we use kpoint to sample BZ to obtain convergent electron density , for nscf calculation , we use k-point path to obtain bandstructure and other quantities . k-point path is the reciprocal path between high symmetry points of your sytem. > > 6)When I run xcrysden --pwo si.scf.out, then i get " the hardware does not > support the stereo" error massage. My computer is "Intel Core2 Duo, > 1.86GHz, 512MB RAM". how can i solve this problem. > while, this problem should be due to your video card, or your improper installing related driver program of video card > > I will be very greatful to you, if you kindly give me the solutions of the > above mentioned problems. I want to learn about this method clearly. I will > be very happy if you give me yours further valuable successions that can > help me to learn. > > > I am looking forward to hearing from you. > Thank you for your valuable time. > > Sincerely > Dr. A.K.Md Farid ul Islam > Assistant Professor, > Dept. of Physics, > Carmichael University College, Rangpur-5004, Bangladesh > > > -- > Got No Time? Shop Online for Great Gift Ideas! > http://mail.shopping.com/?linkin_id=8033174 > > _______________________________________________ > Pw_forum mailing list > Pw_forum at pwscf.org > http://www.democritos.it/mailman/listinfo/pw_forum > bests, hai-ping -- Hai-Ping Lan Department of Electronics , Peking University , Bejing, 100871 lanhaiping at gmail.com, hplan at pku.edu.cn -------------- next part -------------- An HTML attachment was scrubbed... URL: http://www.democritos.it/pipermail/pw_forum/attachments/20080106/78cc996a/attachment.htm From fengjie.ma at gmail.com Mon Jan 7 03:50:31 2008 From: fengjie.ma at gmail.com (Fengjie Ma) Date: Mon, 7 Jan 2008 10:50:31 +0800 Subject: [Pw_forum] Problem with pseudopotential generation for Tb Message-ID: Hi all, When I use ld1.x to generate pseudopotential of Tb, I meet some problems below. Can someone help me? Thank you very much. ********************************************************************************* classical turning point too close to mesh 4 3 3 classical turning point too close to mesh 4 3 -4 classical turning point too close to mesh 5 2 2 classical turning point too close to mesh 5 2 -3 ........... ........... Wfc 4F rcut= 1.400 Estimated cut-off energy= 156.83 Ry This function has 0 nodes for 0 < r < 1.385 ns= 4 l= 3 sum= 0.003498123 r(ikk) 2.754160645 from gener_pseudo : info # -1 chi too large beyond r_c ********************************************************************************* The input file is: &input title='Tb', zed=65.0, rel=2, rmax = 100, iswitch =3, rlderiv=3.01, eminld=-6.0, emaxld=4.0, deld=0.02, nld=5, config='[Kr] 4d10 5s2 5p5.5 4f9 6s2 5d0.5', dft='PBE' & &inputp pseudotype=3, lloc=0, file_pseudopw='Tb.pbe', nlcc =.true., rho0=0.01, ! rcore= 1.1, ! rcloc= 2.1 & 8 5S 1 0 2.00 0.00 2.20 2.20 0.50 5P 2 1 2.00 0.00 2.20 2.20 0.50 5P 2 1 3.50 0.00 2.20 2.20 1.50 4F 4 3 6.00 0.00 1.40 1.40 2.50 4F 4 3 3.00 0.00 1.40 1.40 3.50 5D 3 2 0.50 0.00 2.10 2.10 1.50 5D 3 2 0.00 0.00 2.10 2.10 2.50 6S 1 0 2.00 0.00 2.60 2.60 0.50 ************************************************************************************** Yours, Ma -- ======================================== Institute of Theoretical Physics, CAS Address: No.55, Zhong-Guan-Cun East Road Beijing, China Phone: 86-10-62582396 Email: fengjie.ma at gmail.com -------------- next part -------------- An HTML attachment was scrubbed... URL: http://www.democritos.it/pipermail/pw_forum/attachments/20080107/b4389d7f/attachment.htm From bnrj.rudra at yahoo.com Mon Jan 7 06:38:16 2008 From: bnrj.rudra at yahoo.com (Rudra Banerjee) Date: Mon, 7 Jan 2008 11:08:16 +0530 (IST) Subject: [Pw_forum] time requirement Message-ID: <68733.8194.qm@web94108.mail.in2.yahoo.com> dear friends, i am running a serial version of PW to "relax" a hexamer water system. the input file is as follows: &CONTROL calculation = 'relax' , restart_mode = 'restart' , outdir = '.' , pseudo_dir = '../.../pseudo' , prefix = 'cage' , etot_conv_thr = 1.0d-5 , forc_conv_thr = 1.0d-4 , nstep = 100 , tstress = .true. , tprnfor = .true. , / &SYSTEM ibrav = 14, celldm(1) = 35, celldm(2) = 1, celldm(3) = 1, celldm(4) = 0, celldm(5) = 0, celldm(6) = 0, nat = 18, ntyp = 2, ecutwfc = 60 , / &ELECTRONS conv_thr = 1.0d-10 , / &IONS ion_dynamics = 'bfgs' , upscale = 30.D0 , trust_radius_max = 5.0D-1 , trust_radius_min = 1.0D-10, / ATOMIC_SPECIES O 15.99000 O.BLYP.UPF H 1.00000 H.fpmd.UPF ATOMIC_POSITIONS bohr O 0.142792E+01 0.953864E+00 -0.299480E+01 O 0.135714E+01 -0.989754E+00 0.323666E+01 O 0..545197E+01 0.207639E+00 0.259525E+00 O -0.149447E+01 -0.319790E+01 -0..936634E+00 O -0.527540E+01 0.110780E+00 -0.142466E+00 O -0.121084E+01 0.316178E+01 0.559056E+00 H 0.560581E+00 -0.689438E+00 -0.282064E+01 H 0.309141E+01 0.671706E+00 -0.214783E+01 H 0.511241E+00 0.642082E+00 0.293278E+01 H 0.443415E+00 -0.213926E+01 0.208483E+01 H 0.420832E+01 -0.380599E+00 0.156283E+01 H 0.673616E+01 -0.110545E+01 0.154498E+00 H -0.311128E+01 -0.222598E+01 -0.679627E+00 H -0.197729E+01 -0.486806E+01 -0.153807E+01 H -0.625820E+01 -0.558913E-01 0.140533E+01 H -0.403516E+01 0.150454E+01 0.192004E+00 H -0.130885E+00 0.253414E+01 -0.930242E+00 H -0.114494E+01 0.500035E+01 0.527176E+00 K_POINTS automatic 3 3 3 1 1 1 my machine(desktop,linux) is 3 GB ram, but yet it is using more space(SWAP) and giving only 2-3 iteration per day. is my input somewhat inefficient? pl. let me suggest any improvement if possible Thanks -- Rudra Have a Nice Time http://www.bose.res..in/~rudra Please, if possible, don't send me MS Word or PowerPoint attachments Why?See: http://www.gnu.org/philosophy/no-word-attachments.html Get the freedom to save as many mails as you wish. To know how, go to http://help.yahoo.com/l/in/yahoo/mail/yahoomail/tools/tools-08.html -------------- next part -------------- An HTML attachment was scrubbed... URL: http://www.democritos.it/pipermail/pw_forum/attachments/20080107/f9b93894/attachment-0001.htm From fengjie.ma at gmail.com Mon Jan 7 08:14:06 2008 From: fengjie.ma at gmail.com (Fengjie Ma) Date: Mon, 7 Jan 2008 15:14:06 +0800 Subject: [Pw_forum] A supplement of the "Problem with pseudopotential generation for Tb" Message-ID: Hi all, In the unfinshed output-file, I find that the energies of 4F and 5D orbitals are always zero no matter which electronic configuration / matching radii / pseudization energies I choose. 4 3 2.5 4F 1( 6.00) 0.0000 0.0000 0.0000 4 3 3.5 4F 1( 3.00) 0.0000 0.0000 0.0000 6 0 0.5 6S 1( 2.00) -0.2621 -0.1310 -3.5659 5 2 1.5 5D 1( 0.50) 0.0000 0.0000 0.0000 5 2 2.5 5D 1( 0.00) 0.0000 0.0000 0.0000 Yours, Ma > Hi all, > > When I use ld1.x to generate pseudopotential of Tb, I meet some problems > below. > Can someone help me? Thank you very much. > > > ********************************************************************************* > classical turning point too close to mesh 4 3 > 3 > classical turning point too close to mesh 4 3 > -4 > classical turning point too close to mesh 5 2 > 2 > classical turning point too close to mesh 5 2 > -3 > ........... > ........... > Wfc 4F rcut= 1.400 Estimated cut-off energy= 156.83 Ry > This function has 0 nodes for 0 < r < 1.385 > ns= 4 l= 3 sum= 0.003498123 r(ikk) 2.754160645 > from gener_pseudo : info # -1 > chi too large beyond r_c > > > ********************************************************************************* > > The input file is: > > &input > title='Tb', > zed=65.0, > rel=2, > rmax = 100, > iswitch =3, > rlderiv=3.01, > eminld=-6.0, > emaxld=4.0, > deld=0.02, > nld=5, > config='[Kr] 4d10 5s2 5p5.5 4f9 6s2 5d0.5', > dft='PBE' > & > &inputp > pseudotype=3, > lloc=0, > file_pseudopw='Tb.pbe', > nlcc =.true., > rho0=0.01, > ! rcore= 1.1, > ! rcloc= 2.1 > & > 8 > 5S 1 0 2.00 0.00 2.20 2.20 0.50 > 5P 2 1 2.00 0.00 2.20 2.20 0.50 > 5P 2 1 3.50 0.00 2.20 2.20 1.50 > 4F 4 3 6.00 0.00 1.40 1.40 2.50 > 4F 4 3 3.00 0.00 1.40 1.40 3.50 > 5D 3 2 0.50 0.00 2.10 2.10 1.50 > 5D 3 2 0.00 0.00 2.10 2.10 2.50 > 6S 1 0 2.00 0.00 2.60 2.60 0.50 > > > ************************************************************************************** > > Yours, > Ma > > > -- > ======================================== > Institute of Theoretical Physics, CAS > Address: No.55, Zhong-Guan-Cun East Road > Beijing, China > Phone: 86-10-62582396 > Email: fengjie.ma at gmail.com > -------------- next part -------------- > An HTML attachment was scrubbed... > URL: > http://www.democritos.it/pipermail/pw_forum/attachments/20080107/b4389d7f/attachment-0001.htm > > > -------------- next part -------------- An HTML attachment was scrubbed... URL: http://www.democritos.it/pipermail/pw_forum/attachments/20080107/c3c3f436/attachment.htm From giannozz at nest.sns.it Mon Jan 7 09:35:21 2008 From: giannozz at nest.sns.it (Paolo Giannozzi) Date: Mon, 7 Jan 2008 09:35:21 +0100 Subject: [Pw_forum] time requirement In-Reply-To: <68733.8194.qm@web94108.mail.in2.yahoo.com> References: <68733.8194.qm@web94108.mail.in2.yahoo.com> Message-ID: <1212F130-5A71-40DC-9621-C20BF53AC6ED@nest.sns.it> On Jan 7, 2008, at 6:38 , Rudra Banerjee wrote: > my machine(desktop,linux) is 3 GB ram, but yet it is using more space > (SWAP) and giving only 2-3 iteration per day. > is my input somewhat inefficient? pl. let me suggest any > improvement if possible assuming that you cannot reduce the size cell and the cutoff: - do not calculate stress if you do not need to: it is expensive - conv_thr = 1.0d-10 is quite high, 1.0d-8 should be enough - is your system an isolated system? then use K_POINTS gamma : it will use k=0 only (which is all you need for an isolated system) and exploit various tricks to reduce memory usage - setting option "diago_david_ndim" to the minimum (2) and "mixing_ndim" to a smaller value (4) reduces memory usage, but may increase CPU time - using diagonalization='cg' will also reduce memory usage, but it will increase CPU time by a sizable amount P. --- Paolo Giannozzi, Dept of Physics, University of Udine via delle Scienze 208, 33100 Udine, Italy Phone +39-0432-558216, fax +39-0432-558222 From bnrj.rudra at yahoo.com Mon Jan 7 09:48:25 2008 From: bnrj.rudra at yahoo.com (Rudra Banerjee) Date: Mon, 7 Jan 2008 14:18:25 +0530 (IST) Subject: [Pw_forum] time requirement Message-ID: <417443.36526.qm@web94107.mail.in2.yahoo.com> but is it possible to calculate phonon after doing gamma point relaxation? -- Rudra Have a Nice Time http://www.bose.res.in/~rudra Please, if possible, don't send me MS Word or PowerPoint attachments Why?See: http://www.gnu.org/philosophy/no-word-attachments.html ----- Original Message ---- From: Paolo Giannozzi To: PWSCF Forum Sent: Monday, 7 January, 2008 2:05:21 PM Subject: Re: [Pw_forum] time requirement On Jan 7, 2008, at 6:38 , Rudra Banerjee wrote: > my machine(desktop,linux) is 3 GB ram, but yet it is using more space > (SWAP) and giving only 2-3 iteration per day. > is my input somewhat inefficient? pl. let me suggest any > improvement if possible assuming that you cannot reduce the size cell and the cutoff: - do not calculate stress if you do not need to: it is expensive - conv_thr = 1.0d-10 is quite high, 1.0d-8 should be enough - is your system an isolated system? then use K_POINTS gamma : it will use k=0 only (which is all you need for an isolated system) and exploit various tricks to reduce memory usage - setting option "diago_david_ndim" to the minimum (2) and "mixing_ndim" to a smaller value (4) reduces memory usage, but may increase CPU time - using diagonalization='cg' will also reduce memory usage, but it will increase CPU time by a sizable amount P. --- Paolo Giannozzi, Dept of Physics, University of Udine via delle Scienze 208, 33100 Udine, Italy Phone +39-0432-558216, fax +39-0432-558222 _______________________________________________ Pw_forum mailing list Pw_forum at pwscf.org http://www.democritos.it/mailman/listinfo/pw_forum 5, 50, 500, 5000 - Store N number of mails in your inbox. Go to http://help.yahoo.com/l/in/yahoo/mail/yahoomail/tools/tools-08.html -------------- next part -------------- An HTML attachment was scrubbed... URL: http://www.democritos.it/pipermail/pw_forum/attachments/20080107/0946a3ec/attachment.htm From giannozz at nest.sns.it Mon Jan 7 10:13:12 2008 From: giannozz at nest.sns.it (Paolo Giannozzi) Date: Mon, 7 Jan 2008 10:13:12 +0100 Subject: [Pw_forum] time requirement In-Reply-To: <417443.36526.qm@web94107.mail.in2.yahoo.com> References: <417443.36526.qm@web94107.mail.in2.yahoo.com> Message-ID: On Jan 7, 2008, at 9:48 , Rudra Banerjee wrote: > but is it possible to calculate phonon after doing gamma point > relaxation? you can calculate phonons at q=0 only with the code contained in Gamma/ . Otherwise you can perform the structural optimization with Gamma and then make a single-point calculation as you like. Paolo --- Paolo Giannozzi, Dept of Physics, University of Udine via delle Scienze 208, 33100 Udine, Italy Phone +39-0432-558216, fax +39-0432-558222 From bnrj.rudra at yahoo.com Mon Jan 7 10:35:27 2008 From: bnrj.rudra at yahoo.com (Rudra Banerjee) Date: Mon, 7 Jan 2008 15:05:27 +0530 (IST) Subject: [Pw_forum] time requirement Message-ID: <111917.24816.qm@web94112.mail.in2.yahoo.com> Dear Giannozzi, thanks for your reply, but i didn't understand what you meant by single-point calculation. is it something like ,first we do a Gamma point relaxation and then do a SCF calculation with the optimised structure? -- Rudra Have a Nice Time http://www.bose.res.in/~rudra Please, if possible, don't send me MS Word or PowerPoint attachments Why?See: http://www.gnu.org/philosophy/no-word-attachments.html ----- Original Message ---- From: Paolo Giannozzi To: PWSCF Forum Sent: Monday, 7 January, 2008 2:43:12 PM Subject: Re: [Pw_forum] time requirement On Jan 7, 2008, at 9:48 , Rudra Banerjee wrote: > but is it possible to calculate phonon after doing gamma point > relaxation? you can calculate phonons at q=0 only with the code contained in Gamma/ . Otherwise you can perform the structural optimization with Gamma and then make a single-point calculation as you like. Paolo --- Paolo Giannozzi, Dept of Physics, University of Udine via delle Scienze 208, 33100 Udine, Italy Phone +39-0432-558216, fax +39-0432-558222 _______________________________________________ Pw_forum mailing list Pw_forum at pwscf.org http://www.democritos.it/mailman/listinfo/pw_forum Why delete messages? Unlimited storage is just a click away. Go to http://help.yahoo.com/l/in/yahoo/mail/yahoomail/tools/tools-08.html -------------- next part -------------- An HTML attachment was scrubbed... URL: http://www.democritos.it/pipermail/pw_forum/attachments/20080107/4fe3228a/attachment.htm From giannozz at nest.sns.it Mon Jan 7 11:30:00 2008 From: giannozz at nest.sns.it (Paolo Giannozzi) Date: Mon, 7 Jan 2008 11:30:00 +0100 Subject: [Pw_forum] time requirement In-Reply-To: <111917.24816.qm@web94112.mail.in2.yahoo.com> References: <111917.24816.qm@web94112.mail.in2.yahoo.com> Message-ID: On Jan 7, 2008, at 10:35 , Rudra Banerjee wrote: > i didn't understand what you meant by single-point calculation. quantum chemistry terminology for a scf calculation at fixed nuclei > is it something like ,first we do a Gamma point relaxation and > then do a SCF calculation with the optimised structure? exactly this Paolo --- Paolo Giannozzi, Dept of Physics, University of Udine via delle Scienze 208, 33100 Udine, Italy Phone +39-0432-558216, fax +39-0432-558222 From bnrj.rudra at yahoo.com Mon Jan 7 13:23:15 2008 From: bnrj.rudra at yahoo.com (Rudra Banerjee) Date: Mon, 7 Jan 2008 17:53:15 +0530 (IST) Subject: [Pw_forum] time requirement Message-ID: <574897.99056.qm@web94115.mail.in2.yahoo.com> thanks paolo...its running fine -- Rudra Have a Nice Time http://www.bose.res.in/~rudra Please, if possible, don't send me MS Word or PowerPoint attachments Why?See: http://www.gnu.org/philosophy/no-word-attachments.html ----- Original Message ---- From: Paolo Giannozzi To: PWSCF Forum Sent: Monday, 7 January, 2008 4:00:00 PM Subject: Re: [Pw_forum] time requirement On Jan 7, 2008, at 10:35 , Rudra Banerjee wrote: > i didn't understand what you meant by single-point calculation. quantum chemistry terminology for a scf calculation at fixed nuclei > is it something like ,first we do a Gamma point relaxation and > then do a SCF calculation with the optimised structure? exactly this Paolo --- Paolo Giannozzi, Dept of Physics, University of Udine via delle Scienze 208, 33100 Udine, Italy Phone +39-0432-558216, fax +39-0432-558222 _______________________________________________ Pw_forum mailing list Pw_forum at pwscf.org http://www.democritos.it/mailman/listinfo/pw_forum Share files, take polls, and discuss your passions - all under one roof. Go to http://in.promos.yahoo.com/groups -------------- next part -------------- An HTML attachment was scrubbed... URL: http://www.democritos.it/pipermail/pw_forum/attachments/20080107/37febca2/attachment-0001.htm From matteo at umn.edu Mon Jan 7 18:50:15 2008 From: matteo at umn.edu (Matteo Cococcioni) Date: Mon, 07 Jan 2008 11:50:15 -0600 Subject: [Pw_forum] Newbie Questions In-Reply-To: <6c65435e0801050859s4868ab06ma1d665b5c069ef60@mail.gmail.com> References: <6c65435e0801050859s4868ab06ma1d665b5c069ef60@mail.gmail.com> Message-ID: <47826657.7070506@umn.edu> Dear Jack, I'm not sure I understand all of your questions. probably other people in the forum can help you more with some of them. Jack Shultz wrote: > Hello, > > I am new to Quantume Espresso and hope I can integrate it into my > distributed computing project as I decribe in this post > http://www.hydrogenathome.org/forum_thread.php?id=173 > > As I am formulating my analytical methods, there may be flaws in what > I am proposing. I hope to use Climbing-Image Nudged Elastic Band > method to identify a transition state structure. Then use that > transition state structure in a molecular docking simulation. I then > run these simulations on metalic enzymes in hopes of identifing > optimal enzymes for water spliting hydrogen generating reactions. Of > course before I get that far I will run positve/negative controls to > QC my methods. Having resource sharing among volunteers through > Berkley's BOINC platform is really benefitial otherwise I would not > have the ability to do this kind of analysis on this scale. > > Anyway, my questions right now: > 1) What output formats does QE support? I'm trying to get the command > line for open babel to convert xyz to other molecular formats and it > is giving me a hard time but if QE can output to PDB or MOL2 or other > molecular file format, I would really like to know how. > I'm not sure about the specific characteristic of these formats. As you probably know QE is well interfaced with XCrysDen and you may consider using this visualization program. > 2) Are there command line tools that generate parameters for these > simulations like Exampl17? Where do I find them? I want to be able to > just supply a structure representing the substrate and another > representing the product of my reaction. I want to make these > parameters interchangeable because I am testing controls first. So I > would be able to test H2O2 binding to Catalase for instance. It's not totally clear to me what you want to do here and what parameters you need. energy cut offs? lattice spacings? Many of them are determined by convergence. so I guess what you need is a script doing these convergences? > > 3) Finally, I don't know how to pose this question, perhaps I need > read more, how does one integrate DFT+U into models of reaction pathways? > In the paper PRL 97 103001 that Nicola Marzari has already suggested to you NEB was used with LDA+U. You don't need any integration: you just run an LDA+U calculation for every image. Hope this is of some help. Matteo > Thank you in advance for any assistance. > > Jack Shultz > Project Leader http://hydrogenathome.org > > ------------------------------------------------------------------------ > > _______________________________________________ > Pw_forum mailing list > Pw_forum at pwscf.org > http://www.democritos.it/mailman/listinfo/pw_forum > -------------- next part -------------- A non-text attachment was scrubbed... Name: matteo.vcf Type: text/x-vcard Size: 294 bytes Desc: not available Url : http://www.democritos.it/pipermail/pw_forum/attachments/20080107/5e6733d1/attachment.vcf From jackygrahamez at gmail.com Mon Jan 7 19:40:14 2008 From: jackygrahamez at gmail.com (Jack Shultz) Date: Mon, 7 Jan 2008 13:40:14 -0500 Subject: [Pw_forum] Newbie Questions In-Reply-To: <47826657.7070506@umn.edu> References: <6c65435e0801050859s4868ab06ma1d665b5c069ef60@mail.gmail.com> <47826657.7070506@umn.edu> Message-ID: <6c65435e0801071040p51674602r1f3242360cf80a17@mail.gmail.com> Thank you Matteo. Axel and I had number of correspondences and he gave me answers and his opinion on some my questions. I'm seeing the multiple challenges involved with a high throughput analysis for identifying catalysts based on molecular data. I am getting help on babel, just having trouble with windows platform for now. Easier than writing my own script to convert formats. I am looking at different ways of combining estimates on transition state structures with molecular docking. Theoretically the transition state should be able to bind to the active site of an enzyme. I am going to look at computationally light methods that can run on a standard desktop at most three days. Perhaps something that gives similar but different transition state structures for each run. Starting off with simple catalysts and parameterizing these simulations. NEB might not be the best approach within the confines of an entire enzyme due to the degrees of freedom. If studied in isolation it may yield inaccurate results due to missing entropy. Maybe it is close enough. Otherwise i could have substrate binding to the catalyst then only look at the 30 closest atoms near the binding site. Hopefully DFT plane-wave pseudo potentials are not too massive a computational task with that many atoms. I really just learning this stuff. Or I can use semi-empirical QM/MM to do either of those types of experiments. At first I only plan to look at simple one step reactions. Either zero or first order. Would it be easier to analyze zero order or do those have a saddle point energy? Anyway, I hear there are many theoretical problems with DFTs for the purpose I'm exploring. At the very least I hope to see what those problems are for my own edification. Where do I look for information on LDA+U calculations? Is it in journals that I have buy or is it posted somewhere? Jack On Jan 7, 2008 12:50 PM, Matteo Cococcioni wrote: > Dear Jack, > > I'm not sure I understand all of your questions. probably other people > in the forum can help you more with some of them. > > Jack Shultz wrote: > > Hello, > > > > I am new to Quantume Espresso and hope I can integrate it into my > > distributed computing project as I decribe in this post > > http://www.hydrogenathome.org/forum_thread.php?id=173 > > > > As I am formulating my analytical methods, there may be flaws in what > > I am proposing. I hope to use Climbing-Image Nudged Elastic Band > > method to identify a transition state structure. Then use that > > transition state structure in a molecular docking simulation. I then > > run these simulations on metalic enzymes in hopes of identifing > > optimal enzymes for water spliting hydrogen generating reactions. Of > > course before I get that far I will run positve/negative controls to > > QC my methods. Having resource sharing among volunteers through > > Berkley's BOINC platform is really benefitial otherwise I would not > > have the ability to do this kind of analysis on this scale. > > > > Anyway, my questions right now: > > 1) What output formats does QE support? I'm trying to get the command > > line for open babel to convert xyz to other molecular formats and it > > is giving me a hard time but if QE can output to PDB or MOL2 or other > > molecular file format, I would really like to know how. > > > > I'm not sure about the specific characteristic of these formats. As you > probably know QE is well interfaced with XCrysDen and you may consider > using this visualization program. > > > 2) Are there command line tools that generate parameters for these > > simulations like Exampl17? Where do I find them? I want to be able to > > just supply a structure representing the substrate and another > > representing the product of my reaction. I want to make these > > parameters interchangeable because I am testing controls first. So I > > would be able to test H2O2 binding to Catalase for instance. > > It's not totally clear to me what you want to do here and what > parameters you need. energy cut offs? lattice spacings? Many of them are > determined by convergence. so I guess what you need is a script doing > these convergences? > > > > > 3) Finally, I don't know how to pose this question, perhaps I need > > read more, how does one integrate DFT+U into models of reaction > pathways? > > > > In the paper PRL 97 103001 that Nicola Marzari has already suggested to > you NEB was used with LDA+U. You don't need any integration: you just > run an LDA+U calculation for every image. > > Hope this is of some help. > > Matteo > > > > > Thank you in advance for any assistance. > > > > Jack Shultz > > Project Leader http://hydrogenathome.org > > > > ------------------------------------------------------------------------ > > > > _______________________________________________ > > Pw_forum mailing list > > Pw_forum at pwscf.org > > http://www.democritos.it/mailman/listinfo/pw_forum > > > > > _______________________________________________ > Pw_forum mailing list > Pw_forum at pwscf.org > http://www.democritos.it/mailman/listinfo/pw_forum > > -------------- next part -------------- An HTML attachment was scrubbed... URL: http://www.democritos.it/pipermail/pw_forum/attachments/20080107/ab48b649/attachment.htm From giannozz at nest.sns.it Mon Jan 7 23:21:00 2008 From: giannozz at nest.sns.it (Paolo Giannozzi) Date: Mon, 7 Jan 2008 23:21:00 +0100 Subject: [Pw_forum] Newbie Questions In-Reply-To: <6c65435e0801050859s4868ab06ma1d665b5c069ef60@mail.gmail.com> References: <6c65435e0801050859s4868ab06ma1d665b5c069ef60@mail.gmail.com> Message-ID: <11C3C58A-CD13-433D-AE5A-623AF57309C1@nest.sns.it> On Jan 5, 2008, at 17:59 , Jack Shultz wrote: > 1) What output formats does QE support? basically its own :-( but it should be easy to write output in a different format, if needed. Input, output, and data file format are old and still open problems... P. --- Paolo Giannozzi, Dept of Physics, University of Udine via delle Scienze 208, 33100 Udine, Italy Phone +39-0432-558216, fax +39-0432-558222 From bnrj.rudra at yahoo.com Wed Jan 9 08:14:27 2008 From: bnrj.rudra at yahoo.com (Rudra Banerjee) Date: Wed, 9 Jan 2008 12:44:27 +0530 (IST) Subject: [Pw_forum] compilation problem Message-ID: <298301.84914.qm@web94115.mail.in2.yahoo.com> when i am trying to install the 3.2.3 version in parallel mode,its shwing error: espresso-3.2.3/bin/pw.x not existent or not executable the problem is over if i chmod the source file i.e. PW/pw.x to 777....but what i am wondering is it was not a problem of the serial version....so whats wrong is going? -- Rudra Have a Nice Time http://www.bose.res.in/~rudra Please, if possible, don't send me MS Word or PowerPoint attachments Why?See: http://www.gnu.org/philosophy/no-word-attachments.html Why delete messages? Unlimited storage is just a click away. Go to http://help.yahoo.com/l/in/yahoo/mail/yahoomail/tools/tools-08.html -------------- next part -------------- An HTML attachment was scrubbed... URL: http://www.democritos.it/pipermail/pw_forum/attachments/20080109/2c2ed02a/attachment.htm From akohlmey at cmm.chem.upenn.edu Wed Jan 9 17:04:05 2008 From: akohlmey at cmm.chem.upenn.edu (Axel Kohlmeyer) Date: Wed, 9 Jan 2008 11:04:05 -0500 (EST) Subject: [Pw_forum] compilation problem In-Reply-To: <298301.84914.qm@web94115.mail.in2.yahoo.com> References: <298301.84914.qm@web94115.mail.in2.yahoo.com> Message-ID: On Wed, 9 Jan 2008, Rudra Banerjee wrote: RB> when i am trying to install the 3.2.3 version in parallel mode,its shwing error: espresso-3.2.3/bin/pw.x not existent or not executable RB> the problem is over if i chmod the source file i.e. PW/pw.x to 777....but what i am wondering is it was not a problem of the serial version....so whats wrong is going? rudra, if the compiler does not set the permissions of a final link to be executable, than something went wrong while linking. you'll have to delete the .x file and "make" again and watch the output of the compilation very carefully. there have to be some warnings or error messages. look them up in the manual or on the web and see if you can find out what went wrong. 9 out of 10 times, it is related to an incorrect or corrupted installation or setup of the individual machine. cheers, axel. -- Rudra Have a Nice Time http://www.bose.res.in/~rudra Please, if possible, don't send me MS Word or PowerPoint attachments Why?See: http://www.gnu.org/philosophy/no-word-attachments.html Why delete messages? Unlimited storage is just a click away. Go to http://help.yahoo.com/l/in/yahoo/mail/yahoomail/tools/tools-08.html -- ======================================================================= Axel Kohlmeyer akohlmey at cmm.chem.upenn.edu http://www.cmm.upenn.edu Center for Molecular Modeling -- University of Pennsylvania Department of Chemistry, 231 S.34th Street, Philadelphia, PA 19104-6323 tel: 1-215-898-1582, fax: 1-215-573-6233, office-tel: 1-215-898-5425 ======================================================================= If you make something idiot-proof, the universe creates a better idiot. From ustc0200 at gmail.com Thu Jan 10 05:12:35 2008 From: ustc0200 at gmail.com (Zhu Xi) Date: Thu, 10 Jan 2008 12:12:35 +0800 Subject: [Pw_forum] energy of 33CNT Message-ID: Dear users, I run a scf calculation of (3,3)CNT, but the fermi energy is 0.652eV, can anyone give some suggestion? thanks ==================================================================================== &control calculation='scf', PSEUDO_DIR='./', prefix='33', forc_conv_thr=1.0D-4, / &SYSTEM ibrav = 0, celldm(1)= 18.8972612499, nat = 12, ntyp = 1, ecutwfc = 40.0 , nosym = .true. , nbnd = 48 , occupations= 'smearing',degauss = 0.001 / &electrons conv_thr = 1.D-6, mixing_beta = 0.5D0, / CELL_PARAMETERS 1.0 0.000000000 0.000000000 0.000000000 1.0 0.000000000 0.000000000 0.000000000 0.245951 ATOMIC_SPECIES C 12.00000 C.pz-vbc.UPF ATOMIC_POSITIONS (crystal) C 0.625289967 0.669611155 0.250000000 C 0.487077242 0.709466388 0.250000000 C 0.291384631 0.523168925 0.250000000 C 0.374710033 0.330388845 0.750000000 C 0.512922758 0.290533612 0.750000000 C 0.708615369 0.476831075 0.750000000 C 0.416429985 0.692001578 0.750000000 C 0.675464001 0.616817067 0.750000000 C 0.312292454 0.592869751 0.750000000 C 0.583570015 0.307998422 0.250000000 C 0.324535999 0.383182933 0.250000000 C 0.687707546 0.407130249 0.250000000 K_POINTS automatic 1 1 11 0 0 0 ================================================== -- Zhu Xi University of Science and Technology of China. -------------- next part -------------- An HTML attachment was scrubbed... URL: http://www.democritos.it/pipermail/pw_forum/attachments/20080110/194f5d83/attachment.htm From nedward at MIT.EDU Thu Jan 10 05:57:35 2008 From: nedward at MIT.EDU (Nicholas E. Singh-Miller) Date: Wed, 9 Jan 2008 23:57:35 -0500 (EST) Subject: [Pw_forum] energy of 33CNT In-Reply-To: References: Message-ID: Dear Zhu Xi, I have not run your input file, but assuming that it is correct and this 0.625eV is what you read directly from the output, this Fermi energy should also be correct. If you are interested in the work function of the (3,3)CNT you will have to obtain the difference between a vacuum potential and the Fermi energy. In short the potential in the vacuum != 0 due to the use of periodic boundary conditions. If you use pp.x to obtain the potential remember that is is output in Rydbergs. This is all discussed other times on the forum. best of luck, Nicholas On Thu, 10 Jan 2008, Zhu Xi wrote: > Dear users, > > I run a scf calculation of (3,3)CNT, but the fermi energy is 0.652eV, can > anyone give some suggestion? thanks > > ==================================================================================== > > &control > calculation='scf', > PSEUDO_DIR='./', > prefix='33', > forc_conv_thr=1.0D-4, > / > &SYSTEM > ibrav = 0, > celldm(1)= 18.8972612499, > nat = 12, > ntyp = 1, > ecutwfc = 40.0 , > nosym = .true. , > nbnd = 48 , > occupations= 'smearing',degauss = 0.001 > > / > &electrons > conv_thr = 1.D-6, > mixing_beta = 0.5D0, > / > CELL_PARAMETERS > 1.0 0.000000000 0.000000000 > 0.000000000 1.0 0.000000000 > 0.000000000 0.000000000 0.245951 > ATOMIC_SPECIES > C 12.00000 C.pz-vbc.UPF > ATOMIC_POSITIONS (crystal) > C 0.625289967 0.669611155 0.250000000 > C 0.487077242 0.709466388 0.250000000 > C 0.291384631 0.523168925 0.250000000 > C 0.374710033 0.330388845 0.750000000 > C 0.512922758 0.290533612 0.750000000 > C 0.708615369 0.476831075 0.750000000 > C 0.416429985 0.692001578 0.750000000 > C 0.675464001 0.616817067 0.750000000 > C 0.312292454 0.592869751 0.750000000 > C 0.583570015 0.307998422 0.250000000 > C 0.324535999 0.383182933 0.250000000 > C 0.687707546 0.407130249 0.250000000 > K_POINTS automatic > 1 1 11 0 0 0 > > ================================================== > ***************************************** Nicholas E. Singh-Miller Ph.D. Candidate Prof. Marzari Group (quasiamore.mit.edu) Materials Science and Engineering Massachusetts Institute of Technology 13-4066 (617)324-0372 ***************************************** From w2agz at pacbell.net Thu Jan 10 06:22:20 2008 From: w2agz at pacbell.net (Paul M. Grant) Date: Wed, 9 Jan 2008 21:22:20 -0800 Subject: [Pw_forum] CuO, LDA+U, Pseudopots, again Message-ID: <000e01c85348$c714d0c0$553e7240$@net> To All: Please refer to a previous pw_forum thread "CuO, LDA+U, Pseudopots, etc." I'm continuing to try to achieve a pwscf calculation which will yield a "Hubbard gap" in any compound containing Cu++ and O--. My efforts to date have been unsucessful. My "model" has been a pseudo-rocksalt CuO structure based on "proxy" lattice constants near those of FeO, NiO and MgO. I strongly suspect my problem involves use (or misuse or unavailability) of a proper Cu pseudopotential. The issue of Cu++ bonded to O-- is central to the physics of high temperature superconductivity, and a failure of pwscf to properly resolve this difficulty would be detrimential to the future deployment of quantum-espresso to this very important and critical area of computational materials physics, not just HTSC, but possibly GMR structures as well. Note in what follows is that I changed occ_loc in tabd.f90 from 10 to 9. I attach below 1) the list of PPs I've tried and their d-level configurations. None really contain the generally assumed 3d9 Hund's Rule configuration for CuO complexes. Following this is the output 2) of a "hand written grep-er" of the scf.out file for a typical example (one of about 20) combinations of Cu PPs and the "starting_ns_eigenvalue suggestions" to escape the "minimum energy trap." I selected my "best attempt" at this effort. The critical (IMHO) lines are pointed to by a "-->" marker. BTW, I didn't try any Wannier projections since the eigenvalue forcings didn't work, and my lesson from Example 25 is that these have to be done successfully first. Any of those in the pwscf community aware of a "better" PP for my problem, please e-mail such to me. In the meantime, I'm experimenting "rolling my own" with the tools available at the Berlin-FHI site, trying 3d9-4s2 as a first attempt--stay tuned. Sorry for all my American slang. We didn't invent the language...the Brits did, and shoved it down our throats. We just made English more user friendly. -Paul ---------------------------------- 1) Cu.blyp-n-van_ak.UPF nl pn l occ 3D 3 2 9.50 4S 4 0 1.00 4P 4 1 0.50 Cu.pbe-d-rrkjus.UPF nl pn l occ 3D 3 2 10.00 3D 3 2 0.00 4P 2 1 -1.00 4S 1 0 1.00 Cu.pbe-n-van_ak.UPF nl pn l occ 3D 3 2 9.50 4S 4 0 1.00 4P 4 1 0.50 Cu.pw91-n-van_ak.UPF nl pn l occ 3D 3 2 9.50 4S 4 0 1.00 4P 4 1 0.50 Cu.pz-d-rrkjus.UPF nl pn l occ 3D 3 2 10.00 3D 3 2 0.00 4P 2 1 -1.00 4S 1 0 1.00 Cu.pz-n-van_ak.UPF nl pn l occ 3D 3 2 9.50 4S 4 0 1.00 4P 4 1 0.50 --------------------------------------- 2) Today is Wed Jan 9 19:07:17 PST 2008 Now is Wed Jan 9 19:10:07 PST 2008 SCF Run preparatory to post-processing Script is ./CuO_3.scf.sh, Output is CuO_3.scf.data kinetic-energy cutoff = 30.0000 Ry charge density cutoff = 240.0000 Ry convergence threshold = 1.0E-06 --> pwscf_exec = /home/pmpgrant/O-sesame/PW/pw.x a_Ang = 4.200 a_Bohr = 7.93950 nbnd = 20, nelec = , occupations = 'smearing', smearing = gauss, degauss = 0.01, la2F = .false., diagonalization = 'david', starting_magnetization(1) = 0.0 starting_magnetization(2) = 0.5 starting_magnetization(3) = -0.5 nspin = 2, lda_plus_u = .true., --> Hubbard_U(2) = 5.0 --> Hubbard_U(3) = 5.0 --> U_projection_type = norm-atomic --> starting_ns_eigenvalue(1,2,2) = 0.0 starting_ns_eigenvalue(2,2,2) = starting_ns_eigenvalue(3,2,2) = starting_ns_eigenvalue(4,2,2) = starting_ns_eigenvalue(5,2,2) = --> starting_ns_eigenvalue(1,1,3) = 0.0 starting_ns_eigenvalue(2,1,3) = starting_ns_eigenvalue(3,1,3) = starting_ns_eigenvalue(4,1,3) = starting_ns_eigenvalue(5,1,3) = --> TM_pseudopot = Cu.pz-d-rrkjus.UPF --> A_pseudopot = O.LDA.US.RRKJ3.UPF Run_Type npk Ef(eV) Total E(Ry) CPU_Time Wall_Time scf 8 11.1457 -239.16751151 1m 8.04s 2m50.40s d-orbital occupations 138:eigenvalues: 1.0000000 1.0000000 1.0000000 1.0000000 1.0000000 ==> 152:eigenvalues: 0.8000000 0.8000000 0.8000000 0.8000000 0.8000000 ==> 167:eigenvalues: 0.8000000 0.8000000 0.8000000 0.8000000 0.8000000 181:eigenvalues: 1.0000000 1.0000000 1.0000000 1.0000000 1.0000000 213:eigenvalues: 0.9976937 0.9976937 0.9984132 0.9984132 1.0008045 227:eigenvalues: 0.5164626 0.5164626 0.9680621 0.9823458 0.9823458 242:eigenvalues: 0.5164626 0.5164626 0.9680621 0.9823458 0.9823458 256:eigenvalues: 0.9976937 0.9976937 0.9984132 0.9984132 1.0008045 277:eigenvalues: 0.9976937 0.9976937 0.9984132 0.9984132 1.0008045 --> 291:eigenvalues: 0.0000000 0.5164626 0.9680621 0.9823458 0.9823458 --> 306:eigenvalues: 0.0000000 0.5164626 0.9680621 0.9823458 0.9823458 320:eigenvalues: 0.9976937 0.9976937 0.9984132 0.9984132 1.0008045 353:eigenvalues: 0.9936756 0.9936756 0.9968054 0.9968054 0.9991184 --> 367:eigenvalues: 0.7870114 0.7870114 0.9938818 0.9938818 0.9969944 --> 382:eigenvalues: 0.7867963 0.7867963 0.9938806 0.9938806 0.9969906 396:eigenvalues: 0.9936787 0.9936787 0.9968055 0.9968055 0.9991185 429:eigenvalues: 0.9914062 0.9914062 0.9971611 0.9971611 0.9994970 443:eigenvalues: 0.7968500 0.7968500 0.9944826 0.9944826 0.9975186 458:eigenvalues: 0.7971215 0.7971215 0.9944915 0.9944915 0.9975167 472:eigenvalues: 0.9914126 0.9914126 0.9971603 0.9971603 0.9994966 507:eigenvalues: 0.9667233 0.9667233 0.9969086 0.9969086 0.9992909 521:eigenvalues: 0.8723219 0.8723219 0.9959798 0.9959798 0.9984415 536:eigenvalues: 0.8713559 0.8713559 0.9959649 0.9959649 0.9984298 550:eigenvalues: 0.9663176 0.9663176 0.9968918 0.9968918 0.9992743 585:eigenvalues: 0.9525509 0.9525509 0.9963181 0.9963181 0.9986638 599:eigenvalues: 0.9127300 0.9127300 0.9961748 0.9961748 0.9985388 614:eigenvalues: 0.9146296 0.9146296 0.9962191 0.9962191 0.9985819 628:eigenvalues: 0.9537915 0.9537915 0.9963783 0.9963783 0.9987221 663:eigenvalues: 0.9458265 0.9458265 0.9962868 0.9962868 0.9986542 677:eigenvalues: 0.9104092 0.9104092 0.9959371 0.9959371 0.9983246 692:eigenvalues: 0.9047312 0.9047312 0.9957990 0.9957990 0.9982026 706:eigenvalues: 0.9417229 0.9417229 0.9961282 0.9961282 0.9984984 741:eigenvalues: 0.9354950 0.9354950 0.9960520 0.9960520 0.9984263 755:eigenvalues: 0.9174694 0.9174694 0.9958534 0.9958534 0.9982404 770:eigenvalues: 0.9179431 0.9179431 0.9958601 0.9958601 0.9982469 784:eigenvalues: 0.9358828 0.9358828 0.9960625 0.9960625 0.9984347 819:eigenvalues: 0.9341625 0.9341625 0.9960316 0.9960316 0.9984028 833:eigenvalues: 0.9222122 0.9222122 0.9959062 0.9959062 0.9982848 848:eigenvalues: 0.9216331 0.9216331 0.9958882 0.9958882 0.9982692 862:eigenvalues: 0.9336231 0.9336231 0.9960127 0.9960127 0.9983843 897:eigenvalues: 0.9327282 0.9327282 0.9960266 0.9960266 0.9983951 911:eigenvalues: 0.9284646 0.9284646 0.9959858 0.9959858 0.9983585 926:eigenvalues: 0.9283145 0.9283145 0.9959811 0.9959811 0.9983538 940:eigenvalues: 0.9325413 0.9325413 0.9960218 0.9960218 0.9983904 975:eigenvalues: 0.9318453 0.9318453 0.9960271 0.9960271 0.9983966 989:eigenvalues: 0.9297349 0.9297349 0.9960062 0.9960062 0.9983772 1004:eigenvalues: 0.9299729 0.9299729 0.9960136 0.9960136 0.9983841 1018:eigenvalues: 0.9320457 0.9320457 0.9960350 0.9960350 0.9984044 1053:eigenvalues: 0.9311035 0.9311035 0.9960273 0.9960273 0.9983963 1067:eigenvalues: 0.9316853 0.9316853 0.9960330 0.9960330 0.9984016 1082:eigenvalues: 0.9316968 0.9316968 0.9960336 0.9960336 0.9984023 1096:eigenvalues: 0.9310994 0.9310994 0.9960275 0.9960275 0.9983966 --> 1131:eigenvalues: 0.9314650 0.9314650 0.9960338 0.9960338 0.9984027 --> 1145:eigenvalues: 0.9312033 0.9312033 0.9960312 0.9960312 0.9984004 --> 1160:eigenvalues: 0.9311989 0.9311989 0.9960309 0.9960309 0.9984001 --> 1174:eigenvalues: 0.9314517 0.9314517 0.9960337 0.9960337 0.9984026 Paul M. Grant, PhD Principal, W2AGZ Technologies Visiting Scholar, Applied Physics, Stanford University EPRI Science Fellow (Retired) IBM Research Staff Member Emeritus w2agz at pacbell.net http://www.w2agz.com -------------- next part -------------- An HTML attachment was scrubbed... URL: http://www.democritos.it/pipermail/pw_forum/attachments/20080109/1ca8de8a/attachment-0001.htm From ustc0200 at gmail.com Thu Jan 10 06:37:06 2008 From: ustc0200 at gmail.com (Zhu Xi) Date: Thu, 10 Jan 2008 13:37:06 +0800 Subject: [Pw_forum] energy of 33CNT In-Reply-To: References: Message-ID: Hi Nicholas, thanks for your reply, but to my understaning, the 0.625eV fermi energy is wrong, it should be between -5~-4eV, I don't know why the code give such a fermi energy..... On 1/10/08, Nicholas E. Singh-Miller wrote: > > Dear Zhu Xi, > > I have not run your input file, but assuming that it is correct and this > 0.625eV is what you read directly from the output, this Fermi energy > should also be correct. If you are interested in the work function of > the (3,3)CNT you will have to obtain the difference between a vacuum > potential and the Fermi energy. In short the potential in the vacuum != 0 > due to the use of periodic boundary conditions. If you use pp.x to > obtain the potential remember that is is output in Rydbergs. This is all > discussed other times on the forum. > > best of luck, > > Nicholas > > On Thu, 10 Jan 2008, Zhu Xi wrote: > > > Dear users, > > > > I run a scf calculation of (3,3)CNT, but the fermi energy is 0.652eV, > can > > anyone give some suggestion? thanks > > > > > ==================================================================================== > > > > &control > > calculation='scf', > > PSEUDO_DIR='./', > > prefix='33', > > forc_conv_thr=1.0D-4, > > / > > &SYSTEM > > ibrav = 0, > > celldm(1)= 18.8972612499, > > nat = 12, > > ntyp = 1, > > ecutwfc = 40.0 , > > nosym = .true. , > > nbnd = 48 , > > occupations= 'smearing',degauss = 0.001 > > > > / > > &electrons > > conv_thr = 1.D-6, > > mixing_beta = 0.5D0, > > / > > CELL_PARAMETERS > > 1.0 0.000000000 0.000000000 > > 0.000000000 1.0 0.000000000 > > 0.000000000 0.000000000 0.245951 > > ATOMIC_SPECIES > > C 12.00000 C.pz-vbc.UPF > > ATOMIC_POSITIONS (crystal) > > C 0.625289967 0.669611155 0.250000000 > > C 0.487077242 0.709466388 0.250000000 > > C 0.291384631 0.523168925 0.250000000 > > C 0.374710033 0.330388845 0.750000000 > > C 0.512922758 0.290533612 0.750000000 > > C 0.708615369 0.476831075 0.750000000 > > C 0.416429985 0.692001578 0.750000000 > > C 0.675464001 0.616817067 0.750000000 > > C 0.312292454 0.592869751 0.750000000 > > C 0.583570015 0.307998422 0.250000000 > > C 0.324535999 0.383182933 0.250000000 > > C 0.687707546 0.407130249 0.250000000 > > K_POINTS automatic > > 1 1 11 0 0 0 > > > > ================================================== > > > > ***************************************** > Nicholas E. Singh-Miller > Ph.D. Candidate > Prof. Marzari Group (quasiamore.mit.edu) > Materials Science and Engineering > Massachusetts Institute of Technology > 13-4066 > (617)324-0372 > ***************************************** > _______________________________________________ > Pw_forum mailing list > Pw_forum at pwscf.org > http://www.democritos.it/mailman/listinfo/pw_forum > -- X -------------- next part -------------- An HTML attachment was scrubbed... URL: http://www.democritos.it/pipermail/pw_forum/attachments/20080110/d206c17d/attachment.htm From lanhaiping at gmail.com Thu Jan 10 07:25:05 2008 From: lanhaiping at gmail.com (lan haiping) Date: Thu, 10 Jan 2008 07:25:05 +0100 Subject: [Pw_forum] energy of 33CNT In-Reply-To: References: Message-ID: Hi, Zhu Xi to my knowledge , we always compare workfunctions of nanotubes with experimental data, but not fermi level ! In order to check your result, you can follow nicolas's suggestion : using pp.x to obtain the vacuum level, the difference between fermi level and vacuum level is workfunction of nanotube. please note, due to periodic images' interaction, the cell size of your system will affect the final result you obtain. Bests, Hai-Ping On Jan 10, 2008 6:37 AM, Zhu Xi wrote: > Hi Nicholas, > thanks for your reply, but to my understaning, the 0.625eV fermi energy is > wrong, it should be between -5~-4eV, I don't know why the code give such a > fermi energy..... > > > On 1/10/08, Nicholas E. Singh-Miller wrote: > > > > Dear Zhu Xi, > > > > I have not run your input file, but assuming that it is correct and this > > 0.625eV is what you read directly from the output, this Fermi energy > > should also be correct. If you are interested in the work function of > > the (3,3)CNT you will have to obtain the difference between a vacuum > > potential and the Fermi energy. In short the potential in the vacuum != > > 0 > > due to the use of periodic boundary conditions. If you use pp.x to > > obtain the potential remember that is is output in Rydbergs. This is > > all > > discussed other times on the forum. > > > > best of luck, > > > > Nicholas > > > > On Thu, 10 Jan 2008, Zhu Xi wrote: > > > > > Dear users, > > > > > > I run a scf calculation of (3,3)CNT, but the fermi energy is 0.652eV, > > can > > > anyone give some suggestion? thanks > > > > > > > > ==================================================================================== > > > > > > > > &control > > > calculation='scf', > > > PSEUDO_DIR='./', > > > prefix='33', > > > forc_conv_thr=1.0D-4, > > > / > > > &SYSTEM > > > ibrav = 0, > > > celldm(1)= 18.8972612499, > > > nat = 12, > > > ntyp = 1, > > > ecutwfc = 40.0 , > > > nosym = .true. , > > > nbnd = 48 , > > > occupations= 'smearing',degauss = 0.001 > > > > > > / > > > &electrons > > > conv_thr = 1.D-6, > > > mixing_beta = 0.5D0, > > > / > > > CELL_PARAMETERS > > > 1.0 0.000000000 0.000000000 > > > 0.000000000 1.0 0.000000000 > > > 0.000000000 0.000000000 0.245951 > > > ATOMIC_SPECIES > > > C 12.00000 C.pz-vbc.UPF > > > ATOMIC_POSITIONS (crystal) > > > C 0.625289967 0.669611155 0.250000000 > > > C 0.487077242 0.709466388 0.250000000 > > > C 0.291384631 0.523168925 0.250000000 > > > C 0.374710033 0.330388845 0.750000000 > > > C 0.512922758 0.290533612 0.750000000 > > > C 0.708615369 0.476831075 0.750000000 > > > C 0.416429985 0.692001578 0.750000000 > > > C 0.675464001 0.616817067 0.750000000 > > > C 0.312292454 0.592869751 0.750000000 > > > C 0.583570015 0.307998422 0.250000000 > > > C 0.324535999 0.383182933 0.250000000 > > > C 0.687707546 0.407130249 0.250000000 > > > K_POINTS automatic > > > 1 1 11 0 0 0 > > > > > > ================================================== > > > > > > > ***************************************** > > Nicholas E. Singh-Miller > > Ph.D. Candidate > > Prof. Marzari Group ( quasiamore.mit.edu) > > Materials Science and Engineering > > Massachusetts Institute of Technology > > 13-4066 > > (617)324-0372 > > ***************************************** > > _______________________________________________ > > Pw_forum mailing list > > Pw_forum at pwscf.org > > http://www.democritos.it/mailman/listinfo/pw_forum > > > > > > -- > X > _______________________________________________ > Pw_forum mailing list > Pw_forum at pwscf.org > http://www.democritos.it/mailman/listinfo/pw_forum > > -- Hai-Ping Lan Department of Electronics , Peking University , Bejing, 100871 lanhaiping at gmail.com, hplan at pku.edu.cn -------------- next part -------------- An HTML attachment was scrubbed... URL: http://www.democritos.it/pipermail/pw_forum/attachments/20080110/e91eb7ee/attachment.htm From nedward at MIT.EDU Thu Jan 10 07:28:34 2008 From: nedward at MIT.EDU (Nicholas E. Singh-Miller) Date: Thu, 10 Jan 2008 01:28:34 -0500 (EST) Subject: [Pw_forum] energy of 33CNT In-Reply-To: References: Message-ID: Hi Zhu, The Fermi energy (and the Kohn-Sham eigenvalues for that matter) found in the output file are referenced specifially to the potential for the calculation. So to recover the 4~5 eV that you are expecting you need to determine the vacuum potential. hope that helps, Nicholas On Thu, 10 Jan 2008, Zhu Xi wrote: > Hi Nicholas, > thanks for your reply, but to my understaning, the 0.625eV fermi energy is > wrong, it should be between -5~-4eV, I don't know why the code give such a > fermi energy..... > > > On 1/10/08, Nicholas E. Singh-Miller wrote: >> >> Dear Zhu Xi, >> >> I have not run your input file, but assuming that it is correct and this >> 0.625eV is what you read directly from the output, this Fermi energy >> should also be correct. If you are interested in the work function of >> the (3,3)CNT you will have to obtain the difference between a vacuum >> potential and the Fermi energy. In short the potential in the vacuum != 0 >> due to the use of periodic boundary conditions. If you use pp.x to >> obtain the potential remember that is is output in Rydbergs. This is all >> discussed other times on the forum. >> >> best of luck, >> >> Nicholas >> >> On Thu, 10 Jan 2008, Zhu Xi wrote: >> >>> Dear users, >>> >>> I run a scf calculation of (3,3)CNT, but the fermi energy is 0.652eV, >> can >>> anyone give some suggestion? thanks >>> >>> >> ==================================================================================== >>> >>> &control >>> calculation='scf', >>> PSEUDO_DIR='./', >>> prefix='33', >>> forc_conv_thr=1.0D-4, >>> / >>> &SYSTEM >>> ibrav = 0, >>> celldm(1)= 18.8972612499, >>> nat = 12, >>> ntyp = 1, >>> ecutwfc = 40.0 , >>> nosym = .true. , >>> nbnd = 48 , >>> occupations= 'smearing',degauss = 0.001 >>> >>> / >>> &electrons >>> conv_thr = 1.D-6, >>> mixing_beta = 0.5D0, >>> / >>> CELL_PARAMETERS >>> 1.0 0.000000000 0.000000000 >>> 0.000000000 1.0 0.000000000 >>> 0.000000000 0.000000000 0.245951 >>> ATOMIC_SPECIES >>> C 12.00000 C.pz-vbc.UPF >>> ATOMIC_POSITIONS (crystal) >>> C 0.625289967 0.669611155 0.250000000 >>> C 0.487077242 0.709466388 0.250000000 >>> C 0.291384631 0.523168925 0.250000000 >>> C 0.374710033 0.330388845 0.750000000 >>> C 0.512922758 0.290533612 0.750000000 >>> C 0.708615369 0.476831075 0.750000000 >>> C 0.416429985 0.692001578 0.750000000 >>> C 0.675464001 0.616817067 0.750000000 >>> C 0.312292454 0.592869751 0.750000000 >>> C 0.583570015 0.307998422 0.250000000 >>> C 0.324535999 0.383182933 0.250000000 >>> C 0.687707546 0.407130249 0.250000000 >>> K_POINTS automatic >>> 1 1 11 0 0 0 >>> >>> ================================================== >>> >> >> ***************************************** >> Nicholas E. Singh-Miller >> Ph.D. Candidate >> Prof. Marzari Group (quasiamore.mit.edu) >> Materials Science and Engineering >> Massachusetts Institute of Technology >> 13-4066 >> (617)324-0372 >> ***************************************** >> _______________________________________________ >> Pw_forum mailing list >> Pw_forum at pwscf.org >> http://www.democritos.it/mailman/listinfo/pw_forum >> > > > > ***************************************** Nicholas E. Singh-Miller Ph.D. Candidate Prof. Marzari Group (quasiamore.mit.edu) Materials Science and Engineering Massachusetts Institute of Technology 13-4066 (617)324-0372 ***************************************** From chen_shao_hua197 at yahoo.com.tw Thu Jan 10 21:21:33 2008 From: chen_shao_hua197 at yahoo.com.tw (=?big5?q?=B3=AF=20=A4=D6=B5=D8?=) Date: Fri, 11 Jan 2008 04:21:33 +0800 (CST) Subject: [Pw_forum] q2r.f90 error message "wrong nb read" Message-ID: <331256.51182.qm@web73011.mail.tp2.yahoo.com> Dear Forum's members, There is a error message "wrong nb read" in the In.q2r.out . I check its meaning in q2r.f90 . But I am not good at fortran's program. Does anyone could tell me how to avoid this error ? Thanks in advance max Physics department, National Taiwan University,Taiwan P.S. In.q2r.in &input zasr='simple', fildyn='In.dyn', flfrc='In.fc', la2F=.true. / ____________________________________________________________________________________ ????????????????http://tw.mini.yahoo.com/ From eyvaz_isaev at yahoo.com Fri Jan 11 10:39:10 2008 From: eyvaz_isaev at yahoo.com (Eyvaz Isaev) Date: Fri, 11 Jan 2008 01:39:10 -0800 (PST) Subject: [Pw_forum] q2r.f90 error message "wrong nb read" In-Reply-To: <331256.51182.qm@web73011.mail.tp2.yahoo.com> Message-ID: <972384.58081.qm@web60322.mail.yahoo.com> Hi, It seems someting wrong in your dynamical matrix. Check carefully your input file (atomic positions and types) and dymanical matrices. The error happens when there is a mismatch between atomic types. Bests, Eyvaz. --- ?? ???? wrote: > Dear Forum's members, > > There is a error message "wrong nb read" in the > In.q2r.out . > I check its meaning in q2r.f90 . > But I am not good at fortran's program. > Does anyone could tell me how to avoid this error ? > > Thanks in advance > > max > Physics department, National Taiwan > University,Taiwan > > P.S. In.q2r.in > &input > zasr='simple', fildyn='In.dyn', flfrc='In.fc', > la2F=.true. > / > > > > ____________________________________________________________________________________ > ?K?O?U???g?A???A???^???????P?d?Ihttp://tw.mini.yahoo.com/ > > _______________________________________________ > Pw_forum mailing list > Pw_forum at pwscf.org > http://www.democritos.it/mailman/listinfo/pw_forum > ------------------------------------------------------------------- Prof. Eyvaz Isaev, Theoretical Physics Department, Moscow State Institute of Steel & Alloys, Russia, and Condensed Matter Theory Group, Uppsala University, Sweden Eyvaz.Isaev at fysik.uu.se, eyvaz_isaev at yahoo.com ____________________________________________________________________________________ Looking for last minute shopping deals? Find them fast with Yahoo! Search. http://tools.search.yahoo.com/newsearch/category.php?category=shopping From giannozz at nest.sns.it Fri Jan 11 12:18:53 2008 From: giannozz at nest.sns.it (Paolo Giannozzi) Date: Fri, 11 Jan 2008 12:18:53 +0100 Subject: [Pw_forum] CuO, LDA+U, Pseudopots, again In-Reply-To: <000e01c85348$c714d0c0$553e7240$@net> References: <000e01c85348$c714d0c0$553e7240$@net> Message-ID: <4356D195-A634-4160-A7A2-E963890415F4@nest.sns.it> On Jan 10, 2008, at 6:22 , Paul M. Grant wrote: > I strongly suspect my problem involves use (or misuse or > unavailability) of a proper Cu pseudopotential. > I vaguely remember having read that getting the correct d-s transition energies in atoms is quite important for a good pseudopotential for transition metals. The ultimate pseudopotential test is an all-electron calculation, for instance with LMTO (I think that the first implementation of lda+U was developed for LMTO). Pseudopotentials are "the usual suspects" when something doesn't work as expected, but the functional is also a likely suspect in "exotic" cases. > Sorry for all my American slang. We didn't invent the > language...the Brits did > "two nations divided by a common language" Paolo --- Paolo Giannozzi, Dept of Physics, University of Udine via delle Scienze 208, 33100 Udine, Italy Phone +39-0432-558216, fax +39-0432-558222 From chen_shao_hua197 at yahoo.com.tw Fri Jan 11 19:32:19 2008 From: chen_shao_hua197 at yahoo.com.tw (=?big5?q?=B3=AF=20=A4=D6=B5=D8?=) Date: Sat, 12 Jan 2008 02:32:19 +0800 (CST) Subject: [Pw_forum] q2r.f90 error message "wrong nb read" In-Reply-To: <972384.58081.qm@web60322.mail.yahoo.com> Message-ID: <857042.72537.qm@web73004.mail.tp2.yahoo.com> Dear Prof. Eyvaz Isaev, Thanks for your advice. I will recalculate it carefully. max --- Eyvaz Isaev ?? > Hi, > > It seems someting wrong in your dynamical matrix. > Check carefully your input file (atomic positions > and > types) and dymanical matrices. The error happens > when > there is a mismatch between atomic types. > > Bests, > Eyvaz. > > > --- ? ?? wrote: > > > Dear Forum's members, > > > > There is a error message "wrong nb read" in the > > In.q2r.out . > > I check its meaning in q2r.f90 . > > But I am not good at fortran's program. > > Does anyone could tell me how to avoid this error > ? > > > > Thanks in advance > > > > max > > Physics department, National Taiwan > > University,Taiwan > > > > P.S. In.q2r.in > > &input > > zasr='simple', fildyn='In.dyn', flfrc='In.fc', > > la2F=.true. > > / > > > > > > > > > ____________________________________________________________________________________ > > > ????????????????http://tw.mini.yahoo.com/ > > > > _______________________________________________ > > Pw_forum mailing list > > Pw_forum at pwscf.org > > http://www.democritos.it/mailman/listinfo/pw_forum > > > > > ------------------------------------------------------------------- > Prof. Eyvaz Isaev, > Theoretical Physics Department, Moscow State > Institute of Steel & Alloys, Russia, and > Condensed Matter Theory Group, Uppsala University, > Sweden > Eyvaz.Isaev at fysik.uu.se, eyvaz_isaev at yahoo.com > > > > ____________________________________________________________________________________ > Looking for last minute shopping deals? > Find them fast with Yahoo! Search. > http://tools.search.yahoo.com/newsearch/category.php?category=shopping > _______________________________________________ > Pw_forum mailing list > Pw_forum at pwscf.org > http://www.democritos.it/mailman/listinfo/pw_forum > ____________________________________________________________________________________ ????????????????http://tw.mini.yahoo.com/ From gaoguoying at gmail.com Sat Jan 12 01:52:34 2008 From: gaoguoying at gmail.com (Guoying Gao) Date: Sat, 12 Jan 2008 08:52:34 +0800 Subject: [Pw_forum] question on the tetrahedron method for the phonon DOS Message-ID: <6234c0c60801111652h522d6d41p222386ac82b88905@mail.gmail.com> Dear pwscf users, I intend to study the tetrahedron method used in the phonon DOS. I have read the source program in flib/dost.f90. However, I still cann't understand this method. Could anyone give me some hints or some papers about this aspect? Thanks in advance. Guoying National lab of superhard materials Jilin University From eyvaz_isaev at yahoo.com Sat Jan 12 12:17:31 2008 From: eyvaz_isaev at yahoo.com (Eyvaz Isaev) Date: Sat, 12 Jan 2008 03:17:31 -0800 (PST) Subject: [Pw_forum] question on the tetrahedron method for the phonon DOS In-Reply-To: <6234c0c60801111652h522d6d41p222386ac82b88905@mail.gmail.com> Message-ID: <70293.78833.qm@web60316.mail.yahoo.com> Hi, Please read original papers: 1. G. Lehman and M. Taut, Phys. Status Solidi B54, 469 1972 2. PE Bloechl, O. Jepsen and O. K. Andersen , PRB49, 16223 (1994) Bests, Eyvaz. --- Guoying Gao wrote: > Dear pwscf users, > > I intend to study the tetrahedron method used > in the phonon > DOS. I have read the source program in > flib/dost.f90. However, I still > cann't understand this method. > Could anyone give me some hints or some papers about > this aspect? > Thanks in advance. > Guoying > National lab of superhard materials > Jilin University > _______________________________________________ > Pw_forum mailing list > Pw_forum at pwscf.org > http://www.democritos.it/mailman/listinfo/pw_forum > ------------------------------------------------------------------- Prof. Eyvaz Isaev, Theoretical Physics Department, Moscow State Institute of Steel & Alloys, Russia, and Condensed Matter Theory Group, Uppsala University, Sweden Eyvaz.Isaev at fysik.uu.se, eyvaz_isaev at yahoo.com ____________________________________________________________________________________ Be a better friend, newshound, and know-it-all with Yahoo! Mobile. Try it now. http://mobile.yahoo.com/;_ylt=Ahu06i62sR8HDtDypao8Wcj9tAcJ From faridul at email.com Sat Jan 12 18:25:02 2008 From: faridul at email.com (Faridul Isalm) Date: Sat, 12 Jan 2008 12:25:02 -0500 Subject: [Pw_forum] Help about bands.x Message-ID: <20080112172503.08C511BF28D@ws1-1.us4.outblaze.com> Dear Sir, I do hope you are fine and well. When I run bands.x bands.out, then I have following problems: 1) at the bands.out file I got the following lines only: Program POST-PROC v.3.2.3 starts ... Today is 12Jan2008 at 23: 4:34 nbndx = 34 nbnd = 34 natomwfc = 91 npwx = 3109 nelec = 56.00 nkb = 56 ngl = 2793 ************************************************************************** xk=( 0.50000, 0.50000, 0.50000 ) point group C_1 (1) there are 1 classes the character table: E A 1.00 ------------------ 2} there are other 3 files are created (a) AlAs.band.dat.rap (it is empty), (b) bands.x.stackdump, that contains: Exception: STATUS_ACCESS_VIOLATION at eip=6A4983A4 eax=00000040 ebx=7F1DA9C0 ecx=00000010 edx=00000BE7 esi=A880FDD0 edi=00000001 ebp=002282C0 esp=002282A4 program=C:\cygwin\home\dk\espresso\PP\bands.x, pid 1240, thread main cs=001B ds=0023 es=0023 fs=003B gs=0000 ss=0023 Stack trace: Frame Function Args 002282C0 6A4983A4 (002283A8, 01D75110, A880FDD0, 002283FC) 00228428 00428690 (7F850020, 01FFE3B0, 00776280, 01D75108) 0022C948 00427ABA (0022CB74, 0022CCAC, 0022CCB4, 0022CCB8) 0022CCC8 00401316 (00000001, 6116A950, 005681F0, 00568203) 0022CCE8 00568226 (00000001, 6116A950, 01FF0090, 00000000) 0022CD98 61006198 (00000000, 0022CDD0, 61005510, 0022CDD0) 61005510 61004416 (0000009C, A02404C7, E8611021, FFFFFF48) 10 [main] bands.x 1240 _cygtls::handle_exceptions: Error while dumping state (probably corrupted stack) and (c) and a dat file. why a and b files are created, How can i slove these problems. Please help me to solve these problems. I am looking forward to hearing from you. Thank you for your time. Sincerely A.K.Md Farid ul Islam -- Are we headed for a recession? Read more on the Money Portal Mail.com Money - http://www.mail.com/Money.aspx?cat=money From jackygrahamez at gmail.com Sun Jan 13 00:33:50 2008 From: jackygrahamez at gmail.com (Jack Shultz) Date: Sat, 12 Jan 2008 18:33:50 -0500 Subject: [Pw_forum] Prep CI-NEB analysis for Espresso Message-ID: <6c65435e0801121533k2ae7a60ar5ded01f467253004@mail.gmail.com> I am attempting to simulate a H2O2 redox to O2 and H2O such as this 2 H2O2 --> O2 + H2O I'm not sure exactly what I'm doing here. I'm trying to apply example 17 (proton transfer) to another reaction so I can understand how this works. If you got a better reaction for me to learn on, I'm all ears. I tried a couple pseudo potentials for Oxygen. Here are parameters for Atomic species which I used ATOMIC_SPECIES H 1.00794 HUSPBE.RRKJ3 O 15.9994 O.LDA.US.RRKJ3.UPF Then I get this error when I run it. Warning: card O 15.9994 O.LDA.US.RRKJ3.UPF ignored %%%%%%%%%%%%%%%%%%%%%%%%%%%%%%%%%%%%%%%%%%%%%%%%%%%%%%%%%%%%%%%%%%%%%%%%%%%%%% from read_cards : error # 1 wrong index in ATOMIC_POSITIONS %%%%%%%%%%%%%%%%%%%%%%%%%%%%%%%%%%%%%%%%%%%%%%%%%%%%%%%%%%%%%%%%%%%%%%%%%%%%%% stopping ... -------------- next part -------------- An HTML attachment was scrubbed... URL: http://www.democritos.it/pipermail/pw_forum/attachments/20080112/683ba314/attachment.htm -------------- next part -------------- An embedded and charset-unspecified text was scrubbed... Name: test Url: http://www.democritos.it/pipermail/pw_forum/attachments/20080112/683ba314/attachment.txt -------------- next part -------------- A non-text attachment was scrubbed... Name: H2O2.in Type: application/octet-stream Size: 1896 bytes Desc: not available Url : http://www.democritos.it/pipermail/pw_forum/attachments/20080112/683ba314/attachment.obj -------------- next part -------------- A non-text attachment was scrubbed... Name: H2O2.out Type: application/octet-stream Size: 578 bytes Desc: not available Url : http://www.democritos.it/pipermail/pw_forum/attachments/20080112/683ba314/attachment-0001.obj From akohlmey at cmm.chem.upenn.edu Sun Jan 13 14:25:55 2008 From: akohlmey at cmm.chem.upenn.edu (Axel Kohlmeyer) Date: Sun, 13 Jan 2008 13:25:55 +0000 Subject: [Pw_forum] Prep CI-NEB analysis for Espresso In-Reply-To: <6c65435e0801121533k2ae7a60ar5ded01f467253004@mail.gmail.com> References: <6c65435e0801121533k2ae7a60ar5ded01f467253004@mail.gmail.com> Message-ID: <7b6913e90801130525u761be62er39030b7564011d63@mail.gmail.com> On 1/12/08, Jack Shultz wrote: > I am attempting to simulate a H2O2 redox to O2 and H2O such as this > 2 H2O2 --> O2 + H2O > > I'm not sure exactly what I'm doing here. I'm trying to apply example 17 > (proton transfer) to another reaction so I can understand how this works. If > you got a better reaction for me to learn on, I'm all ears. jack, you are trying to 'run before you can walk', which means you are bound to fail and not understand what went wrong. please have a look at the tutorials for simple pw.x calculations (single point energy, geometry optimizations) and then at the file Doc/INPUT_PW which explains all input parameters and the structure of the input file. particularly the entries for 'nat' and 'ntyp' warrant closer inspection. but also 'nspin', 'ecutwfc' and 'ecutrho'. the latter two are connected to your choice of pseudopotentials, and (again) those have to be chosen properly (e.g. you cannot mix and match pseudopotentials generated for different DFT functionals. finally, you'll have to look into how the size of the unit cell affects the accuracy of your results. plane wave pseudopotential calculations are _always_ periodic, so you have to minimize the impact of the periodic images on the structures. if you don't those right and a working geometry optimization, there is no point in starting a NEB calculation. then please also have a look at what are the prerequisites or starting points for NEB calculations. on the quantum espresso homepage are links to collections of tutorial material (click on 'learn') that has been tried, tested and refined over several years and provide a good starting point into this field. as with any first principles application, you have to have a reasonably good understanding of what approximations you are making so that you are getting meaningful results and are not generating expensive random numbers. even very experienced users (hmmm... or perhaps particularly experience users) tend to have always doubts whether everything really went ok, or whether they have overlooked something, that does not show immediately. if you have seen 'home improvement' you'll know, that powerful tools can also do the most damage. ;-) cheers, axel. > > I tried a couple pseudo potentials for Oxygen. Here are parameters for > Atomic species which I used > ATOMIC_SPECIES > > > H 1.00794 HUSPBE.RRKJ3 > > O 15.9994 O.LDA.US.RRKJ3.UPF > Then I get this error when I run it. > Warning: card O 15.9994 O.LDA.US.RRKJ3.UPF ignored > %%%%%%%%%%%%%%%%%%%%%%%%%%%%%%%%%%%%%%%%%%%%%%%%%%%%%%%%%%%%%%%%%%%%%%%%%%%%%% > from read_cards : error # 1 > wrong index in ATOMIC_POSITIONS > %%%%%%%%%%%%%%%%%%%%%%%%%%%%%%%%%%%%%%%%%%%%%%%%%%%%%%%%%%%%%%%%%%%%%%%%%%%%%% > stopping ... > _______________________________________________ > Pw_forum mailing list > Pw_forum at pwscf.org > http://www.democritos.it/mailman/listinfo/pw_forum > > > -- ======================================================================= Axel Kohlmeyer akohlmey at cmm.chem.upenn.edu http://www.cmm.upenn.edu Center for Molecular Modeling -- University of Pennsylvania Department of Chemistry, 231 S.34th Street, Philadelphia, PA 19104-6323 tel: 1-215-898-1582, fax: 1-215-573-6233, office-tel: 1-215-898-5425 ======================================================================= If you make something idiot-proof, the universe creates a better idiot. From giannozz at nest.sns.it Sun Jan 13 15:35:05 2008 From: giannozz at nest.sns.it (Paolo Giannozzi) Date: Sun, 13 Jan 2008 15:35:05 +0100 Subject: [Pw_forum] Help about bands.x In-Reply-To: <20080112172503.08C511BF28D@ws1-1.us4.outblaze.com> References: <20080112172503.08C511BF28D@ws1-1.us4.outblaze.com> Message-ID: <200801131535.05565.giannozz@nest.sns.it> On Saturday 12 January 2008 18:25, Faridul Isalm wrote: > When I run bands.x bands.out, then I have following problems please provide a test job so that the error can be reproduced (or can be declared to be irreproducible -:) Paolo -- Paolo Giannozzi, Democritos and Udine University From szs_naghavi at yahoo.com Sun Jan 13 16:16:51 2008 From: szs_naghavi at yahoo.com (zahra sadat naghavi) Date: Sun, 13 Jan 2008 07:16:51 -0800 (PST) Subject: [Pw_forum] raman cross section Message-ID: <560900.63968.qm@web63011.mail.re1.yahoo.com> dear everybody! i wanted to calculate raman tensor in ph.x for a metal(at gamma point) and then run dynmat.x to find raman cross section but ph.x can not be run if ' lraman' is added in input file . what is wrong ? is there any other way? --------------------------------- Never miss a thing. Make Yahoo your homepage. -------------- next part -------------- An HTML attachment was scrubbed... URL: http://www.democritos.it/pipermail/pw_forum/attachments/20080113/8e277647/attachment.htm From marcel at physik.tu-berlin.de Sun Jan 13 17:49:58 2008 From: marcel at physik.tu-berlin.de (Marcel Mohr) Date: Sun, 13 Jan 2008 17:49:58 +0100 (CET) Subject: [Pw_forum] raman cross section In-Reply-To: <560900.63968.qm@web63011.mail.re1.yahoo.com> References: <560900.63968.qm@web63011.mail.re1.yahoo.com> Message-ID: <20080113174841.K30704@rosa.physik-pool.tu-berlin.de> Hello the problem is that the calculation does not work for metals. It was discussed a few month ago here. Best Marcel On Sun, 13 Jan 2008, zahra sadat naghavi wrote: > dear everybody! > i wanted to calculate raman tensor in ph.x for a metal(at gamma point) and then run dynmat.x to find raman cross section but ph.x can not be run if ' lraman' is added in input file . > what is wrong ? > is there any other way? > > > --------------------------------- > Never miss a thing. Make Yahoo your homepage. From mmarques at staffmail.ed.ac.uk Mon Jan 14 11:50:53 2008 From: mmarques at staffmail.ed.ac.uk (Miriam Marques) Date: Mon, 14 Jan 2008 10:50:53 +0000 Subject: [Pw_forum] Raman intensities for "metallic systems" Message-ID: <20080114105053.fwqv24sym8w484oo@www.staffmail.ed.ac.uk> Dear ESPRESSO users, I am emailing a previous unanswered question. Is possible to calculate Raman intensities for "metallic systems" such as hcp-metals: Zn, Fe, Zr, Mg, high pressure phases of oxygen or high pressure phases of alkali metals using the ESPRESSO code? Thank you very much, Miriam. Dr. Miriam Marques Centre for Science at Extreme Conditions. The University of Edinburgh. e-mail: mmarques at staffmail.ed.ac.uk -- The University of Edinburgh is a charitable body, registered in Scotland, with registration number SC005336. From uccaddi at ucl.ac.uk Mon Jan 14 17:18:40 2008 From: uccaddi at ucl.ac.uk (Devis Di Tommaso) Date: Mon, 14 Jan 2008 16:18:40 -0000 (GMT) Subject: [Pw_forum] constrained molecular dynamics Message-ID: <1710.128.40.77.197.1200327520.squirrel@www.squirrelmail.ucl.ac.uk> Dear all, is it possible to perform constrained molecular dynamics simulation with the present version of CP? Thanks, Devis -- Devis Di Tommaso Department of Chemistry Christopher Ingold Laboratories University College of London 20 Gordon Street London WC1H 0AJ United Kingdom Phone: +44 (0)20 76 79 74 65 Internal: Phone 2 74 65 From w2agz at pacbell.net Tue Jan 15 03:27:50 2008 From: w2agz at pacbell.net (Paul M. Grant) Date: Mon, 14 Jan 2008 18:27:50 -0800 Subject: [Pw_forum] *UPF - Easy Question Message-ID: <00cf01c8571e$3a1eedc0$ae5cc940$@net> Occasionally I see "asterisks" on the principal quantum number label in UPF PP representations such as the following for OPBE_nc.UPC: nl pn l occ *S 0 0 2.00 *P 0 1 3.00 And others, some "translated" from FHI and Opium. I searched the excellent guide in O-Sesame on PP construction authored by the Wizard of Udine for insight and came up empty. Paul M. Grant, PhD Principal, W2AGZ Technologies Visiting Scholar, Applied Physics, Stanford University EPRI Science Fellow (Retired) IBM Research Staff Member Emeritus w2agz at pacbell.net http://www.w2agz.com -------------- next part -------------- An HTML attachment was scrubbed... URL: http://www.democritos.it/pipermail/pw_forum/attachments/20080114/3735d198/attachment.htm From giannozz at nest.sns.it Tue Jan 15 08:59:06 2008 From: giannozz at nest.sns.it (Paolo Giannozzi) Date: Tue, 15 Jan 2008 08:59:06 +0100 Subject: [Pw_forum] *UPF - Easy Question In-Reply-To: <00cf01c8571e$3a1eedc0$ae5cc940$@net> References: <00cf01c8571e$3a1eedc0$ae5cc940$@net> Message-ID: On Jan 15, 2008, at 3:27 , Paul M. Grant wrote: > Occasionally I see ?asterisks? on the principal quantum number > label in UPF PP representations such as the following for OPBE_nc.UPC: > > nl pn l occ > > *S 0 0 2.00 > > *P 0 1 3.00 > > And others, some ?translated? from FHI and Opium. > this typically happens when other formats are converted into UPF. Very often the information on the principal quantum number is missing in the original format, so the (software) converter puts a *. It is the task of the (human) converter to edit the file and put the correct value of N (which, by the way, is not used in any actual calculation, but may be shown in the output of some types of calculations). Paolo --- Paolo Giannozzi, Dept of Physics, University of Udine via delle Scienze 208, 33100 Udine, Italy Phone +39-0432-558216, fax +39-0432-558222 From min0220 at postech.ac.kr Tue Jan 15 09:00:53 2008 From: min0220 at postech.ac.kr (Min Seung Kyu) Date: Tue, 15 Jan 2008 17:00:53 +0900 (KST) Subject: [Pw_forum] Eigenfunctions in PWSCF Message-ID: Dear users and developers, I have a question about eigenfunctions in PWSCF codes. I looked at PWSCF source files and I found that the 'evc' variable is KS eigenvectors in G-space, right? Because Hamiltonian matrix H(G,G') is hermitian, 'evc' should be real. When I print out 'evc', however, 'evc' contains imaginary parts. For example, when I run 'co.rx.in' (example03 folder), I got these values: DBLE(evc(1:4,1)) 0.186884 0.154551 0.154551 0.142085 AIMAG(evc(1:4,1)) 0.000000 0.000000 0.000000 -0.048560 DBLE(evc(1:4,2)) -0.024799 -0.021751 -0.021751 -0.068645 AIMAG(evc(1:4,2)) 0.000000 0.000000 0.000000 -0.145963 DBLE(evc(1:4,3)) 0.000000 0.000000 0.000000 0.000000 AIMAG(evc(1:4,3)) 0.000000 -0.005668 0.152298 0.000000 DBLE(evc(1:4,4)) 0.000000 0.000000 0.000000 0.000000 AIMAG(evc(1:4,4)) 0.000000 -0.152298 -0.005668 0.000000 DBLE(evc(1:4,5)) -0.299154 -0.185970 -0.185970 0.011252 AIMAG(evc(1:4,5)) 0.000000 0.000000 0.000000 0.168998 ( I added 'WRITE(*,*) evc(1:4,ibnd)' in 'sum_band.f90'. ) What is the problem? Thank you in advance. Sincerely, Min. -------------------------------Sig.--------------------------- Center for Superfunctional Materials, Department of Chemistry, Pohang University of Science and Technology (POSTECH) Mr. Seung Kyu, Min Ph.D. Candidate E-Mail: min0220 at postech.ac.kr Tel. : 82-54-279-5858 ---------------------------------------------- From min0220 at postech.ac.kr Tue Jan 15 09:26:15 2008 From: min0220 at postech.ac.kr (Min Seung Kyu) Date: Tue, 15 Jan 2008 17:26:15 +0900 (KST) Subject: [Pw_forum] Making density matrix in sum_band subroutine Message-ID: Dear users and developers, I'm studying PWSCF source code. Looking in 'sum_band_gamma' subroutine in 'sum_band.f90', this subroutine does two FFTs at the same time. DO ibnd = 1, nbnd, 2 psic(:) = ( 0.D0, 0.D0 ) IF ( ibnd < nbnd ) THEN ! ! ... two ffts at the same time ! psic(nls(igk(1:npw))) = evc(1:npw,ibnd) + & ( 0.D0, 1.D0 ) * evc(1:npw,ibnd+1) psic(nlsm(igk(1:npw))) = CONJG( evc(1:npw,ibnd) - & ( 0.D0, 1.D0 ) * evc(1:npw,ibnd+1) ) ELSE psic(nls(igk(1:npw))) = evc(1:npw,ibnd) psic(nlsm(igk(1:npw))) = CONJG( evc(1:npw,ibnd) ) END IF CALL cft3s( psic, nr1s, nr2s, nr3s, nrx1s, nrx2s, nrx3s, 2 ) w1 = wg(ibnd,ik) / omega ! ! ... increment the charge density ... ! IF ( ibnd < nbnd ) THEN ! ! ... two ffts at the same time ! w2 = wg(ibnd+1,ik) / omega ELSE w2 = w1 END IF DO ir = 1, nrxxs rho(ir,current_spin) = rho(ir,current_spin) + & w1 * DBLE( psic(ir) )**2 + & w2 * AIMAG( psic(ir) )**2 END DO ENDDO I changed above lines into below. DO ibnd = 1, nbnd psic(:) = ( 0.D0, 0.D0 ) psic(nls(igk(1:npw))) = evc(1:npw,ibnd) CALL cft3s( psic, nr1s, nr2s, nr3s, nrx1s, nrx2s, nrx3s, 2 ) w1 = wg(ibnd,ik) / omega ! ! ... increment the charge density ... ! DO ir = 1, nrxxs rho(ir,current_spin) = rho(ir,current_spin) + & w1 * ( DBLE( psic(ir) )**2 + & AIMAG( psic(ir) )**2 ) END DO END DO But I failed to reproduce charge densities. For 'co.rx.in' in example03 folder, I got this message. WARNING: integrated charge= 6.86602003, expected= 10.00000000 Could you tell me what is wrong? Thank you in advance. Sincerely, Min. -------------------------------Sig.--------------------------- Center for Superfunctional Materials, Department of Chemistry, Pohang University of Science and Technology (POSTECH) Mr. Seung Kyu, Min Ph.D. Candidate E-Mail: min0220 at postech.ac.kr Tel. : 82-54-279-5858 ---------------------------------------------- From marzari at MIT.EDU Tue Jan 15 10:03:58 2008 From: marzari at MIT.EDU (Nicola Marzari) Date: Tue, 15 Jan 2008 04:03:58 -0500 Subject: [Pw_forum] Eigenfunctions in PWSCF In-Reply-To: References: Message-ID: <478C76FE.70606@mit.edu> Dear Min Seung, it's the eigenvalues that are real, not the eigenvectors. Consider the kinetic energy operator - its eigenvectors are plane waves, that are not real. Now, in periodic boundary conditions, we usually choose eigenstates of the periodic Hamiltonian that are also eigenstates of the translation operator - i.e. they have the Bloch form u_nk(r)exp(ikr), with u_nk(r) periodic. u_nk(r) satisfies the Schroedinger-like equation with the k-dependent Hamiltonian [-h^2/2m (del+ik)^2+V(r)] u_nk = e_nk u_nk. From this, you see that if u_nk is a solution at a k-point k, its complex conjugate is a solution at the k-point -k, i.e. u_nk(r)=(u_n-k(r))* . Actually, you have the gauge freedom of multiplying the eigenvector by a phase factor, so the equality is true only a part from a phase factor (if you were to use a conjugate gradient code, you'd infact find a random phase factor, at selfconsistency, between u_nk and (u_n-k)* ). These u_nk are what is stored in evc (I hope - I'm not that familiar with the innards of PWscf), in their *plane wave expansion*. Only at Gamma you have that the eigenstates can be chosen as real (since u_nk(r)=(u_n-k(r))*, and Gamma=-Gamma) - still, this would mean that u_n0(r) is real, but not the coefficients of its plane wave expansion - these would satisfy c_G^nk=c(_-G^nk)*. This symmetry can be exploited, and is, in CP, and in the Gamma version of CP. The distribution should have some notes on FFTs that would explain your second question on the double FFT - for Gamma only calculations, the code lumps together two bands in a single complex-to-complex FFT. Note that if u_n0(r) has inversion symmetry, then the coefficients of the plane wave expansion are also real - albeit this has not been exploited, afaik, since early calculations (1991-1992) with the Cambridge CASTEP codes. Best, nicola Min Seung Kyu wrote: > Dear users and developers, > > I have a question about eigenfunctions in PWSCF codes. > > I looked at PWSCF source files and I found that the 'evc' variable is > KS eigenvectors in G-space, right? > > Because Hamiltonian matrix H(G,G') is hermitian, 'evc' should be real. > > When I print out 'evc', however, 'evc' contains imaginary parts. > > For example, when I run 'co.rx.in' (example03 folder), I got these values: > > DBLE(evc(1:4,1)) 0.186884 0.154551 0.154551 0.142085 > AIMAG(evc(1:4,1)) 0.000000 0.000000 0.000000 -0.048560 > DBLE(evc(1:4,2)) -0.024799 -0.021751 -0.021751 -0.068645 > AIMAG(evc(1:4,2)) 0.000000 0.000000 0.000000 -0.145963 > DBLE(evc(1:4,3)) 0.000000 0.000000 0.000000 0.000000 > AIMAG(evc(1:4,3)) 0.000000 -0.005668 0.152298 0.000000 > DBLE(evc(1:4,4)) 0.000000 0.000000 0.000000 0.000000 > AIMAG(evc(1:4,4)) 0.000000 -0.152298 -0.005668 0.000000 > DBLE(evc(1:4,5)) -0.299154 -0.185970 -0.185970 0.011252 > AIMAG(evc(1:4,5)) 0.000000 0.000000 0.000000 0.168998 > > ( I added 'WRITE(*,*) evc(1:4,ibnd)' in 'sum_band.f90'. ) > > What is the problem? > > Thank you in advance. > > Sincerely, Min. -- --------------------------------------------------------------------- Prof Nicola Marzari Department of Materials Science and Engineering 13-5066 MIT 77 Massachusetts Avenue Cambridge MA 02139-4307 USA tel 617.4522758 fax 2586534 marzari at mit.edu http://quasiamore.mit.edu From giannozz at nest.sns.it Tue Jan 15 10:22:53 2008 From: giannozz at nest.sns.it (Paolo Giannozzi) Date: Tue, 15 Jan 2008 10:22:53 +0100 Subject: [Pw_forum] Making density matrix in sum_band subroutine In-Reply-To: References: Message-ID: <02B8B753-8150-4F5E-BEEB-FE0EB3C39CE2@nest.sns.it> On Jan 15, 2008, at 9:26 , Min Seung Kyu wrote: > Looking in 'sum_band_gamma' subroutine in 'sum_band.f90', this > subroutine > does two FFTs at the same time [...] > I changed above lines into below [...] > But I failed to reproduce charge densities. in the "gamma" case, i.e. k=0, the eigenvectors are real in real space. Thus only half of the plane waves are stored, because \psi(-G)=\psi*(G). In the FFT array, you need all of them. So you need psic(nls(igk(1:npw))) = evc(1:npw,ibnd) AND psic(nlsm(igk(1:npw))) = CONJG(evc(1:npw,ibnd)) (the G=0 is copied twice but since \psi(0) is real it doesn't matter) P. --- Paolo Giannozzi, Dept of Physics, University of Udine via delle Scienze 208, 33100 Udine, Italy Phone +39-0432-558216, fax +39-0432-558222 From giannozz at nest.sns.it Tue Jan 15 10:27:46 2008 From: giannozz at nest.sns.it (Paolo Giannozzi) Date: Tue, 15 Jan 2008 10:27:46 +0100 Subject: [Pw_forum] Eigenfunctions in PWSCF In-Reply-To: <478C76FE.70606@mit.edu> References: <478C76FE.70606@mit.edu> Message-ID: <7E11D25D-CAF0-4910-AA54-A9BF028C24D7@nest.sns.it> On Jan 15, 2008, at 10:03 , Nicola Marzari wrote: > These u_nk are what is stored in evc (I hope - I'm not that familiar > with the innards of PWscf), in their *plane wave expansion*. confirmed > Note that if u_n0(r) has inversion symmetry, then the coefficients > of the plane wave expansion are also real - albeit this has not been > exploited, afaik, since early calculations (1991-1992) with the > Cambridge CASTEP codes. once upon a time there was a version of the PWscf code that exploited inversion symmetry and used real wavefunctions and hamiltonian matrix (at the time, the hamiltonian was still stored in a matrix). I think it was also ported to separable pseudopotentials (i.e. hamiltonian stored as an operator, not as a matrix). It disappeared a few years ago for no other reason than the difficulty to keep 2N+1 versions of the code alive at the same time. It should be relatively straightforward to implement it again, though, adapting the existing Gamma-only technology. It could be a good and useful exercise for anybody who would like to learn how plane-wave codes work Paolo --- Paolo Giannozzi, Dept of Physics, University of Udine via delle Scienze 208, 33100 Udine, Italy Phone +39-0432-558216, fax +39-0432-558222 From helen at fh.huji.ac.il Tue Jan 15 11:27:37 2008 From: helen at fh.huji.ac.il (Helen) Date: Tue, 15 Jan 2008 12:27:37 +0200 Subject: [Pw_forum] wurtzite AlN Message-ID: <001e01c85761$3fbc04f0$cd604084@fh.huji.ac.il> Hi, I've successfully calculated the bandstructure for zinc-blende AlN, and I wanted to do the same for wurtzite AlN but am running into problems. I started off with a=5.88 a.u. c/a=1.600 and u=0.382 , I then kept a and c constant and altered u. The results showed that u needed to be smaller but when I reached u=0.369 my scf calculations stopped converging. I tried using both cg and davidson diagonalization. I then tried keeping u constant and altering c, I don't get convergence problems but I get unrealistically small values of c. Also the bandgap data I'm getting is nonsense (no band gap at all). Perhaps I have a mistake in my input file? Thank you for your help and advice, Dr. Helen Eisenberg (Fritz Haber Center, Hebrew University) &control calculation='scf' restart_mode='from_scratch', tstress = .true. tprnfor = .true. prefix='alas', pseudo_dir = '$PSEUDO_DIR/', outdir='$TMP_DIR/' / &system ibrav= 4, celldm(1) =5.88, celldm(3)=1.600, nat= 2, ntyp= 2, ecutwfc =80.0, nbnd = 8, nr1=32, nr2=32, nr3=32, ecutrho = 240.0 / &electrons conv_thr = 1.0d-6 mixing_beta = 0.7 / ATOMIC_SPECIES Al 26.98 Al.vbc.UPF N 14.007 N.pz-vbc.UPF ATOMIC_POSITIONS Al 0.00 0.00 0.00 N 0.00 0.00 0.611 K_POINTS (automatic) 4 4 4 0 0 0 EOF -------------- next part -------------- An HTML attachment was scrubbed... URL: http://www.democritos.it/pipermail/pw_forum/attachments/20080115/740f8a75/attachment.htm From marcel at physik.tu-berlin.de Tue Jan 15 12:07:27 2008 From: marcel at physik.tu-berlin.de (Marcel Mohr) Date: Tue, 15 Jan 2008 12:07:27 +0100 (CET) Subject: [Pw_forum] wurtzite AlN In-Reply-To: <001e01c85761$3fbc04f0$cd604084@fh.huji.ac.il> References: <001e01c85761$3fbc04f0$cd604084@fh.huji.ac.il> Message-ID: <20080115120654.B80605@rosa.physik-pool.tu-berlin.de> Hi, wurtzite has 4 atoms in the unit cell. Cheers Marcel On Tue, 15 Jan 2008, Helen wrote: > Hi, > I've successfully calculated the bandstructure for zinc-blende AlN, and I wanted to do the same for wurtzite AlN but am running into problems. I started off with a=5.88 a.u. c/a=1.600 and u=0.382 , I then kept a and c constant and altered u. The results showed that u needed to be smaller but when I reached u=0.369 my scf calculations stopped converging. I tried using both cg and davidson diagonalization. I then tried keeping u constant and altering c, I don't get convergence problems but I get unrealistically small values of c. Also the bandgap data I'm getting is nonsense (no band gap at all). Perhaps I have a mistake in my input file? > Thank you for your help and advice, > Dr. Helen Eisenberg (Fritz Haber Center, Hebrew University) > > &control > calculation='scf' > restart_mode='from_scratch', > tstress = .true. > tprnfor = .true. > prefix='alas', > pseudo_dir = '$PSEUDO_DIR/', > outdir='$TMP_DIR/' > / > &system > ibrav= 4, celldm(1) =5.88, celldm(3)=1.600, nat= 2, ntyp= 2, > ecutwfc =80.0, nbnd = 8, > nr1=32, > nr2=32, > nr3=32, > ecutrho = 240.0 > / > &electrons > conv_thr = 1.0d-6 > mixing_beta = 0.7 > / > ATOMIC_SPECIES > Al 26.98 Al.vbc.UPF > N 14.007 N.pz-vbc.UPF > ATOMIC_POSITIONS > Al 0.00 0.00 0.00 > N 0.00 0.00 0.611 > K_POINTS (automatic) > 4 4 4 0 0 0 > EOF From yupiaofei at mail.sdu.edu.cn Tue Jan 15 12:08:51 2008 From: yupiaofei at mail.sdu.edu.cn (KS Yang) Date: Tue, 15 Jan 2008 19:08:51 +0800 Subject: [Pw_forum] Mulliken population Message-ID: <400395646.05061@mail.sdu.edu.cn> Hi, everyone It is very easy to obtain the Mulliken population of the atom in the output file using CASTEP or VASP code. I am sorry but i cann't find it in the output file of PWSCF code. Somebody can tell me where can i find it? Does the "effective charges" mentioned in the users-guide mean the Mulliken population? Best wishes. Kesong Yang School of Physics and Microelectronics Shandong University, Jinan 250100, People?s Republic of China Email: ksyang at mail.sdu.edu.cn; yupiaofei at mail.sdu.edu.cn -------------- next part -------------- An HTML attachment was scrubbed... URL: http://www.democritos.it/pipermail/pw_forum/attachments/20080115/d81b3a34/attachment.htm From akohlmey at cmm.chem.upenn.edu Tue Jan 15 12:22:45 2008 From: akohlmey at cmm.chem.upenn.edu (Axel Kohlmeyer) Date: Tue, 15 Jan 2008 11:22:45 +0000 Subject: [Pw_forum] Mulliken population In-Reply-To: <400395646.05061@mail.sdu.edu.cn> References: <400395646.05061@mail.sdu.edu.cn> Message-ID: <7b6913e90801150322x6c6f814eudb98f9bb5277b12@mail.gmail.com> On 1/15/08, KS Yang wrote: > > > Hi, everyone > > It is very easy to obtain the Mulliken population of the atom in the output > file using CASTEP or VASP code. sorry. this is a forum for quantum espresso not CASTEP or VASP. please ask in the appropriate places. > I am sorry but i cann't find it in the output file of PWSCF code. Somebody PWSCF is not gaussian! > can tell me where can i find it? mulliken analysis requires the an "atomic" basis set, the codes in the quantum espresso package use plane wave basis sets, so you first have to project the wavefunction from one basis on another. of course this projection is always incomplete, so you'll have to treat the results with even more care than from codes that use local basis set. for details on how to project, have a look at Doc/INPUT_PROJWFC. > Does the "effective charges" mentioned in the users-guide mean the Mulliken > population? no. cheers, axel. > > > Best wishes. > > > Kesong Yang > > School of Physics and Microelectronics > > Shandong University, Jinan 250100, People's Republic of China > > Email: ksyang at mail.sdu.edu.cn; > yupiaofei at mail.sdu.edu.cn > > _______________________________________________ > Pw_forum mailing list > Pw_forum at pwscf.org > http://www.democritos.it/mailman/listinfo/pw_forum > > -- ======================================================================= Axel Kohlmeyer akohlmey at cmm.chem.upenn.edu http://www.cmm.upenn.edu Center for Molecular Modeling -- University of Pennsylvania Department of Chemistry, 231 S.34th Street, Philadelphia, PA 19104-6323 tel: 1-215-898-1582, fax: 1-215-573-6233, office-tel: 1-215-898-5425 ======================================================================= If you make something idiot-proof, the universe creates a better idiot. From giannozz at nest.sns.it Tue Jan 15 12:31:15 2008 From: giannozz at nest.sns.it (Paolo Giannozzi) Date: Tue, 15 Jan 2008 12:31:15 +0100 Subject: [Pw_forum] Mulliken population In-Reply-To: <400395646.05061@mail.sdu.edu.cn> References: <400395646.05061@mail.sdu.edu.cn> Message-ID: On Jan 15, 2008, at 12:08 , KS Yang wrote: > Does the "effective charges" mentioned in the users-guide mean > the Mulliken population? they should be "Lowdin charges". Once upon a time I wrote the attached file to remember what the difference is. No warranty that what is written there is correct. Paolo -------------- next part -------------- A non-text attachment was scrubbed... Name: mulliken.tex Type: application/octet-stream Size: 3558 bytes Desc: not available Url : http://www.democritos.it/pipermail/pw_forum/attachments/20080115/47aa6f6b/attachment.obj -------------- next part -------------- --- Paolo Giannozzi, Dept of Physics, University of Udine via delle Scienze 208, 33100 Udine, Italy Phone +39-0432-558216, fax +39-0432-558222 From helen at fh.huji.ac.il Tue Jan 15 13:13:51 2008 From: helen at fh.huji.ac.il (Helen) Date: Tue, 15 Jan 2008 14:13:51 +0200 Subject: [Pw_forum] wurtzite AlN References: <001e01c85761$3fbc04f0$cd604084@fh.huji.ac.il> <20080115120654.B80605@rosa.physik-pool.tu-berlin.de> Message-ID: <003e01c85770$16f57d80$cd604084@fh.huji.ac.il> Thank you! I'm now reading up on the wurtzite unit cell to make sure I get my input file correct, Helen ----- Original Message ----- From: "Marcel Mohr" To: "PWSCF Forum" Sent: Tuesday, January 15, 2008 1:07 PM Subject: Re: [Pw_forum] wurtzite AlN > Hi, > > wurtzite has 4 atoms in the unit cell. > > Cheers Marcel > > > On Tue, 15 Jan 2008, Helen wrote: > >> Hi, >> I've successfully calculated the bandstructure for zinc-blende AlN, and I >> wanted to do the same for wurtzite AlN but am running into problems. I >> started off with a=5.88 a.u. c/a=1.600 and u=0.382 , I then kept a and c >> constant and altered u. The results showed that u needed to be smaller >> but when I reached u=0.369 my scf calculations stopped converging. I >> tried using both cg and davidson diagonalization. I then tried keeping u >> constant and altering c, I don't get convergence problems but I get >> unrealistically small values of c. Also the bandgap data I'm getting is >> nonsense (no band gap at all). Perhaps I have a mistake in my input file? >> Thank you for your help and advice, >> Dr. Helen Eisenberg (Fritz Haber Center, Hebrew University) >> >> &control >> calculation='scf' >> restart_mode='from_scratch', >> tstress = .true. >> tprnfor = .true. >> prefix='alas', >> pseudo_dir = '$PSEUDO_DIR/', >> outdir='$TMP_DIR/' >> / >> &system >> ibrav= 4, celldm(1) =5.88, celldm(3)=1.600, nat= 2, ntyp= 2, >> ecutwfc =80.0, nbnd = 8, >> nr1=32, >> nr2=32, >> nr3=32, >> ecutrho = 240.0 >> / >> &electrons >> conv_thr = 1.0d-6 >> mixing_beta = 0.7 >> / >> ATOMIC_SPECIES >> Al 26.98 Al.vbc.UPF >> N 14.007 N.pz-vbc.UPF >> ATOMIC_POSITIONS >> Al 0.00 0.00 0.00 >> N 0.00 0.00 0.611 >> K_POINTS (automatic) >> 4 4 4 0 0 0 >> EOF > > _______________________________________________ > Pw_forum mailing list > Pw_forum at pwscf.org > http://www.democritos.it/mailman/listinfo/pw_forum > From baroni at sissa.it Tue Jan 15 13:36:39 2008 From: baroni at sissa.it (Stefano Baroni) Date: Tue, 15 Jan 2008 13:36:39 +0100 Subject: [Pw_forum] Eigenfunctions in PWSCF In-Reply-To: References: Message-ID: <785816A1-D7A7-4AB3-99C4-EBCB3383898D@sissa.it> On Jan 15, 2008, at 9:00 AM, Min Seung Kyu wrote: > Dear users and developers, > > I have a question about eigenfunctions in PWSCF codes. > > I looked at PWSCF source files and I found that the 'evc' variable is > KS eigenvectors in G-space, right? > > Because Hamiltonian matrix H(G,G') is hermitian, 'evc' should be real. Who said this? The eigenvalues of a Hermitean matrix are real, the eigenvectors not necessarily so. If in addition to being Hermitean the matrix is also real (which implies it is symmetric), the the eigenvectors CAN BE CHOSEN to be real. If you feed a real matrix to a complex eigenvalue solver, then the eigenvectors will most probably come out complex, but can they be easily converted to real (if needed) by an appropriate choice of phases (again, if the original matrix is real symmetric, not merely Hermitean) Hope this helps Stefano > > > When I print out 'evc', however, 'evc' contains imaginary parts. > > For example, when I run 'co.rx.in' (example03 folder), I got these > values: > > DBLE(evc(1:4,1)) 0.186884 0.154551 0.154551 0.142085 > AIMAG(evc(1:4,1)) 0.000000 0.000000 0.000000 -0.048560 > DBLE(evc(1:4,2)) -0.024799 -0.021751 -0.021751 -0.068645 > AIMAG(evc(1:4,2)) 0.000000 0.000000 0.000000 -0.145963 > DBLE(evc(1:4,3)) 0.000000 0.000000 0.000000 0.000000 > AIMAG(evc(1:4,3)) 0.000000 -0.005668 0.152298 0.000000 > DBLE(evc(1:4,4)) 0.000000 0.000000 0.000000 0.000000 > AIMAG(evc(1:4,4)) 0.000000 -0.152298 -0.005668 0.000000 > DBLE(evc(1:4,5)) -0.299154 -0.185970 -0.185970 0.011252 > AIMAG(evc(1:4,5)) 0.000000 0.000000 0.000000 0.168998 > > ( I added 'WRITE(*,*) evc(1:4,ibnd)' in 'sum_band.f90'. ) > > What is the problem? > > Thank you in advance. > > Sincerely, Min. > > -------------------------------Sig.--------------------------- > Center for Superfunctional Materials, Department of Chemistry, > Pohang University of Science and Technology (POSTECH) > > Mr. Seung Kyu, Min > > Ph.D. Candidate > > E-Mail: min0220 at postech.ac.kr > Tel. : 82-54-279-5858 > ---------------------------------------------- > > > > > > _______________________________________________ > Pw_forum mailing list > Pw_forum at pwscf.org > http://www.democritos.it/mailman/listinfo/pw_forum --- Stefano Baroni - SISSA & DEMOCRITOS National Simulation Center - Trieste [+39] 040 3787 406 (tel) -528 (fax) / stefanobaroni (skype) Please, if possible, don't send me MS Word or PowerPoint attachments Why? See: http://www.gnu.org/philosophy/no-word-attachments.html -------------- next part -------------- An HTML attachment was scrubbed... URL: http://www.democritos.it/pipermail/pw_forum/attachments/20080115/c47780b8/attachment-0001.htm From Andreas.Kissavos at onera.fr Tue Jan 15 14:30:22 2008 From: Andreas.Kissavos at onera.fr (Andreas Kissavos) Date: Tue, 15 Jan 2008 14:30:22 +0100 Subject: [Pw_forum] Question about tests Message-ID: <0BA30F62-67EB-4C5B-A214-0547B2AC108C@zig> Hi, after compiling, I ran most of the test included. example01 produced identical results, while in example02, the Si Gamma point calculation, everything is identical up to the inversion of the dynamical matrix. After the inversion, my first 3 frequencies deviate a bit from the reference ones: My are 0.101977 [THz] while the reference is 0.093301 [THz]. Note that everything is identical (to 6 decimals at least) before this. The same thing occurs for all other examples where the dynamical matrix is inverted, while all other examples produce perfect agreement. I noticed that the examples were calculated with version 3.2, while my calculations were performed with the latest one, 3.2.3. I also note that there has been some problems with phonons earlier. My question now is: Has anything been changed with regards to this inversion between 3.2 and 3.2.3 that can account for the error I am getting? I certainly hope so, since I am not too fond of the idea of having defect LAPACK-routines. Best regards, Andreas Kissavos From min0220 at postech.ac.kr Tue Jan 15 15:14:44 2008 From: min0220 at postech.ac.kr (Min Seung Kyu) Date: Tue, 15 Jan 2008 23:14:44 +0900 (KST) Subject: [Pw_forum] Eigenfunctions in PWSCF In-Reply-To: <785816A1-D7A7-4AB3-99C4-EBCB3383898D@sissa.it> References: <785816A1-D7A7-4AB3-99C4-EBCB3383898D@sissa.it> Message-ID: Oops, I made a mistake. :-( You are totally right. Thank you for your reply. > > On Jan 15, 2008, at 9:00 AM, Min Seung Kyu wrote: > >> Dear users and developers, >> >> I have a question about eigenfunctions in PWSCF codes. >> >> I looked at PWSCF source files and I found that the 'evc' variable is >> KS eigenvectors in G-space, right? >> >> Because Hamiltonian matrix H(G,G') is hermitian, 'evc' should be real. > > Who said this? > > The eigenvalues of a Hermitean matrix are real, the eigenvectors not > necessarily so. If in addition to being Hermitean the matrix is also real > (which implies it is symmetric), the the eigenvectors CAN BE CHOSEN to be > real. If you feed a real matrix to a complex eigenvalue solver, then the > eigenvectors will most probably come out complex, but can they be easily > converted to real (if needed) by an appropriate choice of phases (again, if > the original matrix is real symmetric, not merely Hermitean) > > Hope this helps > Stefano > >> >> >> When I print out 'evc', however, 'evc' contains imaginary parts. >> >> For example, when I run 'co.rx.in' (example03 folder), I got these values: >> >> DBLE(evc(1:4,1)) 0.186884 0.154551 0.154551 0.142085 >> AIMAG(evc(1:4,1)) 0.000000 0.000000 0.000000 -0.048560 >> DBLE(evc(1:4,2)) -0.024799 -0.021751 -0.021751 -0.068645 >> AIMAG(evc(1:4,2)) 0.000000 0.000000 0.000000 -0.145963 >> DBLE(evc(1:4,3)) 0.000000 0.000000 0.000000 0.000000 >> AIMAG(evc(1:4,3)) 0.000000 -0.005668 0.152298 0.000000 >> DBLE(evc(1:4,4)) 0.000000 0.000000 0.000000 0.000000 >> AIMAG(evc(1:4,4)) 0.000000 -0.152298 -0.005668 0.000000 >> DBLE(evc(1:4,5)) -0.299154 -0.185970 -0.185970 0.011252 >> AIMAG(evc(1:4,5)) 0.000000 0.000000 0.000000 0.168998 >> >> ( I added 'WRITE(*,*) evc(1:4,ibnd)' in 'sum_band.f90'. ) >> >> What is the problem? >> >> Thank you in advance. >> >> Sincerely, Min. >> >> -------------------------------Sig.--------------------------- >> Center for Superfunctional Materials, Department of Chemistry, >> Pohang University of Science and Technology (POSTECH) >> >> Mr. Seung Kyu, Min >> >> Ph.D. Candidate >> >> E-Mail: min0220 at postech.ac.kr >> Tel. : 82-54-279-5858 >> ---------------------------------------------- >> >> >> >> >> >> _______________________________________________ >> Pw_forum mailing list >> Pw_forum at pwscf.org >> http://www.democritos.it/mailman/listinfo/pw_forum > > --- > Stefano Baroni - SISSA & DEMOCRITOS National Simulation Center - Trieste > [+39] 040 3787 406 (tel) -528 (fax) / stefanobaroni (skype) > > Please, if possible, don't send me MS Word or PowerPoint attachments > Why? See: http://www.gnu.org/philosophy/no-word-attachments.html > > > From giannozz at nest.sns.it Tue Jan 15 16:34:25 2008 From: giannozz at nest.sns.it (Paolo Giannozzi) Date: Tue, 15 Jan 2008 16:34:25 +0100 Subject: [Pw_forum] Question about tests In-Reply-To: <0BA30F62-67EB-4C5B-A214-0547B2AC108C@zig> References: <0BA30F62-67EB-4C5B-A214-0547B2AC108C@zig> Message-ID: <0D074E66-4E23-4EA7-B590-5FE5454B10CC@nest.sns.it> On Jan 15, 2008, at 14:30 , Andreas Kissavos wrote: > everything is identical up to the inversion of the dynamical matrix the dynamic al matrix is diagonalized, not inverted > My are 0.101977 [THz] while the reference is 0.093301 [THz]. > Note that everything is identical (to 6 decimals at least) before > this. in the specific case: those frequencies should be exactly zero, so I wouldn't mind too much if the "almost zero" you get is not exactly the same "almost zero" of the examples (the reason for nonzero acoustic frequencies is explained in the user guide). In general: it would be nice to have reproducibility of the printed figures, but I am afraid it is a lost cause: there are too many places where small differences show up for no apparent reason. If anybody has any idea on possible actions, he/she is welcome. P. --- Paolo Giannozzi, Dept of Physics, University of Udine via delle Scienze 208, 33100 Udine, Italy Phone +39-0432-558216, fax +39-0432-558222 From amigliore at cmm.upenn.edu Tue Jan 15 18:02:59 2008 From: amigliore at cmm.upenn.edu (Agostino Migliore) Date: Tue, 15 Jan 2008 12:02:59 -0500 (EST) Subject: [Pw_forum] Mulliken population In-Reply-To: <001601c85767$024c4520$5120c2ca@flysky> References: <001601c85767$024c4520$5120c2ca@flysky> Message-ID: <3935.130.91.67.162.1200416579.squirrel@cmm.upenn.edu> Hello Kesong The effective charges mentioned in the user-guide are to be considered within response theory. Moreover, if you want to reason on an atomic basis, they should correspond to Lowdin charges. Instead, as stressed by Axel, it is not appropriate and not "directly" feasible to look at Mulliken population (which already has various shortcomings by itself, although can be useful for qualitative, comparative analyses) from data by a plane-wave code. A good possibility is to exploit the information from the projwfc post-processing. This is anyway a necessary step if you insist in making a further population analysis through some home-made program. If, on the other hand, your aim is to quantify, in some manner, charge distribution, you can use pp.x. As a matter of fact, from this post-processing, you can also get the overall charge distribution which fits the resulting electrostatic potential (to this aim, anyhow, Mulliken charges should not be good). In fact, through the option plot_num=11 (in 3.2 version) you can get the electrostatic potential. Hence, by exploiting Poisson equation, you can write an easy program to get charge distribution (anyway, never forget that atomic cores are treated by means of pseudopotentials). The latter takes rightly into account valence electron distribution. So, it is much better than any point-charge schemes (like, e.g., resp charges in NWChem). I hope my information is useful to you Best, Agostino Migliore > Hi, everyone > > It is very easy to obtain the Mulliken population of the atom in the > output file using CASTEP or VASP code. > > I am sorry but i cann't find it in the output file of PWSCF code. Somebody > can tell me where can i find it? > > Does the "effective charges" mentioned in the users-guide mean the > Mulliken population? > > > Best wishes. > > > Kesong Yang > > School of Physics and Microelectronics > > Shandong University, Jinan 250100, People??s Republic of China > > Email: ksyang at mail.sdu.edu.cn; > yupiaofei at mail.sdu.edu.cn > _______________________________________________ > Pw_forum mailing list > Pw_forum at pwscf.org > http://www.democritos.it/mailman/listinfo/pw_forum > From konstantin_kudin at yahoo.com Tue Jan 15 21:43:21 2008 From: konstantin_kudin at yahoo.com (Konstantin Kudin) Date: Tue, 15 Jan 2008 12:43:21 -0800 (PST) Subject: [Pw_forum] Mulliken population In-Reply-To: <7b6913e90801150322x6c6f814eudb98f9bb5277b12@mail.gmail.com> Message-ID: <310324.1999.qm@web57702.mail.re3.yahoo.com> --- Axel Kohlmeyer wrote: > On 1/15/08, KS Yang wrote: > > > can tell me where can i find it? > > mulliken analysis requires the an "atomic" basis set, the > codes in the quantum espresso package use plane wave basis sets, > so you first have to project the wavefunction from one basis on > another. > of course this projection is always incomplete, so you'll have to > treat > the results with even more care than from codes that use local basis > set. > for details on how to project, have a look at Doc/INPUT_PROJWFC. Well, the original question does raise a common recurring issue, if one abstracts beyond the specifics of it. Many codes do a bit of basic inexpensive analysis after the calculation, and PW output is not very wordy in this respect :-) At some point I did calculate Lowdin, and the scripting part of it was not very convenient since a different code needed to be used. Probably 90% of the users (I just made this number up) would prefer to have a keyword in PW for Lowdin, which is infinitely easier than adding yet another input and script. Kostya ____________________________________________________________________________________ Looking for last minute shopping deals? Find them fast with Yahoo! Search. http://tools.search.yahoo.com/newsearch/category.php?category=shopping From w2agz at pacbell.net Wed Jan 16 04:36:45 2008 From: w2agz at pacbell.net (Paul M. Grant) Date: Tue, 15 Jan 2008 19:36:45 -0800 Subject: [Pw_forum] UPF-PP File Naming Convention Message-ID: <012101c857f1$174a76b0$45df6410$@net> The file naming convention within the library of PPs on O-Sesame and PWscf seems rather arcane. I can figure out most, but one that has me stumped is -d- vs -nd-. "d-orbitals," "norm-conserving?" Paul M. Grant, PhD Principal, W2AGZ Technologies Visiting Scholar, Applied Physics, Stanford University EPRI Science Fellow (Retired) IBM Research Staff Member Emeritus w2agz at pacbell.net http://www.w2agz.com -------------- next part -------------- An HTML attachment was scrubbed... URL: http://www.democritos.it/pipermail/pw_forum/attachments/20080115/67c2cd42/attachment.htm From giannozz at nest.sns.it Wed Jan 16 08:14:15 2008 From: giannozz at nest.sns.it (Paolo Giannozzi) Date: Wed, 16 Jan 2008 08:14:15 +0100 Subject: [Pw_forum] UPF-PP File Naming Convention In-Reply-To: <012101c857f1$174a76b0$45df6410$@net> References: <012101c857f1$174a76b0$45df6410$@net> Message-ID: On Jan 16, 2008, at 4:36 , Paul M. Grant wrote: > The file naming convention within the library of PPs on O-Sesame > and PWscf seems rather arcane. I can figure out most, but one that > has me stumped is ?d- vs ?nd-. ?d-orbitals,? ?norm-conserving?? > explained in Doc/nomefile.upf P. --- Paolo Giannozzi, Dept of Physics, University of Udine via delle Scienze 208, 33100 Udine, Italy Phone +39-0432-558216, fax +39-0432-558222 From giannozz at nest.sns.it Wed Jan 16 11:29:05 2008 From: giannozz at nest.sns.it (Paolo Giannozzi) Date: Wed, 16 Jan 2008 11:29:05 +0100 Subject: [Pw_forum] Raman intensities for "metallic systems" In-Reply-To: <20080114105053.fwqv24sym8w484oo@www.staffmail.ed.ac.uk> References: <20080114105053.fwqv24sym8w484oo@www.staffmail.ed.ac.uk> Message-ID: <90BD866D-30FE-4453-9E95-C464E5A423A0@nest.sns.it> On Jan 14, 2008, at 11:50 , Miriam Marques wrote: > I am emailing a previous unanswered question. > Is it possible to calculate Raman intensities for "metallic systems" > such as hcp-metals: Zn, Fe, Zr, Mg, high pressure phases of > oxygen or high pressure phases of alkali metals using the > ESPRESSO code? it depends: what is the expression of Raman intensities in "metallic systems"? The usual definition of Raman intensities in the nonresonant limit via derivatives of the polarizabily (this is what is calculated in Q-E) does not apply to metals. P. --- Paolo Giannozzi, Dept of Physics, University of Udine via delle Scienze 208, 33100 Udine, Italy Phone +39-0432-558216, fax +39-0432-558222 From kondor.jess at gmail.com Wed Jan 16 15:09:28 2008 From: kondor.jess at gmail.com (Jess Kondor) Date: Wed, 16 Jan 2008 08:09:28 -0600 Subject: [Pw_forum] Berry Phase calculation for gdir=1,2 ; cvs version Message-ID: <1d9d5d9d0801160609p7ab0d24cjb0b0880759f8ee02@mail.gmail.com> Dear developers, According Changelog for CVS version, it is possible now to perform Berry Phase calculation for gdir=1 and gdir=2 . I've tried it for rhombohedral unit cell, but got the following message: .... ================================================== POLARIZATION CALCULATION !!! NOT THOROUGHLY TESTED !!! -------------------------------------------------- error: translated G= -1.00993858493275 -0.583088313876004 0.406263994517323 with crystal coordinates 0 0 1 corresponds to ng= 0 but G(ng)= 0.000000000000000E+000 0.000000000000000E+000 4.362056180567782E-319 probably because G_par is NOT a reciprocal lattice vector Possible choices as smallest G_par: i= 1 G= 1.00993858493275 -0.583088313876004 0.406263994517323 i= 2 G= 0.000000000000000E+000 -1.16617662775201 0.812527989034646 i= 3 G= -1.00993858493275 1.74926494162801 5.467848396278896E-014 i= 4 G= 2.01987716986549 0.000000000000000E+000 0.000000000000000E+000 i= 5 G= -1.00993858493275 -1.74926494162801 1.21879198355197 i= 6 G= 0.000000000000000E+000 2.33235325550402 -0.406263994517268 i= 7 G= -2.01987716986549 1.16617662775201 0.406263994517378 i= 8 G= 3.02981575479824 0.583088313876004 -0.406263994517323 i= 9 G= 1.00993858493275 2.91544156938002 -0.812527989034591 i= 10 G= -3.02981575479824 0.583088313876004 0.812527989034701 i= 11 G= -2.01987716986549 -2.33235325550402 1.62505597806929 i= 12 G= 1.00993858493275 -2.91544156938002 -2.84384796162148 i= 13 G= 2.01987716986549 3.49852988325603 -1.21879198355191 i= 14 G= -4.03975433973098 0.000000000000000E+000 ..... Lattice/reciprocal vectors are: crystal axes: (cart. coord. in units of a_0) a(1) = ( 0.495080 -0.285834 0.820485 ) a(2) = ( 0.000000 0.571669 0.820485 ) a(3) = ( -0.495080 -0.285834 0.820485 ) reciprocal axes: (cart. coord. in units 2 pi/a_0) b(1) = ( 1.009939 -0.583088 0.406264 ) b(2) = ( 0.000000 1.166177 0.406264 ) b(3) = ( -1.009939 -0.583088 0.406264 ) There is no error in case of gdir=3 . Does anybody have such problem before? Best wishes, -- ===================================== Jess Kondor PICYT, Av. Venustiano Carranza 2425-A, San Luis Potos? 78210, M?xico ===================================== -------------- next part -------------- An HTML attachment was scrubbed... URL: http://www.democritos.it/pipermail/pw_forum/attachments/20080116/fad19fb7/attachment.htm From justo001 at umn.edu Thu Jan 17 04:18:26 2008 From: justo001 at umn.edu (justo001 at umn.edu) Date: 16 Jan 2008 21:18:26 -0600 Subject: [Pw_forum] Question about d-like gap levels Message-ID: I have used ESPRESSO to relax a certain structure and compute the electronic band structure of a transition metal (TM) in MgO. This TM creates a pair of band gap levels. Now I need to compute the percentage of d-character (or the contribution of l=2) of those levels. My question is if there is already some feature in the PWSCF (or a companion program) to calculate such percentages. Thanks for your help, Joao Justo ------------------------------------------------------- Joao Francisco Justo Visiting Associate Professor Chemical Engineering and Materials Science University of Minnesota 117 Pleasant SE St., Minneapolis, MN 55455 Phone: 1 (612) 624-7425 ------------------------------------------------------- From umari at democritos.it Thu Jan 17 11:11:55 2008 From: umari at democritos.it (umari at democritos.it) Date: Thu, 17 Jan 2008 11:11:55 +0100 Subject: [Pw_forum] Berry Phase calculation for gdir=1,2 ; cvs version Message-ID: <20080117111155.qhkz25sz404w4ogk@mail.democritos.it> Dear Jess, It could be that k points where not string along the same direction defined by gdir, In the sense, let's suppose nppstr=3 if gdir=3, you should have something as K_POINTS (crystal) N number of total k_points 0. 0. 0. 1. 0. 0. 0.333333 1. 0. 0. 0.666666 1. 0.5 0. 0. 1. 0.5 0. 0.3333333 1. 0.5 0. 0.6666666 1. ...and so on... if for examples gdir = 1, the strings of 3 k_points must be in the direction 1, as: K_POINTS (crystal) N number of total k_points 0. 0. 0. 1. 0.333333 0. 0. 1. 0.6666666 0. 0. 1. 0. 0.5 0. 1. 0.333333 0.5 0. 1. 0.6666666 0.5 0. 1. ...and so on.. note that if gdir=i you are applying the electric field in the direction parallel to the i-th vector of the direct lattice. The electronic polarization is also given on that axis. The value for the ionic polarization is still not correct, but a cvs commit should arrive today or tomorrow. Paolo Umari Democritos, Trieste ---------------------------------------------------------------- This message was sent using IMP, the Internet Messaging Program. From h.zhang at ifw-dresden.de Thu Jan 17 13:43:43 2008 From: h.zhang at ifw-dresden.de (Hongbin Zhang) Date: Thu, 17 Jan 2008 13:43:43 +0100 (CET) Subject: [Pw_forum] Smearing for metal/insulator supercell In-Reply-To: Message-ID: Dear all pwscf user, I am trying to do some calculations of interfaces between metals and insulators. I am not so familiar with PWSCF, so would you please help me with questions below? 1) How to define whether a system is (or expected to be) insulating or conducting by parameters of PWSCF? (I guess we use "smearing" to do this, but not sure.) 2) for metal/insulator supercell, how to set the related parameters? If it's smearing, should I use a as small as possible smearing? Thank you in advance for any suggestions or comments. Best wishes, Hongbin From marzari at MIT.EDU Thu Jan 17 14:32:47 2008 From: marzari at MIT.EDU (Nicola Marzari) Date: Thu, 17 Jan 2008 08:32:47 -0500 Subject: [Pw_forum] Smearing for metal/insulator supercell In-Reply-To: References: Message-ID: <478F58FF.2040200@mit.edu> > 1) How to define whether a system is (or expected to be) insulating or > conducting by parameters of PWSCF? You could start by using fractional occupations, and smearing, and see, after selfconsistency, if there is a gap or not. > 2) for metal/insulator supercell, how to set the related parameters? If > it's smearing, should I use a as small as possible smearing? Generally speaking, you want the result in the limit of zero smearing. In that limit, a metallic system will usually requires an extremely fine k-point sampling, since you are trying to integrate a discontinuous function. So, you need to find a smearing that is not too large to give you incorrect results, but large enough that you can use a reasonable number of k-points to achieve full sampling accuracy. See Chap 4 of http://quasiamore.mit.edu/phd for an introduction. Generally speaking, a smearing of 0.3eV=0.022Ry is a conservative starting points. PWscf uses Ry for smearing, CP (I believe) Ha. nicola --------------------------------------------------------------------- Prof Nicola Marzari Department of Materials Science and Engineering 13-5066 MIT 77 Massachusetts Avenue Cambridge MA 02139-4307 USA tel 617.4522758 fax 2586534 marzari at mit.edu http://quasiamore.mit.edu From wangjunjie1981_0 at 163.com Thu Jan 17 15:59:12 2008 From: wangjunjie1981_0 at 163.com (wangjunjie1981_0) Date: Thu, 17 Jan 2008 22:59:12 +0800 (CST) Subject: [Pw_forum] about the core radii of Li and N Message-ID: <1715024428.675361200581952980.JavaMail.coremail@bj163app104.163.com> Dear pwscf users When i genernate the pseudopotentials of Li and N atoms, respectively, i find that the core radius of Li atom is bigger than that of N atom in original ini file. But Why? In general, one believes that the core radius of Li is smaller than that of N. Thank you very much for your advance! Junjie Wang -------------- next part -------------- An HTML attachment was scrubbed... URL: http://www.democritos.it/pipermail/pw_forum/attachments/20080117/a527cc4c/attachment.htm From paulatto at sissa.it Thu Jan 17 16:34:50 2008 From: paulatto at sissa.it (Lorenzo Paulatto) Date: Thu, 17 Jan 2008 16:34:50 +0100 (CET) Subject: [Pw_forum] about the core radii of Li and N In-Reply-To: <1715024428.675361200581952980.JavaMail.coremail@bj163app104.163.com> References: <1715024428.675361200581952980.JavaMail.coremail@bj163app104.163.com> Message-ID: <28107.147.122.5.182.1200584090.squirrel@webmail.sissa.it> On Thu, January 17, 2008 15:59, wangjunjie1981_0 wrote: > When i genernate the pseudopotentials of Li and N atoms, respectively, i > find that the core radius of Li atom is bigger than that of N atom in > original ini file. But Why? If you are referring to this ld1.x message: r > 0.41 : true rho core than it's not the core size, but a radius before which real core charge is replaced with a pseudized, softer, core charge. You can set that radius by hand (rcore=...) or let ld1 decide where to cut, the rule used is: rcore= matching radius (a.u.) for the smoothing of the core charge If not specified, the matching radius is determined by the condition rho_core(rcore) = 2*rho_valence(rcore) So the fact that rcore is smaller for N than for Li only means that the ratio rho_core/rho_valence become small enough sooner for N than for Li. In other words valence charge is higher and more localized in N than in Li, compared to their respective core charges. Hope it helps, goodbye. -- Lorenzo Paulatto SISSA & DEMOCRITOS (Trieste) +39 040 3787 511 http://people.sissa.it/~paulatto/ ---------------------------------------------------------------- SISSA Webmail https://webmail.sissa.it/ Powered by SquirrelMail http://www.squirrelmail.org/ From w2agz at pacbell.net Thu Jan 17 22:20:06 2008 From: w2agz at pacbell.net (Paul M. Grant) Date: Thu, 17 Jan 2008 13:20:06 -0800 Subject: [Pw_forum] ld1.x Questions Message-ID: <017801c8594e$bcf568a0$36e039e0$@net> I'm just beginning to learn how to construct PPs with the ld1.x tool, Opium and fhi. I understand PP theory, but their construction, as many point out, is more art than science. I can see one can spend an enormous amount of time acquiring the appropriate skill. I have the following questions re ld1.x: 1) What is the purpose of the "negative ocs(nl)" settings one sees frequently in the input files in /atomic_doc/pseudo_library? (not covered that I can see in INPUT_LD1) It doesn't seem to make physical sense. 2) Why doesn't this library contain some transition metal examples? 3) I've observed ld1.x from O-sesame (CVS) performs much better the executable in the present distribution espresso-3.2.3, which yields a number of formatting errors. 4) It's stated that ld1.x output can be plotted with Grace, but I see no options to produce or launch .agr files such as exists in Opium. I guess it has to be done "by hand." -Paul Paul M. Grant, PhD Principal, W2AGZ Technologies Visiting Scholar, Applied Physics, Stanford University EPRI Science Fellow (Retired) IBM Research Staff Member Emeritus w2agz at pacbell.net http://www.w2agz.com -------------- next part -------------- An HTML attachment was scrubbed... URL: http://www.democritos.it/pipermail/pw_forum/attachments/20080117/cee6061d/attachment.htm From h.zhang at ifw-dresden.de Thu Jan 17 22:26:16 2008 From: h.zhang at ifw-dresden.de (Hongbin Zhang) Date: Thu, 17 Jan 2008 22:26:16 +0100 (CET) Subject: [Pw_forum] Questions about ionic & cell relaxation. In-Reply-To: <478F58FF.2040200@mit.edu> Message-ID: Dear all pwscf users, I want to do some ionic relaxations, but there's one question unclear: Will the relaxation lower the original symmetry? The reason why I ask this is that I want to optimize the cell and ionic positions with some constraints, say, to keep the mirror symmetry along z direction of the unit cell. Any suggestion and comment are welcome. Thank you in advance. Best wishes, Hongbin Zhang From marzari at MIT.EDU Thu Jan 17 22:36:13 2008 From: marzari at MIT.EDU (Nicola Marzari) Date: Thu, 17 Jan 2008 16:36:13 -0500 Subject: [Pw_forum] Questions about ionic & cell relaxation. In-Reply-To: References: Message-ID: <478FCA4D.5030800@mit.edu> Hongbin Zhang wrote: > Dear all pwscf users, > > I want to do some ionic relaxations, but there's one question unclear: > > Will the relaxation lower the original symmetry? Ionic forces will satisfy all the symmetry operations that PWSCF has found, and so relaxations will preserve those symmetry operations. Generally speaking, you should be fine - if anything, one needs to be careful of the opposite problem, when the ionic ground state has less symmetry than the initial configuration (since the relaxations do not break the symmetry). nicola > > The reason why I ask this is that I want to optimize the cell and ionic > positions with some constraints, say, to keep the mirror symmetry along > z direction of the unit cell. > > Any suggestion and comment are welcome. Thank you in advance. > > Best wishes, > > Hongbin Zhang > > _______________________________________________ > Pw_forum mailing list > Pw_forum at pwscf.org > http://www.democritos.it/mailman/listinfo/pw_forum -- --------------------------------------------------------------------- Prof Nicola Marzari Department of Materials Science and Engineering 13-5066 MIT 77 Massachusetts Avenue Cambridge MA 02139-4307 USA tel 617.4522758 fax 2586534 marzari at mit.edu http://quasiamore.mit.edu From giannozz at nest.sns.it Thu Jan 17 23:21:11 2008 From: giannozz at nest.sns.it (Paolo Giannozzi) Date: Thu, 17 Jan 2008 23:21:11 +0100 Subject: [Pw_forum] Question about d-like gap levels In-Reply-To: References: Message-ID: On Jan 17, 2008, at 4:18 , justo001 at umn.edu wrote: > I have used ESPRESSO to relax a certain structure and compute > the electronic band structure of a transition metal (TM) in MgO. > This TM creates a pair of band gap levels. Now I need to compute > the percentage of d-character (or the contribution of l=2) of those > levels. My question is if there is already some feature in the PWSCF > (or a companion program) to calculate such percentages. projwfc.x calculates the projection of each electronic states over atomic states. The latter are stored in the pseudopotential files. Documentation in Doc/INPUT_PROJWFC, plus example 8 Paolo --- Paolo Giannozzi, Dept of Physics, University of Udine via delle Scienze 208, 33100 Udine, Italy Phone +39-0432-558216, fax +39-0432-558222 From giannozz at nest.sns.it Thu Jan 17 23:24:02 2008 From: giannozz at nest.sns.it (Paolo Giannozzi) Date: Thu, 17 Jan 2008 23:24:02 +0100 Subject: [Pw_forum] constrained molecular dynamics In-Reply-To: <1710.128.40.77.197.1200327520.squirrel@www.squirrelmail.ucl.ac.uk> References: <1710.128.40.77.197.1200327520.squirrel@www.squirrelmail.ucl.ac.uk> Message-ID: <3AA61842-840B-4982-B0F6-3AE6DFC447B7@nest.sns.it> On Jan 14, 2008, at 17:18 , Devis Di Tommaso wrote: > is it possible to perform constrained molecular dynamics > simulation with the present version of CP? at least a few constraints are implemented. Please have a look at the input documentation for CP (INPUT_CP) for cards CONSTRAINTS Paolo --- Paolo Giannozzi, Dept of Physics, University of Udine via delle Scienze 208, 33100 Udine, Italy Phone +39-0432-558216, fax +39-0432-558222 From ukizhu at gmail.com Fri Jan 18 08:32:12 2008 From: ukizhu at gmail.com (uki zhu) Date: Fri, 18 Jan 2008 15:32:12 +0800 Subject: [Pw_forum] how to get the number of plane waves Message-ID: Hi all, can anyone tell me how to get the number of plane waves used from the output files? thanks. uu zhu NUS, Singapore. -------------- next part -------------- An HTML attachment was scrubbed... URL: http://www.democritos.it/pipermail/pw_forum/attachments/20080118/e3f1dfef/attachment.htm From paulatto at sissa.it Fri Jan 18 09:53:49 2008 From: paulatto at sissa.it (Lorenzo Paulatto) Date: Fri, 18 Jan 2008 09:53:49 +0100 (CET) Subject: [Pw_forum] how to get the number of plane waves In-Reply-To: References: Message-ID: <5631.147.122.5.182.1200646429.squirrel@webmail.sissa.it> On Fri, January 18, 2008 08:32, uki zhu wrote: > Hi all, > can anyone tell me how to get the number of plane waves used from the > output > files? thanks. > Dear Uki Zhu, the number of plane waves is printed in output "as number of G vectors", e.g. from example01: G cutoff = 189.7462 ( 2733 G-vectors) FFT grid: ( 20, 20, 20) bye -- Lorenzo Paulatto SISSA & DEMOCRITOS (Trieste) +39 040 3787 511 http://people.sissa.it/~paulatto/ ---------------------------------------------------------------- SISSA Webmail https://webmail.sissa.it/ Powered by SquirrelMail http://www.squirrelmail.org/ From giannozz at nest.sns.it Fri Jan 18 09:58:46 2008 From: giannozz at nest.sns.it (Paolo Giannozzi) Date: Fri, 18 Jan 2008 09:58:46 +0100 Subject: [Pw_forum] how to get the number of plane waves In-Reply-To: <5631.147.122.5.182.1200646429.squirrel@webmail.sissa.it> References: <5631.147.122.5.182.1200646429.squirrel@webmail.sissa.it> Message-ID: <4B5F7B57-C793-4C9F-9318-0A73678E3B69@nest.sns.it> On Jan 18, 2008, at 9:53 , Lorenzo Paulatto wrote: >> can anyone tell me how to get the number of plane waves used from the >> output files? thanks. >> > > Dear Uki Zhu, > the number of plane waves is printed in output "as number of G > vectors", > e.g. from example01: > G cutoff = 189.7462 ( 2733 G-vectors) FFT grid: ( 20, 20, > 20) this is actually the number of plane waves in the charge density. The number of plane waves in the wavefunction is printed at the last iteration, together with the k-point and the Kohn-Sham eigenvalues: k = 0.1250 0.1250 0.1250 ( 335 PWs) bands (ev): -5.6039 4.6467 5.9568 5.9568 The maximum number across k-points is printed at the beginning as "npwx" P. --- Paolo Giannozzi, Dept of Physics, University of Udine via delle Scienze 208, 33100 Udine, Italy Phone +39-0432-558216, fax +39-0432-558222 From giannozz at nest.sns.it Fri Jan 18 10:27:15 2008 From: giannozz at nest.sns.it (Paolo Giannozzi) Date: Fri, 18 Jan 2008 10:27:15 +0100 Subject: [Pw_forum] ld1.x Questions In-Reply-To: <017801c8594e$bcf568a0$36e039e0$@net> References: <017801c8594e$bcf568a0$36e039e0$@net> Message-ID: On Jan 17, 2008, at 22:20 , Paul M. Grant wrote: > I understand PP theory, but their construction, as many point out, > is more art than science > I would rather say, "more cooking than science" > 1) What is the purpose of the ?negative ocs(nl)? settings one > sees frequently in the input files in /atomic_doc/pseudo_library? > (not covered that I can see in INPUT_LD1) > you didn't notice the following lines: Negative occupancies are used to flag unbound states; they are not actually used > It doesn?t seem to make physical sense. it doesn't! it is a quick and dirty (dirty more than quick) way to pass a piece of information to the code about the type of state under consideration > 2) Why doesn?t this library contain some transition metal > examples? > ...because they are by far the most difficult cases ?!? > 3) I?ve observed ld1.x from O-sesame (CVS) performs much > better the executable in the present distribution espresso-3.2.3, > which yields a number of formatting errors. > good to hear this and not the opposite. There have been many small changes in the cvs version, some of which aimed at better convergence and numerical stability. > 4) It?s stated that ld1.x output can be plotted with Grace, > but I see no options to produce or launch .agr files such as exists > in Opium. I guess it has to be done ?by hand.? > good guess. I think that the statement above just means that numbers are in a simple format (columns) that can be easily plotted (also with gnuplot). Anybody knowing Grace is encouraged to modify the code so that it directly produces .agr files Paolo --- Paolo Giannozzi, Dept of Physics, University of Udine via delle Scienze 208, 33100 Udine, Italy Phone +39-0432-558216, fax +39-0432-558222 From miguel.martinez at ehu.es Fri Jan 18 10:27:47 2008 From: miguel.martinez at ehu.es (=?windows-1252?Q?Miguel_Marti=ADnez?=) Date: Fri, 18 Jan 2008 10:27:47 +0100 Subject: [Pw_forum] ld1.x Questions In-Reply-To: <017801c8594e$bcf568a0$36e039e0$@net> References: <017801c8594e$bcf568a0$36e039e0$@net> Message-ID: <47907113.8060600@ehu.es> Dear Paul, From my *very limited* knowledge of pseudopotential generation, Paul M. Grant escribi?: > 1) What is the purpose of the ?negative ocs(nl)? settings one sees > frequently in the input files in /atomic_doc/pseudo_library? (not > covered that I can see in INPUT_LD1) It doesn?t seem to make physical > sense. Negative occupancies are used to flag unbound states. This could be useful when one wants to introduce an additional angular momentum channel and, let's say, the 3d orbital is unbound in the default atomic configuration. > 4) It?s stated that ld1.x output can be plotted with Grace, but I > see no options to produce or launch .agr files such as exists in Opium. > I guess it has to be done ?by hand.? Given the column formatting of the output, I usually go for gnuplot, although long lines are a pain* on an interactive gnuplot session. Regards, Miguel * Even more so when Debian/Ubuntu packages are built without filename completion. -- ---------------------------------------- Miguel Mart?nez Canales Dto. F?sica de la Materia Condensada UPV/EHU Facultad de Ciencia y Tecnolog?a Apdo. 644 48080 Bilbao (Spain) Fax: +34 94 601 3500 Tlf: +34 94 601 5326 ---------------------------------------- "If you have an apple and I have an apple and we exchange these apples then you and I will still each have one apple. But if you have an idea and I have an idea and we exchange these ideas, then each of us will have two ideas." George Bernard Shaw From paulatto at sissa.it Fri Jan 18 10:28:23 2008 From: paulatto at sissa.it (Lorenzo Paulatto) Date: Fri, 18 Jan 2008 10:28:23 +0100 (CET) Subject: [Pw_forum] ld1.x Questions In-Reply-To: <017801c8594e$bcf568a0$36e039e0$@net> References: <017801c8594e$bcf568a0$36e039e0$@net> Message-ID: <16351.147.122.5.182.1200648503.squirrel@webmail.sissa.it> On Thu, January 17, 2008 22:20, Paul M. Grant wrote: > 1) What is the purpose of the "negative ocs(nl)" settings one sees > frequently in the input files in /atomic_doc/pseudo_library? (not covered > that I can see in INPUT_LD1) It doesn't seem to make physical sense. Orbitals with negative occupation are excluded from the all-electron calculation; you need to do this when they are unbounded. Putting a negative occupation means: I don't really want to compute this orbital, but I'd like to use it's label later for a specific orbital identified by an arbitrary, but well defined, reference energy, and L and M indexes. There is probably a more formal way to say that. > 2) Why doesn't this library contain some transition metal examples? The examples in the library are the pseudopotentials that Andrea Dal Corso (and only he) has done during the years. There are a few transition metals among them in the CVS version. > 3) I've observed ld1.x from O-sesame (CVS) performs much better the > executable in the present distribution espresso-3.2.3, which yields a > number of formatting errors. There have been a few improvements on that front recently, nothing should really change but the number of things that can will cause crashes has been reduced a bit. Keep in mind that "input files which produce a good pseudo" are a tiny subset of "input files which doesn't cause a crash." > 4) It's stated that ld1.x output can be plotted with Grace, but I see > no options to produce or launch .agr files such as exists in Opium. I > guess it has to be done "by hand." I think you are right. > -Paul -- Lorenzo Paulatto SISSA & DEMOCRITOS (Trieste) +39 040 3787 511 http://people.sissa.it/~paulatto/ ---------------------------------------------------------------- SISSA Webmail https://webmail.sissa.it/ Powered by SquirrelMail http://www.squirrelmail.org/ From rcjhawk at gmail.com Sat Jan 19 00:09:43 2008 From: rcjhawk at gmail.com (Michael Mehl) Date: Fri, 18 Jan 2008 18:09:43 -0500 Subject: [Pw_forum] ld1.x Questions In-Reply-To: <47907113.8060600@ehu.es> References: <017801c8594e$bcf568a0$36e039e0$@net> <47907113.8060600@ehu.es> Message-ID: <479131B7.5010005@gmail.com> Miguel Marti?nez wrote: > Given the column formatting of the output, I usually go for gnuplot, > although long lines are a pain* on an interactive gnuplot session. Regards, > > Miguel > > * Even more so when Debian/Ubuntu packages are built without filename > completion. > > I can actually answer this one. See http://telin.ugent.be/~slippens/drupal/node/158 which removes one of the major annoyances of Ubuntugnuplot -- Michael J. Mehl Who works for the Naval Research Laboratory Which may or may not share his views on this or any subject And in any case he's writing from home. http://hawknotes.blogspot.com/ From w2agz at pacbell.net Sat Jan 19 01:26:42 2008 From: w2agz at pacbell.net (Paul M. Grant) Date: Fri, 18 Jan 2008 16:26:42 -0800 Subject: [Pw_forum] ld1.x Questions In-Reply-To: <16351.147.122.5.182.1200648503.squirrel@webmail.sissa.it> References: <017801c8594e$bcf568a0$36e039e0$@net> <16351.147.122.5.182.1200648503.squirrel@webmail.sissa.it> Message-ID: <01b301c85a31$f756b010$e6041030$@net> Thanks to all who replied. Additional comments follow: 1) Negative occupations. I should have guessed...or scanned INPUT_LD1 more thoroughly. Let me see if I got this right. In constructing a PP for sulfur, I should "unoccupy" the 3d states? But what about heavily oxidized sulfur molecules, like in SF6 where S3d participates in the bonding? I guess this is why PP cooking requires a good chef. 2) TM PP examples. This was, as we Yanks say, a purposely "loaded question." Or casting a net seeking pearls of wisdom. Those following my LDA+U thread know I'm looking for a Cu PP (and probably an O partner) that will yield a gap in cuprate compounds (nominally divalent Cu-O bonds). I strongly suspect given the notoriety of this family of structures, many have tried and failed. The only formalism that I know which has succeeded in this endeavor has been LDA+U on an LMTO basis set. The Cu PP issue may perhaps be the most interesting challenge facing PP "theory and practice" today. Thanks for pointing out that the CVS /pseudo_library/... does contain some TM examples, but they're 4d and 5d. As far as I know, none of their monoxides are Hubbard insulators (if indeed they can be formed chemically at all...during the pandemonium of early Spring, 1987, my group at IBM Almaden mined the whole 4 and 5 series searching for HTSCs with no hits. Those were very heady and na?ve days.). 2a) I'm going to try to "reverse engineer" Fe.pz-nd-rrkjus.UPF used in example 25 into an ".in" file (does anyone have one?) to try and understand better how ld1.x works (an O.pz-rrkjus.in file already exists in /atomic_doc/pseudo_library/LDA/SR, but the O.pz-rrkjus.UPF produced by CVS ld1.x is "somewhat" different than that contained in the distro library /pseudo and used in example 25 (different total energies by a few tenths of a percent, no E pseu and no wfc and rho suggested cutoffs in the latter and fewer mesh points in the former ???)). Simplistically, Fe++ is the minority spin electron analog of the minority spin hole state expected in Cu++. 3) ld1.x - CVS vs Espresso. Espresso ld1.x screws up the &input namelist sequence, especially the "author" entry (is "ADC" the mysterious Andrea Dal Corso?), and about half the time fails to properly format the total energy (all asterisks in the UPF output, likely a Fortran format placeholder limit problem). 4) Grace output. Another loaded question. I anticipated Paolo's answer, but was hoping that some saintly soul in a state of grace...usually a postdoc...in the PWscf community had taken this task on. Paul M. Grant, PhD Principal, W2AGZ Technologies Visiting Scholar, Applied Physics, Stanford University EPRI Science Fellow (Retired) IBM Research Staff Member Emeritus w2agz at pacbell.net http://www.w2agz.com ? ? -----Original Message----- From: pw_forum-bounces at pwscf.org [mailto:pw_forum-bounces at pwscf.org] On Behalf Of Lorenzo Paulatto Sent: Friday, January 18, 2008 1:28 AM To: pw_forum at pwscf.org Subject: Re: [Pw_forum] ld1.x Questions On Thu, January 17, 2008 22:20, Paul M. Grant wrote: > 1) What is the purpose of the "negative ocs(nl)" settings one sees > frequently in the input files in /atomic_doc/pseudo_library? (not covered > that I can see in INPUT_LD1) It doesn't seem to make physical sense. Orbitals with negative occupation are excluded from the all-electron calculation; you need to do this when they are unbounded. Putting a negative occupation means: I don't really want to compute this orbital, but I'd like to use it's label later for a specific orbital identified by an arbitrary, but well defined, reference energy, and L and M indexes. There is probably a more formal way to say that. > 2) Why doesn't this library contain some transition metal examples? The examples in the library are the pseudopotentials that Andrea Dal Corso (and only he) has done during the years. There are a few transition metals among them in the CVS version. > 3) I've observed ld1.x from O-sesame (CVS) performs much better the > executable in the present distribution espresso-3.2.3, which yields a > number of formatting errors. There have been a few improvements on that front recently, nothing should really change but the number of things that can will cause crashes has been reduced a bit. Keep in mind that "input files which produce a good pseudo" are a tiny subset of "input files which doesn't cause a crash." > 4) It's stated that ld1.x output can be plotted with Grace, but I see > no options to produce or launch .agr files such as exists in Opium. I > guess it has to be done "by hand." I think you are right. > -Paul -- Lorenzo Paulatto SISSA & DEMOCRITOS (Trieste) +39 040 3787 511 http://people.sissa.it/~paulatto/ ---------------------------------------------------------------- SISSA Webmail https://webmail.sissa.it/ Powered by SquirrelMail http://www.squirrelmail.org/ _______________________________________________ Pw_forum mailing list Pw_forum at pwscf.org http://www.democritos.it/mailman/listinfo/pw_forum From w2agz at pacbell.net Sat Jan 19 03:14:15 2008 From: w2agz at pacbell.net (Paul M. Grant) Date: Fri, 18 Jan 2008 18:14:15 -0800 Subject: [Pw_forum] ld1.x Questions In-Reply-To: <479131B7.5010005@gmail.com> References: <017801c8594e$bcf568a0$36e039e0$@net> <47907113.8060600@ehu.es> <479131B7.5010005@gmail.com> Message-ID: <01ba01c85a40$fe239250$fa6ab6f0$@net> Thanks, Michael. I'll have a look. BTW, we may have met...one of my buddies is Don Gubser from your shop, along with a number of others, including Warren Pickett before he went off to UC Davis. I like your "sign-off." Maybe I'll steal it except I'm now retired and not in the pay of any institution any more and always writing from home! Paul M. Grant, PhD Principal, W2AGZ Technologies Visiting Scholar, Applied Physics, Stanford University EPRI Science Fellow (Retired) IBM Research Staff Member Emeritus w2agz at pacbell.net http://www.w2agz.com ? ? -----Original Message----- From: pw_forum-bounces at pwscf.org [mailto:pw_forum-bounces at pwscf.org] On Behalf Of Michael Mehl Sent: Friday, January 18, 2008 3:10 PM To: PWSCF Forum Subject: Re: [Pw_forum] ld1.x Questions Miguel Marti?nez wrote: > Given the column formatting of the output, I usually go for gnuplot, > although long lines are a pain* on an interactive gnuplot session. Regards, > > Miguel > > * Even more so when Debian/Ubuntu packages are built without filename > completion. > > I can actually answer this one. See http://telin.ugent.be/~slippens/drupal/node/158 which removes one of the major annoyances of Ubuntugnuplot -- Michael J. Mehl Who works for the Naval Research Laboratory Which may or may not share his views on this or any subject And in any case he's writing from home. http://hawknotes.blogspot.com/ _______________________________________________ Pw_forum mailing list Pw_forum at pwscf.org http://www.democritos.it/mailman/listinfo/pw_forum From oulihui666 at 126.com Sat Jan 19 12:38:25 2008 From: oulihui666 at 126.com (oulihui666) Date: Sat, 19 Jan 2008 19:38:25 +0800 (CST) Subject: [Pw_forum] About spin Message-ID: <19310813.277751200742705660.JavaMail.coremail@bj126app81.126.com> Dear pwscf users, I want to ask a question, when I calculate with pw.x, I use the choice of about spin, that is to say, nspin=2, so I will specify the starting_magnetization. I check the email list, and I find that total_magnetization=(number of unpaired electrons)/(total numbers of electrons), starting_magnetization is the initial value of it. For example, when the system contains Pt, O, H atoms, I specify starting_magnetization=0.2 for Pt atom, starting_magnetization=0.5 for O atom, starting_magnetization=1.0 for H atom. Is it correct and appropriate about my settings? Thanks in advance Lihui Ou -- ====================================== Lihui Ou PH.D Candidate in Electrochemistry College of Chemistry and Molecular Science Wuhan University,430072,Hubei Province,China E-mail:oulihui666 at 126.com ====================================== -------------- next part -------------- An HTML attachment was scrubbed... URL: http://www.democritos.it/pipermail/pw_forum/attachments/20080119/e90ceeb4/attachment.htm From h.zhang at ifw-dresden.de Sat Jan 19 19:13:08 2008 From: h.zhang at ifw-dresden.de (Hongbin Zhang) Date: Sat, 19 Jan 2008 19:13:08 +0100 (CET) Subject: [Pw_forum] Questions about ionic & cell relaxation. In-Reply-To: <478FCA4D.5030800@mit.edu> Message-ID: Dear Nicola and all users, Thanks for you comments. But there're still something unclear for me: 1) if the symmetry will be kept, then how can I break the symmetry if necessary? 2) I think there is a Keyword "nosym" in the namelist "&SYSTEM", is it somehow related to structure relaxation? Since in example 03 for Al(001) slab, when relaxeing, "nosym = .true" is used. 3) In "ATOMIC_POSITIONS" catalog, it's possible to set "if_pos" terms, which will make forces along some directions to be zero. How should I combine these options with "nosym" option? Maybe it would be more clear if I show you my intended calculations: basically I want to optimize atomic positions of a metal/insulator/metal supercell only along z direction (x&y coords won't change), and there're two steps: 1) optimization with mirror symmetry along z direction 2) optimization without mirror symmetry along z direction So if the original supercell have the z-mirror symmetry, how to break it if relaxation will keep all symmetry pwscf find (nicola)? Many thanks for your comments and suggestions. Best wishes, Hongbin On Thu, 17 Jan 2008, Nicola Marzari wrote: > Hongbin Zhang wrote: > > Dear all pwscf users, > > > > I want to do some ionic relaxations, but there's one question unclear: > > > > Will the relaxation lower the original symmetry? > > > Ionic forces will satisfy all the symmetry operations that PWSCF > has found, and so relaxations will preserve those symmetry operations. > > Generally speaking, you should be fine - if anything, one needs > to be careful of the opposite problem, when the ionic ground state has > less symmetry than the initial configuration (since the relaxations > do not break the symmetry). > > nicola > > > > > > > The reason why I ask this is that I want to optimize the cell and ionic > > positions with some constraints, say, to keep the mirror symmetry along > > z direction of the unit cell. > > > > Any suggestion and comment are welcome. Thank you in advance. > > > > Best wishes, > > > > Hongbin Zhang > > > > _______________________________________________ > > Pw_forum mailing list > > Pw_forum at pwscf.org > > http://www.democritos.it/mailman/listinfo/pw_forum > > > -- > --------------------------------------------------------------------- > Prof Nicola Marzari Department of Materials Science and Engineering > 13-5066 MIT 77 Massachusetts Avenue Cambridge MA 02139-4307 USA > tel 617.4522758 fax 2586534 marzari at mit.edu http://quasiamore.mit.edu > _______________________________________________ > Pw_forum mailing list > Pw_forum at pwscf.org > http://www.democritos.it/mailman/listinfo/pw_forum > From akohlmey at cmm.chem.upenn.edu Sat Jan 19 19:28:09 2008 From: akohlmey at cmm.chem.upenn.edu (Axel Kohlmeyer) Date: Sat, 19 Jan 2008 18:28:09 +0000 Subject: [Pw_forum] Questions about ionic & cell relaxation. In-Reply-To: References: <478FCA4D.5030800@mit.edu> Message-ID: <7b6913e90801191028i7e04e694ufdd76055bbdd3412@mail.gmail.com> On 1/19/08, Hongbin Zhang wrote: > > Dear Nicola and all users, > > Thanks for you comments. But there're still something unclear for me: > > 1) if the symmetry will be kept, then how can I break the symmetry if > necessary? > > 2) I think there is a Keyword "nosym" in the namelist "&SYSTEM", is it > somehow related to structure relaxation? Since in example 03 for Al(001) > slab, when relaxeing, "nosym = .true" is used. > > 3) In "ATOMIC_POSITIONS" catalog, it's possible to set "if_pos" terms, > which will make forces along some directions to be zero. How should I > combine these options with "nosym" option? > > Maybe it would be more clear if I show you my intended calculations: > > basically I want to optimize atomic positions of a metal/insulator/metal > supercell only along z direction (x&y coords won't change), and there're > two steps: > > 1) optimization with mirror symmetry along z direction > > 2) optimization without mirror symmetry along z direction > > So if the original supercell have the z-mirror symmetry, how to break it > if relaxation will keep all symmetry pwscf find (nicola)? if you want to break the symmetry, just move 1-2 atoms a bit (or a bit more), so that symmetry is broken (enough). cheers, axel. > > Many thanks for your comments and suggestions. > > Best wishes, > > Hongbin > > > > > > > > > On Thu, 17 Jan 2008, Nicola Marzari wrote: > > > Hongbin Zhang wrote: > > > Dear all pwscf users, > > > > > > I want to do some ionic relaxations, but there's one question unclear: > > > > > > Will the relaxation lower the original symmetry? > > > > > > Ionic forces will satisfy all the symmetry operations that PWSCF > > has found, and so relaxations will preserve those symmetry operations. > > > > Generally speaking, you should be fine - if anything, one needs > > to be careful of the opposite problem, when the ionic ground state has > > less symmetry than the initial configuration (since the relaxations > > do not break the symmetry). > > > > nicola > > > > > > > > > > > > The reason why I ask this is that I want to optimize the cell and ionic > > > positions with some constraints, say, to keep the mirror symmetry along > > > z direction of the unit cell. > > > > > > Any suggestion and comment are welcome. Thank you in advance. > > > > > > Best wishes, > > > > > > Hongbin Zhang > > > > > > _______________________________________________ > > > Pw_forum mailing list > > > Pw_forum at pwscf.org > > > http://www.democritos.it/mailman/listinfo/pw_forum > > > > > > -- > > --------------------------------------------------------------------- > > Prof Nicola Marzari Department of Materials Science and Engineering > > 13-5066 MIT 77 Massachusetts Avenue Cambridge MA 02139-4307 USA > > tel 617.4522758 fax 2586534 marzari at mit.edu http://quasiamore.mit.edu > > _______________________________________________ > > Pw_forum mailing list > > Pw_forum at pwscf.org > > http://www.democritos.it/mailman/listinfo/pw_forum > > > > _______________________________________________ > Pw_forum mailing list > Pw_forum at pwscf.org > http://www.democritos.it/mailman/listinfo/pw_forum > > -- ======================================================================= Axel Kohlmeyer akohlmey at cmm.chem.upenn.edu http://www.cmm.upenn.edu Center for Molecular Modeling -- University of Pennsylvania Department of Chemistry, 231 S.34th Street, Philadelphia, PA 19104-6323 tel: 1-215-898-1582, fax: 1-215-573-6233, office-tel: 1-215-898-5425 ======================================================================= If you make something idiot-proof, the universe creates a better idiot. From eyvaz_isaev at yahoo.com Sun Jan 20 15:35:14 2008 From: eyvaz_isaev at yahoo.com (Eyvaz Isaev) Date: Sun, 20 Jan 2008 06:35:14 -0800 (PST) Subject: [Pw_forum] About spin In-Reply-To: <19310813.277751200742705660.JavaMail.coremail@bj126app81.126.com> Message-ID: <642407.97557.qm@web60324.mail.yahoo.com> Hi, --- oulihui666 wrote: > Dear pwscf users, > I want to ask a question, when I calculate with > pw.x, I use the choice of about spin, that is to > say, nspin=2, so I will specify the > starting_magnetization. I check the email list, and > I find that total_magnetization=(number of unpaired > electrons)/(total numbers of electrons), > starting_magnetization is the initial value of it. > For example, when the system contains Pt, O, H > atoms, I specify starting_magnetization=0.2 for Pt > atom, starting_magnetization=0.5 for O atom, > starting_magnetization=1.0 for H atom. Is it correct > and appropriate about my settings? > Yes, it is. As starting_magnetization is used to break the symmetry, so, I suggest you can leave them equal and around 0.8, but if you like them to be differen, do it. Of course, you can play around this value in order be sure that final magnetization is not changed (i.e. final magnetization should not depend on your initial choice). Bests, Eyvaz. > Thanks in advance > Lihui Ou > > > > > -- > > ====================================== > Lihui Ou > PH.D Candidate in Electrochemistry > > College of Chemistry and Molecular Science > > Wuhan University,430072,Hubei Province,China > E-mail:oulihui666 at 126.com > ======================================> _______________________________________________ > Pw_forum mailing list > Pw_forum at pwscf.org > http://www.democritos.it/mailman/listinfo/pw_forum > ------------------------------------------------------------------- Prof. Eyvaz Isaev, Theoretical Physics Department, Moscow State Institute of Steel & Alloys, Russia, IFM, Linkoping University, Sweden Condensed Matter Theory Group, Uppsala University, Sweden Eyvaz.Isaev at fysik.uu.se, eyvaz_isaev at yahoo.com ____________________________________________________________________________________ Never miss a thing. Make Yahoo your home page. http://www.yahoo.com/r/hs From giannozz at nest.sns.it Sun Jan 20 17:07:51 2008 From: giannozz at nest.sns.it (Paolo Giannozzi) Date: Sun, 20 Jan 2008 17:07:51 +0100 Subject: [Pw_forum] Questions about ionic & cell relaxation. In-Reply-To: References: Message-ID: On Jan 19, 2008, at 19:13 , Hongbin Zhang wrote: > 1) if the symmetry will be kept, then how can I break the symmetry if > necessary? try with a hammer :-) Seriously: move one or a few atoms away from their symmetric positions > 2) I think there is a Keyword "nosym" in the namelist "&SYSTEM", is it > somehow related to structure relaxation? what the "nosym" option (a very misleading name in my opinion) does is explained in detail and exactly in the INPUT_PW file. Do not use it unless you understand it. > 3) In "ATOMIC_POSITIONS" catalog, it's possible to set "if_pos" terms, > which will make forces along some directions to be zero. How should I > combine these options with "nosym" option? these two options do not interact at all > 1) optimization with mirror symmetry along z direction > > 2) optimization without mirror symmetry along z direction > > So if the original supercell have the z-mirror symmetry, how to > break it > if relaxation will keep all symmetry pwscf find (nicola)? move just one atom from (x,y,z) to (x,y,z+dz), with dz > 10^(-5) *lattice parameter Paolo --- Paolo Giannozzi, Dept of Physics, University of Udine via delle Scienze 208, 33100 Udine, Italy Phone +39-0432-558216, fax +39-0432-558222 From jkiitb at gmail.com Mon Jan 21 07:31:44 2008 From: jkiitb at gmail.com (jitendra mishra) Date: Mon, 21 Jan 2008 12:01:44 +0530 Subject: [Pw_forum] Gaussian broadening Message-ID: Dear Sir, I want to know how much the degauss value I should use to plot eliashberg function from lambda.x. I'm not getting the proper idea about that in research papers I've found. Could anyone suggest me a paper on gaussian broadening to use in electron-phonon calculation. -- JITENDRA KUMAR MISHRA Room no:-B-701, H-13, IIT BOMBAY,POWAI, MUMBAI-400076 -------------- next part -------------- An HTML attachment was scrubbed... URL: http://www.democritos.it/pipermail/pw_forum/attachments/20080121/3432cc38/attachment.htm From hashem.yamani at gmail.com Mon Jan 21 14:24:07 2008 From: hashem.yamani at gmail.com (Hashem Al-Yamani) Date: Mon, 21 Jan 2008 13:24:07 +0000 Subject: [Pw_forum] error no. 634 ???? Message-ID: <82a593400801210524s451bfb9bof2b5a3ff20788596@mail.gmail.com> Dear All ; While I were doing a surface relaxation of c-o adsorption the following error appeared after many good iterations : %%%%%%%%%%%%%%%%%%%%%%%%%%%%%%%%%%%%%%%%%%%%%%%%%%%%%%%%%%%%%%%%%%%%%%%%%%%%%% from cdiaghg : error # 634 info =/= 0 %%%%%%%%%%%%%%%%%%%%%%%%%%%%%%%%%%%%%%%%%%%%%%%%%%%%%%%%%%%%%%%%%%%%%%%%%%%%%% stopping ... Any one knows what could be the problem ....... thanks in advance .. Have a nice day . H. Al-Yamani Jordan University -------------- next part -------------- An HTML attachment was scrubbed... URL: http://www.democritos.it/pipermail/pw_forum/attachments/20080121/f4124df4/attachment.htm From paulatto at sissa.it Mon Jan 21 14:32:44 2008 From: paulatto at sissa.it (Lorenzo Paulatto) Date: Mon, 21 Jan 2008 14:32:44 +0100 (CET) Subject: [Pw_forum] error no. 634 ???? In-Reply-To: <82a593400801210524s451bfb9bof2b5a3ff20788596@mail.gmail.com> References: <82a593400801210524s451bfb9bof2b5a3ff20788596@mail.gmail.com> Message-ID: <23632.147.122.5.182.1200922364.squirrel@webmail.sissa.it> On Mon, January 21, 2008 14:24, Hashem Al-Yamani wrote: > While I were doing a surface relaxation of c-o adsorption the following > error appeared after many good iterations : Dear Hashem, please have a look at the troubleshooting page on the QE wiki: References: <82a593400801210524s451bfb9bof2b5a3ff20788596@mail.gmail.com> Message-ID: <47949EFA.8060105@na.infn.it> Hashem Al-Yamani wrote: > Dear All ; > > While I were doing a surface relaxation of c-o adsorption the > following error appeared after many good iterations : > > %%%%%%%%%%%%%%%%%%%%%%%%%%%%%%%%%%%%%%%%%%%%%%%%%%%%%%%%%%%%%%%%%%%%%%%%%%%%%% > from cdiaghg : error # 634 > info =/= 0 > %%%%%%%%%%%%%%%%%%%%%%%%%%%%%%%%%%%%%%%%%%%%%%%%%%%%%%%%%%%%%%%%%%%%%%%%%%%%%% > > stopping ... > > > > > > Any one knows what could be the problem ....... thanks in advance .. > > > > Have a nice day . > > H. Al-Yamani > Jordan University > ------------------------------------------------------------------------ > > _______________________________________________ > Pw_forum mailing list > Pw_forum at pwscf.org > http://www.democritos.it/mailman/listinfo/pw_forum > Hi, you can start looking to previous discussions in the forum, e.g.: http://www.democritos.it/pipermail/pw_forum/2006-July/004574.html http://www.democritos.it/pipermail/pw_forum/2003-June/000379.html http://www.democritos.it/pipermail/pw_forum/2006-July/004576.html http://www.democritos.it/pipermail/pw_forum/2005-April/002389.html ... Some suggestions are given in the User guide as well, see http://www.pwscf.org/guide/3.2.3/users-guide-3.2.3.pdf pag. 55. Giovanni -- Dr. Giovanni Cantele Coherentia CNR-INFM and Dipartimento di Scienze Fisiche Universita' di Napoli "Federico II" Complesso Universitario di Monte S. Angelo - Ed. 6 Via Cintia, I-80126, Napoli, Italy Phone: +39 081 676910 Fax: +39 081 676346 E-mail: Giovanni.Cantele at na.infn.it Web: http://people.na.infn.it/~cantele Research Group: http://www.nanomat.unina.it From costanzo at ms.fci.unibo.it Wed Jan 23 12:00:25 2008 From: costanzo at ms.fci.unibo.it (Francesca Costanzo) Date: Wed, 23 Jan 2008 12:00:25 +0100 (CET) Subject: [Pw_forum] scf convergence calculation In-Reply-To: <4B5F7B57-C793-4C9F-9318-0A73678E3B69@nest.sns.it> References: <5631.147.122.5.182.1200646429.squirrel@webmail.sissa.it> <4B5F7B57-C793-4C9F-9318-0A73678E3B69@nest.sns.it> Message-ID: Dear PWSCF users, I am doing a scf calculation on si(100) slab surface. I have tested two pseudo potentials fo silicon to see which performs better. I am wondering why using the Si.pbe-n-van.UPF and Si.pw91-n-van.UPF pseudoes in the first case I can reach quite easily ( after 400 iterations) the convergence while in the latter case after 1200 iterations I still don't reach the convergence. the mixing beta is 0.5. Is there something wrong? Anyway the input file is below: # self-consistent calculation cat > si-ex01.in << EOF &control calculation='scf' prefix='si', tstress = .true. tprnfor = .true. restart_mode='restart', pseudo_dir ='/sfs/sanfs/home/userincm/incbo407/PWscf/pseudo/', outdir='/scratch/incbo407/au-PWscf/tmp-1/', etot_conv_thr = 1.d-4 / &system ibrav = 8, celldm(1) = 29.02243, celldm(2) =1.0, celldm(3)= 0.89771, nat=56, ntyp=2, ecutwfc=18.0, / &electrons mixing_mode = 'plain' mixing_beta = 0.5 conv_thr = 1.0d-8 / &ions / ATOMIC_SPECIES Si 28.086 Si.pw91-n-van.UPF H 1 H.pw91-van_ak.UPF ATOMIC_POSITIONS {angstrom} Si -.486986953827 -.486915661371 .021837699340 Si 3.254734080912 3.254554580350 -.715466406181 Si .424995921464 5.989287334956 -.032510900988 Si 1.908574447668 7.596205581111 -.896329755432 Si 5.990175519344 .425574367605 -.031655667742 Si 7.597289525085 1.909448358802 -.895741285772 Si 4.863694686691 4.863621862511 -.928089502092 Si 9.019734606435 9.020000340300 -.836996926760 Si 1.332006439043 4.282396930595 -1.483511378893 Si 4.282762677873 1.331640602610 -1.482720710139 Si 1.215421638705 9.726643706866 -1.436019240017 Si 4.069065518611 6.940868463489 -1.558148325068 Si 6.941730748367 4.070143770542 -1.556740812199 Si 9.727400143869 1.215885487447 -1.436139812232 Si 6.926671693616 9.825186228268 -1.480679150046 Si 9.825040961721 6.927019069098 -1.481122837636 Si 2.752199364097 -.023268345504 -2.733402882614 Si -.022185141426 2.752036538539 -2.734578963613 Si 2.755879272416 5.496899501655 -2.970326566691 Si -.004779806850 8.317568614429 -2.969974442472 Si 8.317316174588 -.004017804657 -2.969192076227 Si 5.498360672151 2.755878723458 -2.968474960855 Si 8.346177303503 5.558752009571 -2.782863187750 Si 5.557877655641 8.346312021303 -2.782400281184 Si 1.389966664098 1.388183764950 -4.139518840077 Si 1.366337053897 6.940229376004 -4.258916306178 Si 4.151574127028 4.149476803004 -4.278767316434 Si 4.138929512730 9.694066210006 -4.160002340210 Si 6.940071554385 1.366577579401 -4.258586275398 Si 9.693861222560 4.139131304691 -4.160296524367 Si 6.925553051516 6.926722417990 -4.154730726999 Si 9.700291030289 9.701542440060 -4.273212145658 Si .000000000000 .000000000000 -5.538591342479 Si 2.769296729594 2.769296729594 -5.538591342479 Si .000000000000 5.538591342479 -5.538591342479 Si 2.769296729594 8.307892305490 -5.538591342479 Si 5.538591342479 .000000000000 -5.538591342479 Si 8.307892305490 2.769296729594 -5.538591342479 Si 5.538591342479 5.538591342479 -5.538591342479 Si 8.307892305490 8.307892305490 -5.538591342479 H .847478422628 -.846884685755 -6.504440365581 H -.847316494390 .847694326946 -6.503794769337 H 3.610082118376 1.928241460414 -6.515886469477 H 1.917014435899 3.621363118971 -6.495428477030 H .852174341536 4.685717428620 -6.494724671289 H -.840461532306 6.379649786722 -6.516156349874 H 3.616235391428 7.460033933598 -6.504011732009 H 1.921872283045 9.155369669764 -6.503953522512 H 6.383803828126 -.844617690420 -6.508975414605 H 4.685555500382 .852282293694 -6.494454790892 H 9.156068183733 1.921926259125 -6.503307926268 H 7.460679529841 3.616289367507 -6.504710245978 H 6.385587155455 4.690845156163 -6.504170485184 H 4.690683227924 6.386073998525 -6.503524888940 H 9.160391561857 7.455445966849 -6.494835798512 H 7.462679819843 9.153210626588 -6.508329818361 K_POINTS {automatic} 4 4 1 0 0 0 EOF thank you. Francesca Costanzo -- Francesca Costanzo, Ph.D. Dipartimento di Chimica Fisica ed Inorganica Viale Risorgimento 4 40136 Bologna tel.0039-051-2093710 fax 0039-051-2093690 From ferretti.andrea at unimore.it Wed Jan 23 12:47:44 2008 From: ferretti.andrea at unimore.it (Andrea Ferretti) Date: Wed, 23 Jan 2008 12:47:44 +0100 (CET) Subject: [Pw_forum] scf convergence calculation In-Reply-To: References: <5631.147.122.5.182.1200646429.squirrel@webmail.sissa.it> <4B5F7B57-C793-4C9F-9318-0A73678E3B69@nest.sns.it> Message-ID: Hi Francesca, > / > &system > ibrav = 8, celldm(1) = 29.02243, celldm(2) =1.0, celldm(3)= 0.89771, > nat=56, ntyp=2, ecutwfc=18.0, > / >... > ATOMIC_SPECIES > Si 28.086 Si.pw91-n-van.UPF > H 1 H.pw91-van_ak.UPF since you are using ultrasoft pseudopotentials (USPP), besides ecutwfc, you should also specify ecutrho... as a default ecutrho = 4 * ecutwfc, and this is what it is analytically obtained for norm-conserving pseudopotentials... in the case of USPPs, typical values for ecutrho range from 8 to 12 times ecutwfc (as a rule of thumb to be tested) of course, this can explain convergence problems look also in the forum archive where this topic has been already discussed... cheers andrea -- Andrea Ferretti National Research Center S3, CNR-INFM ( http://s3.infm.it ) Dip. Fisica, Univ. di Modena e Reggio E. (Italy) Tel: +39 059 2055301; Fax: +39 059 374794; Skype: andrea_ferretti URL: http://www.nanoscience.unimo.it Please, if possible, don't send me MS Word or PowerPoint attachments Why? See: http://www.gnu.org/philosophy/no-word-attachments.html From degironc at sissa.it Wed Jan 23 12:50:00 2008 From: degironc at sissa.it (Stefano de Gironcoli) Date: Wed, 23 Jan 2008 12:50:00 +0100 Subject: [Pw_forum] scf convergence calculation In-Reply-To: References: <5631.147.122.5.182.1200646429.squirrel@webmail.sissa.it> <4B5F7B57-C793-4C9F-9318-0A73678E3B69@nest.sns.it> Message-ID: <479729E8.8090302@sissa.it> Dear Francesca, 400 iterations for reaching scf are very many.... there is something wrong in your setup... Are you sure that your coordinates are correct ? By visualizing your input with xcrysden I see that there are gaps in x and y directions (in addition to the probably desited one along z) ... hope it helps stefano Francesca Costanzo wrote: > Dear PWSCF users, > > I am doing a scf calculation on si(100) slab surface. > I have tested two pseudo potentials fo silicon to see > which performs better. > > I am wondering why using > the Si.pbe-n-van.UPF and Si.pw91-n-van.UPF pseudoes > in the first case I can reach quite easily ( after 400 > iterations) the convergence while in the latter case after 1200 iterations > I still don't reach the convergence. > the mixing beta is 0.5. > > Is there something wrong? > > Anyway the input file is below: > > # self-consistent calculation > cat > si-ex01.in << EOF > &control > calculation='scf' > prefix='si', > tstress = .true. > tprnfor = .true. > restart_mode='restart', > pseudo_dir ='/sfs/sanfs/home/userincm/incbo407/PWscf/pseudo/', > outdir='/scratch/incbo407/au-PWscf/tmp-1/', > etot_conv_thr = 1.d-4 > / > &system > ibrav = 8, celldm(1) = 29.02243, celldm(2) =1.0, celldm(3)= 0.89771, > nat=56, ntyp=2, ecutwfc=18.0, > / > &electrons > mixing_mode = 'plain' > mixing_beta = 0.5 > conv_thr = 1.0d-8 > / > &ions > / > ATOMIC_SPECIES > Si 28.086 Si.pw91-n-van.UPF > H 1 H.pw91-van_ak.UPF > ATOMIC_POSITIONS {angstrom} > Si -.486986953827 -.486915661371 .021837699340 > Si 3.254734080912 3.254554580350 -.715466406181 > Si .424995921464 5.989287334956 -.032510900988 > Si 1.908574447668 7.596205581111 -.896329755432 > Si 5.990175519344 .425574367605 -.031655667742 > Si 7.597289525085 1.909448358802 -.895741285772 > Si 4.863694686691 4.863621862511 -.928089502092 > Si 9.019734606435 9.020000340300 -.836996926760 > Si 1.332006439043 4.282396930595 -1.483511378893 > Si 4.282762677873 1.331640602610 -1.482720710139 > Si 1.215421638705 9.726643706866 -1.436019240017 > Si 4.069065518611 6.940868463489 -1.558148325068 > Si 6.941730748367 4.070143770542 -1.556740812199 > Si 9.727400143869 1.215885487447 -1.436139812232 > Si 6.926671693616 9.825186228268 -1.480679150046 > Si 9.825040961721 6.927019069098 -1.481122837636 > Si 2.752199364097 -.023268345504 -2.733402882614 > Si -.022185141426 2.752036538539 -2.734578963613 > Si 2.755879272416 5.496899501655 -2.970326566691 > Si -.004779806850 8.317568614429 -2.969974442472 > Si 8.317316174588 -.004017804657 -2.969192076227 > Si 5.498360672151 2.755878723458 -2.968474960855 > Si 8.346177303503 5.558752009571 -2.782863187750 > Si 5.557877655641 8.346312021303 -2.782400281184 > Si 1.389966664098 1.388183764950 -4.139518840077 > Si 1.366337053897 6.940229376004 -4.258916306178 > Si 4.151574127028 4.149476803004 -4.278767316434 > Si 4.138929512730 9.694066210006 -4.160002340210 > Si 6.940071554385 1.366577579401 -4.258586275398 > Si 9.693861222560 4.139131304691 -4.160296524367 > Si 6.925553051516 6.926722417990 -4.154730726999 > Si 9.700291030289 9.701542440060 -4.273212145658 > Si .000000000000 .000000000000 -5.538591342479 > Si 2.769296729594 2.769296729594 -5.538591342479 > Si .000000000000 5.538591342479 -5.538591342479 > Si 2.769296729594 8.307892305490 -5.538591342479 > Si 5.538591342479 .000000000000 -5.538591342479 > Si 8.307892305490 2.769296729594 -5.538591342479 > Si 5.538591342479 5.538591342479 -5.538591342479 > Si 8.307892305490 8.307892305490 -5.538591342479 > H .847478422628 -.846884685755 -6.504440365581 > H -.847316494390 .847694326946 -6.503794769337 > H 3.610082118376 1.928241460414 -6.515886469477 > H 1.917014435899 3.621363118971 -6.495428477030 > H .852174341536 4.685717428620 -6.494724671289 > H -.840461532306 6.379649786722 -6.516156349874 > H 3.616235391428 7.460033933598 -6.504011732009 > H 1.921872283045 9.155369669764 -6.503953522512 > H 6.383803828126 -.844617690420 -6.508975414605 > H 4.685555500382 .852282293694 -6.494454790892 > H 9.156068183733 1.921926259125 -6.503307926268 > H 7.460679529841 3.616289367507 -6.504710245978 > H 6.385587155455 4.690845156163 -6.504170485184 > H 4.690683227924 6.386073998525 -6.503524888940 > H 9.160391561857 7.455445966849 -6.494835798512 > H 7.462679819843 9.153210626588 -6.508329818361 > K_POINTS {automatic} > 4 4 1 0 0 0 > EOF > > > thank you. > Francesca Costanzo > From costanzo at ms.fci.unibo.it Wed Jan 23 15:13:26 2008 From: costanzo at ms.fci.unibo.it (Francesca Costanzo) Date: Wed, 23 Jan 2008 15:13:26 +0100 (CET) Subject: [Pw_forum] scf convergence calculation In-Reply-To: <479729E8.8090302@sissa.it> References: <5631.147.122.5.182.1200646429.squirrel@webmail.sissa.it> <4B5F7B57-C793-4C9F-9318-0A73678E3B69@nest.sns.it> <479729E8.8090302@sissa.it> Message-ID: Thanx! Francesca On Wed, 23 Jan 2008, Stefano de Gironcoli wrote: > Dear Francesca, > > 400 iterations for reaching scf are very many.... there is something > wrong in your setup... > Are you sure that your coordinates are correct ? > By visualizing your input with xcrysden I see that there are gaps in x > and y directions (in addition to the probably desited one along z) ... > hope it helps > stefano > > > Francesca Costanzo wrote: >> Dear PWSCF users, >> >> I am doing a scf calculation on si(100) slab surface. >> I have tested two pseudo potentials fo silicon to see >> which performs better. >> >> I am wondering why using >> the Si.pbe-n-van.UPF and Si.pw91-n-van.UPF pseudoes >> in the first case I can reach quite easily ( after 400 >> iterations) the convergence while in the latter case after 1200 iterations >> I still don't reach the convergence. >> the mixing beta is 0.5. >> >> Is there something wrong? >> >> Anyway the input file is below: >> >> # self-consistent calculation >> cat > si-ex01.in << EOF >> &control >> calculation='scf' >> prefix='si', >> tstress = .true. >> tprnfor = .true. >> restart_mode='restart', >> pseudo_dir ='/sfs/sanfs/home/userincm/incbo407/PWscf/pseudo/', >> outdir='/scratch/incbo407/au-PWscf/tmp-1/', >> etot_conv_thr = 1.d-4 >> / >> &system >> ibrav = 8, celldm(1) = 29.02243, celldm(2) =1.0, celldm(3)= 0.89771, >> nat=56, ntyp=2, ecutwfc=18.0, >> / >> &electrons >> mixing_mode = 'plain' >> mixing_beta = 0.5 >> conv_thr = 1.0d-8 >> / >> &ions >> / >> ATOMIC_SPECIES >> Si 28.086 Si.pw91-n-van.UPF >> H 1 H.pw91-van_ak.UPF >> ATOMIC_POSITIONS {angstrom} >> Si -.486986953827 -.486915661371 .021837699340 >> Si 3.254734080912 3.254554580350 -.715466406181 >> Si .424995921464 5.989287334956 -.032510900988 >> Si 1.908574447668 7.596205581111 -.896329755432 >> Si 5.990175519344 .425574367605 -.031655667742 >> Si 7.597289525085 1.909448358802 -.895741285772 >> Si 4.863694686691 4.863621862511 -.928089502092 >> Si 9.019734606435 9.020000340300 -.836996926760 >> Si 1.332006439043 4.282396930595 -1.483511378893 >> Si 4.282762677873 1.331640602610 -1.482720710139 >> Si 1.215421638705 9.726643706866 -1.436019240017 >> Si 4.069065518611 6.940868463489 -1.558148325068 >> Si 6.941730748367 4.070143770542 -1.556740812199 >> Si 9.727400143869 1.215885487447 -1.436139812232 >> Si 6.926671693616 9.825186228268 -1.480679150046 >> Si 9.825040961721 6.927019069098 -1.481122837636 >> Si 2.752199364097 -.023268345504 -2.733402882614 >> Si -.022185141426 2.752036538539 -2.734578963613 >> Si 2.755879272416 5.496899501655 -2.970326566691 >> Si -.004779806850 8.317568614429 -2.969974442472 >> Si 8.317316174588 -.004017804657 -2.969192076227 >> Si 5.498360672151 2.755878723458 -2.968474960855 >> Si 8.346177303503 5.558752009571 -2.782863187750 >> Si 5.557877655641 8.346312021303 -2.782400281184 >> Si 1.389966664098 1.388183764950 -4.139518840077 >> Si 1.366337053897 6.940229376004 -4.258916306178 >> Si 4.151574127028 4.149476803004 -4.278767316434 >> Si 4.138929512730 9.694066210006 -4.160002340210 >> Si 6.940071554385 1.366577579401 -4.258586275398 >> Si 9.693861222560 4.139131304691 -4.160296524367 >> Si 6.925553051516 6.926722417990 -4.154730726999 >> Si 9.700291030289 9.701542440060 -4.273212145658 >> Si .000000000000 .000000000000 -5.538591342479 >> Si 2.769296729594 2.769296729594 -5.538591342479 >> Si .000000000000 5.538591342479 -5.538591342479 >> Si 2.769296729594 8.307892305490 -5.538591342479 >> Si 5.538591342479 .000000000000 -5.538591342479 >> Si 8.307892305490 2.769296729594 -5.538591342479 >> Si 5.538591342479 5.538591342479 -5.538591342479 >> Si 8.307892305490 8.307892305490 -5.538591342479 >> H .847478422628 -.846884685755 -6.504440365581 >> H -.847316494390 .847694326946 -6.503794769337 >> H 3.610082118376 1.928241460414 -6.515886469477 >> H 1.917014435899 3.621363118971 -6.495428477030 >> H .852174341536 4.685717428620 -6.494724671289 >> H -.840461532306 6.379649786722 -6.516156349874 >> H 3.616235391428 7.460033933598 -6.504011732009 >> H 1.921872283045 9.155369669764 -6.503953522512 >> H 6.383803828126 -.844617690420 -6.508975414605 >> H 4.685555500382 .852282293694 -6.494454790892 >> H 9.156068183733 1.921926259125 -6.503307926268 >> H 7.460679529841 3.616289367507 -6.504710245978 >> H 6.385587155455 4.690845156163 -6.504170485184 >> H 4.690683227924 6.386073998525 -6.503524888940 >> H 9.160391561857 7.455445966849 -6.494835798512 >> H 7.462679819843 9.153210626588 -6.508329818361 >> K_POINTS {automatic} >> 4 4 1 0 0 0 >> EOF >> >> >> thank you. >> Francesca Costanzo >> > > _______________________________________________ > Pw_forum mailing list > Pw_forum at pwscf.org > http://www.democritos.it/mailman/listinfo/pw_forum > -- Francesca Costanzo, Ph.D. Dipartimento di Chimica Fisica ed Inorganica Viale Risorgimento 4 40136 Bologna tel.0039-051-2093710 fax 0039-051-2093690 From bnrj.rudra at yahoo.com Wed Jan 23 16:07:38 2008 From: bnrj.rudra at yahoo.com (Rudra Banerjee) Date: Wed, 23 Jan 2008 20:37:38 +0530 (IST) Subject: [Pw_forum] creating potential Message-ID: <678564.44805.qm@web94106.mail.in2.yahoo.com> dear friends, for my canculation, i want to put my systems insdie a box potential. any body can give me any tips on how to do that? regards -- Rudra Have a Nice Time http://www.bose.res.in/~rudra Please, if possible, don't send me MS Word or PowerPoint attachments Why?See: http://www.gnu.org/philosophy/no-word-attachments.html Why delete messages? Unlimited storage is just a click away. Go to http://help.yahoo.com/l/in/yahoo/mail/yahoomail/tools/tools-08.html -------------- next part -------------- An HTML attachment was scrubbed... URL: http://www.democritos.it/pipermail/pw_forum/attachments/20080123/96f8d5d1/attachment.htm From h.zhang at ifw-dresden.de Wed Jan 23 19:19:09 2008 From: h.zhang at ifw-dresden.de (Hongbin Zhang) Date: Wed, 23 Jan 2008 19:19:09 +0100 (CET) Subject: [Pw_forum] nonmagnetic solution of bct Fe In-Reply-To: <478F58FF.2040200@mit.edu> Message-ID: Dear all users, I am trying to do some test calculation of Fe, actually, bct Fe, with pwscf 3.2.3. However, with the input file attached, I got a nonmagnetic state --- the total magnetic moments is ZERO. However, the input file works for bcc Fe and FePt, at least I could get Ferromagnetic states for these two. Any comments? or suggestions? Thanks a lot. PS: I also tried without starting_magnetization(2), which is correct according the DOC/INPUT_PW, but nonmagnetic still. Best wishes, Hongbin Zhang -------------- next part -------------- &control calculation='scf' restart_mode='from_scratch', prefix='fe', tprnfor = .true tstress = .true pseudo_dir = './', outdir='./' / &system ibrav=7, celldm(1) =5.3329286, celldm(3)=0.95178592, nat=2, ntyp=1, nspin = 2, starting_magnetization(1)=0.9, starting_magnetization(2)=0.9, ecutwfc = 30.0, ecutrho = 288.0, occupations='smearing', smearing='methfessel-paxton', degauss=0.02 / &electrons conv_thr = 1.0e-8 mixing_beta = 0.7 / ATOMIC_SPECIES Fe 55.847 Fe.pz-sp-van_ak.UPF ATOMIC_POSITIONS Fe 0.0 0.0 0.0 Fe 0.5 0.5 0.5 K_POINTS {automatic} 12 12 12 0 0 0 From eyvaz_isaev at yahoo.com Wed Jan 23 19:49:59 2008 From: eyvaz_isaev at yahoo.com (Eyvaz Isaev) Date: Wed, 23 Jan 2008 10:49:59 -0800 (PST) Subject: [Pw_forum] nonmagnetic solution of bct Fe In-Reply-To: Message-ID: <62874.60734.qm@web60316.mail.yahoo.com> Hi, Your input file is controversional. First you specify ibrav=7 (bct), but then you require that there are two atoms in the unit cell. > ATOMIC_POSITIONS > Fe 0.0 0.0 0.0 > Fe 0.5 0.5 0.5 That is not correct, with ibrav=7 you should specify only one Fe atom, (0,0,0). The atomic positions you have chosen is for simple tetragonal system, ibrav=6. Also keep in mind "alat" is default for atomic positions. Please also follow the Netiquette accepted in this forum providing your affiliation. Bests, Eyvaz. --- Hongbin Zhang wrote: > > Dear all users, > > I am trying to do some test calculation of Fe, > actually, bct Fe, with > pwscf 3.2.3. However, with the input file attached, > I got a nonmagnetic > state --- the total magnetic moments is ZERO. > > However, the input file works for bcc Fe and FePt, > at least I could get > Ferromagnetic states for these two. > > Any comments? or suggestions? Thanks a lot. > > PS: I also tried without starting_magnetization(2), > which is correct > according the DOC/INPUT_PW, but nonmagnetic still. > > Best wishes, > > Hongbin Zhang > > > > > &control > calculation='scf' > restart_mode='from_scratch', > prefix='fe', > tprnfor = .true > tstress = .true > pseudo_dir = './', > outdir='./' > / > &system > ibrav=7, celldm(1) =5.3329286, > celldm(3)=0.95178592, nat=2, ntyp=1, > nspin = 2, starting_magnetization(1)=0.9, > starting_magnetization(2)=0.9, > ecutwfc = 30.0, ecutrho = 288.0, > occupations='smearing', > smearing='methfessel-paxton', degauss=0.02 > / > &electrons > conv_thr = 1.0e-8 > mixing_beta = 0.7 > / > ATOMIC_SPECIES > Fe 55.847 Fe.pz-sp-van_ak.UPF > ATOMIC_POSITIONS > Fe 0.0 0.0 0.0 > Fe 0.5 0.5 0.5 > K_POINTS {automatic} > 12 12 12 0 0 0 > > _______________________________________________ > Pw_forum mailing list > Pw_forum at pwscf.org > http://www.democritos.it/mailman/listinfo/pw_forum > ------------------------------------------------------------------- Prof. Eyvaz Isaev, Theoretical Physics Department, Moscow State Institute of Steel & Alloys, Russia, IFM, Linkoping University, Sweden Condensed Matter Theory Group, Uppsala University, Sweden Eyvaz.Isaev at fysik.uu.se, eyvaz_isaev at yahoo.com ____________________________________________________________________________________ Be a better friend, newshound, and know-it-all with Yahoo! Mobile. Try it now. http://mobile.yahoo.com/;_ylt=Ahu06i62sR8HDtDypao8Wcj9tAcJ From min0220 at postech.ac.kr Thu Jan 24 06:13:34 2008 From: min0220 at postech.ac.kr (Min Seung Kyu) Date: Thu, 24 Jan 2008 14:13:34 +0900 (KST) Subject: [Pw_forum] How to run ground state molecular dynamics Message-ID: Dear users, Could you give me a sample input for ground state molecular dynamics in NVT ensemble? I saw si.md2.in in example04 folder, but there is no information about temperature. Thank you in advance. Sincerely, Min. -------------------------------Sig.--------------------------- Center for Superfunctional Materials, Department of Chemistry, Pohang University of Science and Technology (POSTECH) Mr. Seung Kyu, Min Ph.D. Candidate E-Mail: min0220 at postech.ac.kr Tel. : 82-54-279-5858 ---------------------------------------------- From hqzhou at nju.edu.cn Thu Jan 24 10:26:21 2008 From: hqzhou at nju.edu.cn (hqzhou) Date: Thu, 24 Jan 2008 17:26:21 +0800 (CST) Subject: [Pw_forum] nonmagnetic solution of bct Fe Message-ID: Your system should have only one atom and then you can get a slightly smaller total and absolute magnetization than Fe-bcc. Dr. Huiqun Zhou @Nanjing University, China > -----Original Message----- > ???: Hongbin Zhang > ??: Wed, 23 Jan 2008 19:19:09 +0100 > ???: > ??: [Pw_forum] nonmagnetic solution of bct Fe > > Dear all users, > > I am trying to do some test calculation of Fe, actually, bct Fe, with > pwscf 3.2.3. However, with the input file attached, I got a nonmagnetic > state --- the total magnetic moments is ZERO. > > However, the input file works for bcc Fe and FePt, at least I could get > Ferromagnetic states for these two. > > Any comments? or suggestions? Thanks a lot. > > PS: I also tried without starting_magnetization(2), which is correct > according the DOC/INPUT_PW, but nonmagnetic still. > > Best wishes, > > Hongbin Zhang > > > > From riccardo at fcs.it Thu Jan 24 12:30:57 2008 From: riccardo at fcs.it (Riccardo Sabatini) Date: Thu, 24 Jan 2008 12:30:57 +0100 Subject: [Pw_forum] Cohesive energy Message-ID: <479876F1.8080705@fcs.it> Hello everyone, i have a couple of questions about cohesive energy I'd like to post here at the list. I want to calculate the cohesive energy for the Si in diamond structure. For what i understood the CE (cohesive energy) is the difference between the energy at atoms free and the energy of atoms bounded in the structure you want to study. My first question is about free energy for reference. To evaluate the Si energy alone i made a easy run with one atom in a supercell, big enough to delete all the other atom's effects. The result i had is around -7.47 Ry. My first question is about this result. I'm following a tutorial presentation (http://mpdc.mae.cornell.edu/Courses/MAE715/Symposium/Veera-Presentation.ppt) and there is listed a value of -7.52 Ry for the Si atom free. I tried with different pseudo potentials and with bigger and bigger cells but i can't reproduce that result. I think this difference could arise from a different pseudopotential used by the author and here comes my question, is there a reference database for pwscf calculations with the pseudopotentials used by default ? I tried to search but the NIST database i think uses all-electron's calculations (Si atom results to have -288 hartree) and so is not very useful for a fast check... Anyway, i did the run for the Si in diamond structure. I tried two ways, one with a fcc structure with 2 atoms in the cell and another with a sc cell with 8 atoms. The results seem correct but i don't understand what result should i take. In the tutorial (the one cited above) there is a value called energy/atom and energy/cell. In the first case i suppose one has to divide the total energy by the numbers of atoms in the cell (2 in the first case and 8 in the second) but i can't figure out what is energy/cell (i tried to divide by the volume but the results are order of magnitude different). Is it correct to define a energy for each atom in a cell in this way ? (If i remember correctly the difference in energy for the sc simulation and for the fcc arise from the difference in volume, one is 4 times than the other not from the numbers of atoms). If yes, i tried to compare the energy/atom for the fcc diamond simulation and for the sc diamond simulation and the results are a little different, can this be caused by the number of plane waves used ? Many thanks for the help and for this mailing list, best regards Riccardo From paulatto at sissa.it Thu Jan 24 14:22:45 2008 From: paulatto at sissa.it (Lorenzo Paulatto) Date: Thu, 24 Jan 2008 14:22:45 +0100 (CET) Subject: [Pw_forum] Cohesive energy In-Reply-To: <479876F1.8080705@fcs.it> References: <479876F1.8080705@fcs.it> Message-ID: <6226.147.122.5.182.1201180965.squirrel@webmail.sissa.it> On Thu, January 24, 2008 12:30, Riccardo Sabatini wrote: > The result i had is around -7.47 Ry. > and there is listed a value of -7.52 Ry for the Si atom free. What you have to do is break the symmetry of the isolated atom. Silicon atomic configuration is [Ne] 3s2 3p2, nobody knows what it would if it were alone in the universe, but its ground state configuration fills the 3s orbital completely and then, following Hund's rule, put 1 electron each in two p orbitals. Example: 3s 2 3p-x 1 3p-y 1 When you do the pw calculation for the isolated atom symmetry is actively enforced so the occupations will look more like this: 3s 2 3p-x 2/3 3p-y 2/3 3p-z 2/3 In order to break the symmetry you have first to relax its enforcement by specifying nosym=.true. and then you have to break it actively in the initial position by using a non cubic cell, some random starting wfcs or some other similar trick. You can also specify band occupations by hand, e.g. (if you use nbnd=4): occupations = 'from_input' [...] OCCUPATIONS 1. 1. 1. 0. this will suffice in breaking the symmetry. > In the first case i > suppose one has to divide the total energy by the numbers of atoms in > the cell (2 in the first case and 8 in the second) true > but i can't figure out what is energy/cell it's what you get printed on the screen after "Total energy:", in periodic boundary condition you can't compute "Total" energy > Is it correct to define a energy for each atom in a cell in this way > ? (If i remember correctly the difference in energy for the sc > simulation and for the fcc arise from the difference in volume, one is 4 > times than the other not from the numbers of atoms). The sc cell *contains* 4 fcc cells, has 4 times its volume and 4 times its atoms. Its energy per cell is 4 times larger, so the ratio energy/volume must be the same. Also keep in mind that volume for an fcc cell is (alat**3)/4. Please have a look at crystal structure in you favourite solid state book! > caused by the number of plane waves used ? Without seeing you input files I can't know, there may be many other reasons, most likely numbers of k-points (you need 4 times as many k-points in fcc than in sc). bye, LP P.S. please provide your affiliation. -- Lorenzo Paulatto SISSA & DEMOCRITOS (Trieste) +39 040 3787 511 http://people.sissa.it/~paulatto/ ---------------------------------------------------------------- SISSA Webmail https://webmail.sissa.it/ Powered by SquirrelMail http://www.squirrelmail.org/ From giannozz at nest.sns.it Thu Jan 24 15:52:58 2008 From: giannozz at nest.sns.it (Paolo Giannozzi) Date: Thu, 24 Jan 2008 15:52:58 +0100 Subject: [Pw_forum] Cohesive energy In-Reply-To: <479876F1.8080705@fcs.it> References: <479876F1.8080705@fcs.it> Message-ID: <89DD37E0-85BE-49BF-8CB2-ADBF24FA0CA3@nest.sns.it> On Jan 24, 2008, at 12:30 , Riccardo Sabatini wrote: > The result i had is around -7.47 Ry. My first question is about > this result. I'm following a tutorial presentation (...) and there > is listed a value of -7.52 Ry for the Si atom free. absolute values of the energy depend on the specific pseudopotential. Energy differences shouldn't. Paolo --- Paolo Giannozzi, Dept of Physics, University of Udine via delle Scienze 208, 33100 Udine, Italy Phone +39-0432-558216, fax +39-0432-558222 From baroni at sissa.it Thu Jan 24 16:34:23 2008 From: baroni at sissa.it (Stefano Baroni) Date: Thu, 24 Jan 2008 16:34:23 +0100 Subject: [Pw_forum] Cohesive energy In-Reply-To: <479876F1.8080705@fcs.it> References: <479876F1.8080705@fcs.it> Message-ID: <45CBB8A4-A4F4-4810-8E23-3630174097A4@sissa.it> Hi Riccardo: On Jan 24, 2008, at 12:30 PM, Riccardo Sabatini wrote: > diamond structure. For what i understood the CE (cohesive energy) > is the > difference between the energy at atoms free and the energy of atoms > bounded in the structure you want to study. > > My first question is about free energy for reference. To evaluate > the Si energy alone i made a easy run with one atom in a supercell, > big > enough to delete all the other atom's effects. The result i had is > around -7.47 Ry. My first question is about this result. I'm > following a > tutorial presentation > (http://mpdc.mae.cornell.edu/Courses/MAE715/Symposium/Veera- > Presentation.ppt) > and there is listed a value of -7.52 Ry for the Si atom free. I tried > with different pseudo potentials and with bigger and bigger cells > but i > can't reproduce that result. I think this difference could arise > from a > different pseudopotential used by the author and here comes my > question, is there a reference database for pwscf calculations with > the > pseudopotentials used by default ? I tried to search but the NIST > database i think uses all-electron's calculations (Si atom results to > have -288 hartree) and so is not very useful for a fast check... by construction, different pseudopotentials should give equal one- electron levels, but the absolute value of the total energy does depend on the specific pseudopotential. it is only energy differences that are (almost) independent on the specific pseudopotential, as long the PP has been constructed correctly ... > Anyway, i did the run for the Si in diamond structure. I tried two > ways, one with a fcc structure with 2 atoms in the cell and another > with > a sc cell with 8 atoms. The results seem correct but i don't > understand > what result should i take. either one. the cohesive energy is the total energy of the crystal minus the total energy of an equal number of isolated atoms. of course, for an infinite system this difference is infinite, and you have to decide the refernce with respect to which you want to normalize. if you divide by the total number of atoms, you get a "cohesive anergy per atom" (which I think is the usual definition). for diamond, this is one half the "cohesive energy per cell" (because the unit cell has two atoms). > In the tutorial (the one cited above) there > is a value called energy/atom and energy/cell. In the first case i > suppose one has to divide the total energy by the numbers of atoms in > the cell (2 in the first case and 8 in the second) but i can't figure > out what is energy/cell (i tried to divide by the volume but the > results > are order of magnitude different). total energy per cell = the figure spit out by the code total energy per atom = above / (number of atoms in the unit cell) beware, though, that when you perform a calculation for non elementary cells (as the simple cubic cell with 8 atoms that you were mentioning), the result may or may not be directly comparable with the result you get with an elementary cell, according to how careful you have been in the choice of k points for Brillouin-zone sampling. An example may explain this. suppose you perform a calculation with the 8-atom cell and using only the gamma point (k=000) to sample the Brillouin zone. This point would correspond to k=(000) for the elementary cell, but you will easily convince yourself that k=(100) is a reciprocal-lattice vector for the 8-atom cell, while it is not for the 8-atom cell. in conclusion, sampling the 8-atom cell with k=(000) should give an energy which is exactly (within numerical noise) four times as large as the energy of the 2-atom cell, but only if the latter is sampled with the (000) AND the (100) points. > Is it correct to define a energy for each atom in a cell in > this way > ? (If i remember correctly the difference in energy for the sc > simulation and for the fcc arise from the difference in volume, one > is 4 > times than the other not from the numbers of atoms). If yes, i > tried to > compare the energy/atom for the fcc diamond simulation and for the sc > diamond simulation and the results are a little different, can this be > caused by the number of plane waves used ? I am not sure that what I said would sounds clear to you, unless you are already a bit familiar with electronic-structure techniques for infinite systems. if you are not, please take a glance, for instance, at the excellent textbook by R. Martin http://www.cambridge.org/uk/catalogue/catalogue.asp?isbn=0521782856 there used to be a few chapters available for free online, but I cannot find them anymore Stefano B. oh, yes, I was forgetting: please, do not forget to complete your signature with your current affiliation - thanks --- Stefano Baroni - SISSA & DEMOCRITOS National Simulation Center - Trieste [+39] 040 3787 406 (tel) -528 (fax) / stefanobaroni (skype) Please, if possible, don't send me MS Word or PowerPoint attachments Why? See: http://www.gnu.org/philosophy/no-word-attachments.html -------------- next part -------------- An HTML attachment was scrubbed... URL: http://www.democritos.it/pipermail/pw_forum/attachments/20080124/14a22c4b/attachment-0001.htm From eyvaz_isaev at yahoo.com Thu Jan 24 18:25:04 2008 From: eyvaz_isaev at yahoo.com (Eyvaz Isaev) Date: Thu, 24 Jan 2008 09:25:04 -0800 (PST) Subject: [Pw_forum] Cohesive energy In-Reply-To: <45CBB8A4-A4F4-4810-8E23-3630174097A4@sissa.it> Message-ID: <379141.26671.qm@web60312.mail.yahoo.com> Hi Stefano, > I am not sure that what I said would sounds clear > to you, unless you are already a bit familiar with >electronic-structure techniques for infinite >systems. if you are not, please take a glance, for >instance, at the excellent textbook by R. Martin > http://www.cambridge.org/uk/catalogue/catalogue.asp?isbn=0521782856 > > there used to be a few chapters available for free > online, but I cannot find them anymore > http://books.google.se/books?id=dmRTFLpSGNsC&dq=electronic+structure+basic+theory+and+practical+methods&pg=PP1&ots=rsvpVo4IbE&sig=EdM2gjG4Brz59heDqvWTUO4mVFo&hl=sv&prev=http://www.google.se/search?hl=sv&q=Electronic+Structure%0D%0ABasic+Theory+and+Practical+Methods&btnG=Google-s%C3%B6kning&sa=X&oi=print&ct=title&cad=one-book-with-thumbnail#PPA228,M1 Is this that one you are looking for? Bests, Eyvaz. ------------------------------------------------------------------- Prof. Eyvaz Isaev, Theoretical Physics Department, Moscow State Institute of Steel & Alloys, Russia, IFM, Linkoping University, Sweden Condensed Matter Theory Group, Uppsala University, Sweden Eyvaz.Isaev at fysik.uu.se, eyvaz_isaev at yahoo.com ____________________________________________________________________________________ Looking for last minute shopping deals? Find them fast with Yahoo! Search. http://tools.search.yahoo.com/newsearch/category.php?category=shopping From calderin at physics.queensu.ca Fri Jan 25 07:57:58 2008 From: calderin at physics.queensu.ca (Lazaro Calderin) Date: Fri, 25 Jan 2008 07:57:58 +0100 Subject: [Pw_forum] ld1.x problem Message-ID: <1201244288.19052.17.camel@wolfe> Hello ld1.x runs fine and yields the expected results in "generation and testing" mode with the input file &input title='Bi', zed=83.0, rel=1, iswitch=3, rlderiv=3.50, eminld=-4.0, emaxld=4.0, deld=0.02, nld=2, config='[Xe] 4f14 5d10 6s2 6p3', dft='LDA', xmin=-7.0, dx=0.01250, rmax=155.0 / &inputp pseudotype=2, tm=.true., file_pseudopw='Bi-lda-cc-tm.UPF', nlcc=.true., rcloc=1.80 / 2 6S 1 0 2.00 0.00 1.8564 1.8564 6P 2 1 3.00 0.00 2.4441 2.4441 However, in testing mode only, the input file &input title='Bi', zed=83.0, rel=1, iswitch=2, config='[Xe] 4f14 5d10 6s2 6p3' dft='LDA', / &test nconf=1 file_pseudo='Bi-lda-cc-tm.UPF', configts(1)= '4f14 5d10 6s2 6p3' / generates the errors: from compute phi : info # -1 negative determinant 3 ld= -0.022630 f2ae 1.185728 faenor 0.377898 Solution not found in ascheqps for n,l= 4 3 Solution not found in ascheqps for n,l= 3 2 . . . How come? What am I missing? Is this a bug? The electronic configuration is the same in both input files. Lazaro -- ============================================================= Lazaro Calderin, Ph.D. Dpt. of Theoretical, Atomic and Optical Physics Univ. of Valladolid, 47005 Valladolid. Spain Phone: +34 983 423891 Fax: +34 983 423013 E-mails: calderin at fta.uva.es or calderin at physics.queensu.ca From calderin at physics.queensu.ca Fri Jan 25 06:44:28 2008 From: calderin at physics.queensu.ca (Lazaro Calderin) Date: Fri, 25 Jan 2008 06:44:28 +0100 Subject: [Pw_forum] ld1.x problem Message-ID: <1201239878.26001.57.camel@wolfe> Hello everyone, I got the following problem with ld1.x (both 3.2 and 3.2.2). ld1.x runs without problem with this input file for the generation of a psp: &input title='Bi', zed=83.0, rel=1, iswitch=3, rlderiv=3.50, eminld=-4.0, emaxld=4.0, deld=0.02, nld=2, config='[Xe] 4f14 5d10 6s2 6p3 ', dft='LDA', xmin=-7.0, dx=0.01250, rmax=155.0 / &inputp pseudotype=2, tm=.true., file_pseudopw='Bi-lda-cc-tm.UPF', nlcc=.true., rcloc=1.80 / 2 6S 1 0 2.00 0.00 1.8564 1.8564 6P 2 1 3.00 0.00 2.4441 2.4441 However, for the test input file: &input title='Bi', zed=83.0, rel=1, iswitch=2, config='[Xe] 4f14 5d10 6s2 6p3 ' dft='LDA', / &test nconf=1 file_pseudo='Bi-lda-cc-tm.UPF', configts(1)= '4f14 5d10 6s2 6p3 ' / ld1.x fails giving these error messages from compute phi : info # -1 negative determinant 3 ld= -0.022618 f2ae 1.185735 faenor 0.377893 Solution not found in ascheqps for n,l= 1 0 Solution not found in ascheqps for n,l= 2 1 Solution not found in ascheqps for n,l= 4 3 Solution not found in ascheqps for n,l= 3 2 . . . Why the test of the speudopotential for the ground state performed by ld1.x for the first file runs without problems and in testing mode the same ground state configuration yields such errors? A bug? Thanks, L. -- ============================================================= Lazaro Calderin, Ph.D. Dpt. of Theoretical, Atomic and Optical Physics Univ. of Valladolid, 47005 Valladolid. Spain Phone: +34 983 423891 Fax: +34 983 423013 E-mails: calderin at fta.uva.es or calderin at physics.queensu.ca From paulatto at sissa.it Fri Jan 25 09:37:59 2008 From: paulatto at sissa.it (Lorenzo Paulatto) Date: Fri, 25 Jan 2008 09:37:59 +0100 (CET) Subject: [Pw_forum] ld1.x problem In-Reply-To: <1201239878.26001.57.camel@wolfe> References: <1201239878.26001.57.camel@wolfe> Message-ID: <21527.82.49.215.221.1201250279.squirrel@webmail.sissa.it> On Fri, January 25, 2008 06:44, Lazaro Calderin wrote: > configts(1)= '4f14 5d10 6s2 6p3 ' You should only put the valence states in test configuration, e.g. configts(1) = '6s2 6p3' Goodbye -- Lorenzo Paulatto SISSA & DEMOCRITOS (Trieste) +39 040 3787 511 http://people.sissa.it/~paulatto/ ---------------------------------------------------------------- SISSA Webmail https://webmail.sissa.it/ Powered by SquirrelMail http://www.squirrelmail.org/ From paulatto at sissa.it Fri Jan 25 09:46:50 2008 From: paulatto at sissa.it (Lorenzo Paulatto) Date: Fri, 25 Jan 2008 09:46:50 +0100 (CET) Subject: [Pw_forum] ld1.x problem In-Reply-To: <1201239878.26001.57.camel@wolfe> References: <1201239878.26001.57.camel@wolfe> Message-ID: <21541.82.49.215.221.1201250810.squirrel@webmail.sissa.it> On Fri, January 25, 2008 06:44, Lazaro Calderin wrote: > [cut] Oh, and you have a ghost in the s component at ~ -4 rydberg and one in the p channel at ~ 0.8 Ry. It means that your rcloc is probably far too small. Even if the test works in ld1 this pseudo will never be usable in pw -- Lorenzo Paulatto SISSA & DEMOCRITOS (Trieste) +39 040 3787 511 http://people.sissa.it/~paulatto/ ---------------------------------------------------------------- SISSA Webmail https://webmail.sissa.it/ Powered by SquirrelMail http://www.squirrelmail.org/ From dalcorso at sissa.it Fri Jan 25 09:46:47 2008 From: dalcorso at sissa.it (Dal Corso Andrea) Date: Fri, 25 Jan 2008 09:46:47 +0100 Subject: [Pw_forum] ld1.x problem In-Reply-To: <1201239878.26001.57.camel@wolfe> References: <1201239878.26001.57.camel@wolfe> Message-ID: <1201250807.3363.3.camel@dhpc-5-48.sissa.it> On Fri, 2008-01-25 at 06:44 +0100, Lazaro Calderin wrote: > Hello everyone, > > I got the following problem with ld1.x (both 3.2 and 3.2.2). > > ld1.x runs without problem with this input file for the generation of a > psp: > > &input > title='Bi', > zed=83.0, > rel=1, > iswitch=3, > rlderiv=3.50, > eminld=-4.0, > emaxld=4.0, > deld=0.02, > nld=2, > config='[Xe] 4f14 5d10 6s2 6p3 ', > dft='LDA', > xmin=-7.0, > dx=0.01250, > rmax=155.0 > / > &inputp > pseudotype=2, > tm=.true., > file_pseudopw='Bi-lda-cc-tm.UPF', > nlcc=.true., > rcloc=1.80 > / > 2 > 6S 1 0 2.00 0.00 1.8564 1.8564 > 6P 2 1 3.00 0.00 2.4441 2.4441 > > However, for the test input file: > > &input > title='Bi', > zed=83.0, > rel=1, > iswitch=2, > config='[Xe] 4f14 5d10 6s2 6p3 ' > dft='LDA', > / > &test > nconf=1 > file_pseudo='Bi-lda-cc-tm.UPF', > configts(1)= '4f14 5d10 6s2 6p3 ' > / > The test configuration should be 6s2 6p3 without the f and d electrons. You do not have projectors for these channels. Andrea > ld1.x fails giving these error messages > > from compute phi : info # -1 > negative determinant > > 3 ld= -0.022618 f2ae 1.185735 faenor 0.377893 > Solution not found in ascheqps for n,l= 1 0 > Solution not found in ascheqps for n,l= 2 1 > Solution not found in ascheqps for n,l= 4 3 > Solution not found in ascheqps for n,l= 3 2 > . > . > . > > Why the test of the speudopotential for the ground state performed by > ld1.x for the first file runs without problems and in testing mode the > same ground state configuration yields such errors? A bug? > > Thanks, L. > > > -- Andrea Dal Corso Tel. 0039-040-3787428 SISSA, Via Beirut 2/4 Fax. 0039-040-3787528 34014 Trieste (Italy) e-mail: dalcorso at sissa.it From baroni at sissa.it Fri Jan 25 12:39:47 2008 From: baroni at sissa.it (Stefano Baroni) Date: Fri, 25 Jan 2008 12:39:47 +0100 Subject: [Pw_forum] Cohesive energy In-Reply-To: <379141.26671.qm@web60312.mail.yahoo.com> References: <379141.26671.qm@web60312.mail.yahoo.com> Message-ID: Eyvaz: great! many thanks indeed. Everybody else: please, take note! Cheers - Stefano On Jan 24, 2008, at 6:25 PM, Eyvaz Isaev wrote: > Hi Stefano, > >> I am not sure that what I said would sounds clear >> to you, unless you are already a bit familiar with >> electronic-structure techniques for infinite >> systems. if you are not, please take a glance, for >> instance, at the excellent textbook by R. Martin >> > http://www.cambridge.org/uk/catalogue/catalogue.asp?isbn=0521782856 >> >> there used to be a few chapters available for free >> online, but I cannot find them anymore >> > > http://books.google.se/books?id=dmRTFLpSGNsC&dq=electronic+structure > +basic+theory+and+practical > +methods&pg=PP1&ots=rsvpVo4IbE&sig=EdM2gjG4Brz59heDqvWTUO4mVFo&hl=sv&p > rev=http://www.google.se/search?hl=sv&q=Electronic+Structure%0D% > 0ABasic+Theory+and+Practical+Methods&btnG=Google-s%C3% > B6kning&sa=X&oi=print&ct=title&cad=one-book-with-thumbnail#PPA228,M1 > > Is this that one you are looking for? > > Bests, > Eyvaz. > > ------------------------------------------------------------------- > Prof. Eyvaz Isaev, > Theoretical Physics Department, Moscow State Institute of Steel & > Alloys, Russia, > IFM, Linkoping University, Sweden > Condensed Matter Theory Group, Uppsala University, Sweden > Eyvaz.Isaev at fysik.uu.se, eyvaz_isaev at yahoo.com > > > > ______________________________________________________________________ > ______________ > Looking for last minute shopping deals? > Find them fast with Yahoo! Search. http://tools.search.yahoo.com/ > newsearch/category.php?category=shopping > _______________________________________________ > Pw_forum mailing list > Pw_forum at pwscf.org > http://www.democritos.it/mailman/listinfo/pw_forum --- Stefano Baroni - SISSA & DEMOCRITOS National Simulation Center - Trieste [+39] 040 3787 406 (tel) -528 (fax) / stefanobaroni (skype) Please, if possible, don't send me MS Word or PowerPoint attachments Why? See: http://www.gnu.org/philosophy/no-word-attachments.html -------------- next part -------------- An HTML attachment was scrubbed... URL: http://www.democritos.it/pipermail/pw_forum/attachments/20080125/bcc16da6/attachment-0001.htm From lanhaiping at gmail.com Fri Jan 25 13:00:05 2008 From: lanhaiping at gmail.com (lan haiping) Date: Fri, 25 Jan 2008 13:00:05 +0100 Subject: [Pw_forum] Cohesive energy In-Reply-To: References: <379141.26671.qm@web60312.mail.yahoo.com> Message-ID: In fact, via http://books.google.com/, we can find many books, though there are limited chapters supplied. Regards, H.P On Jan 25, 2008 12:39 PM, Stefano Baroni wrote: > Eyvaz: great! many thanks indeed.Everybody else: please, take note! > Cheers - Stefano > > On Jan 24, 2008, at 6:25 PM, Eyvaz Isaev wrote: > > Hi Stefano, > > I am not sure that what I said would sounds clear > to you, unless you are already a bit familiar with > electronic-structure techniques for infinite > systems. if you are not, please take a glance, for > instance, at the excellent textbook by R. Martin > > http://www.cambridge.org/uk/catalogue/catalogue.asp?isbn=0521782856 > > > there used to be a few chapters available for free > online, but I cannot find them anymore > > > > http://books.google.se/books?id=dmRTFLpSGNsC&dq=electronic+structure+basic+theory+and+practical+methods&pg=PP1&ots=rsvpVo4IbE&sig=EdM2gjG4Brz59heDqvWTUO4mVFo&hl=sv&prev=http://www.google.se/search?hl=sv&q=Electronic+Structure%0D%0ABasic+Theory+and+Practical+Methods&btnG=Google-s%C3%B6kning&sa=X&oi=print&ct=title&cad=one-book-with-thumbnail#PPA228,M1 > > Is this that one you are looking for? > > Bests, > Eyvaz. > > ------------------------------------------------------------------- > Prof. Eyvaz Isaev, > Theoretical Physics Department, Moscow State Institute of Steel & Alloys, > Russia, > IFM, Linkoping University, Sweden > Condensed Matter Theory Group, Uppsala University, Sweden > Eyvaz.Isaev at fysik.uu.se, eyvaz_isaev at yahoo.com > > > > ____________________________________________________________________________________ > Looking for last minute shopping deals? > Find them fast with Yahoo! Search. > http://tools.search.yahoo.com/newsearch/category.php?category=shopping > _______________________________________________ > Pw_forum mailing list > Pw_forum at pwscf.org > http://www.democritos.it/mailman/listinfo/pw_forum > > > --- > Stefano Baroni - SISSA & DEMOCRITOS National Simulation Center - Trieste > [+39] 040 3787 406 (tel) -528 (fax) / stefanobaroni (skype) > > Please, if possible, don't send me MS Word or PowerPoint attachments > Why? See: http://www.gnu.org/philosophy/no-word-attachments.html > > > > > _______________________________________________ > Pw_forum mailing list > Pw_forum at pwscf.org > http://www.democritos.it/mailman/listinfo/pw_forum > > -- Hai-Ping Lan Department of Electronics , Peking University , Bejing, 100871 lanhaiping at gmail.com, hplan at pku.edu.cn -------------- next part -------------- An HTML attachment was scrubbed... URL: http://www.democritos.it/pipermail/pw_forum/attachments/20080125/667e03a6/attachment.htm From bnrj.rudra at yahoo.com Fri Jan 25 13:43:17 2008 From: bnrj.rudra at yahoo.com (Rudra Banerjee) Date: Fri, 25 Jan 2008 12:43:17 +0000 (GMT) Subject: [Pw_forum] fermi energy Message-ID: <779605.34980.qm@web94102.mail.in2.yahoo.com> when i am plotting band structure for non-metallic molecules, how should i choose fermi energy? -- Rudra Have a Nice Time http://www.bose..res.in/~rudra Please, if possible, don't send me MS Word or PowerPoint attachments Why?See: http://www.gnu.org/philosophy/no-word-attachments.html Download prohibited? No problem. CHAT from any browser, without download. Go to http://in.messenger.yahoo.com/webmessengerpromo.php/ -------------- next part -------------- An HTML attachment was scrubbed... URL: http://www.democritos.it/pipermail/pw_forum/attachments/20080125/29bdc6e4/attachment.htm From baroni at sissa.it Fri Jan 25 13:51:56 2008 From: baroni at sissa.it (Stefano Baroni) Date: Fri, 25 Jan 2008 13:51:56 +0100 Subject: [Pw_forum] fermi energy In-Reply-To: <779605.34980.qm@web94102.mail.in2.yahoo.com> References: <779605.34980.qm@web94102.mail.in2.yahoo.com> Message-ID: Hi Rudra: I am a bit embarassed by your question: On Jan 25, 2008, at 1:43 PM, Rudra Banerjee wrote: > when i am plotting band structure for non-metallic molecules, do molecules display any band structure? (I thought the very concept of energy bands would apply to infinite systems, which molecules are not) how would you distinguish a metallic molecule from a non metallic one? (again, being a metal or an insulator is a property of extended systems, not of molecules) > how should i choose fermi energy? for an infinite system, at zero temperature the fermi energy is ill defined and may be chosen anywhere in the energy gap. at small T, it lies at mid gap Please, do not forget to include your affiliation while signing Stefano Baroni > > > -- > Rudra > Have a Nice Time > http://www.bose.res.in/~rudra > Please, if possible, don't send me MS Word or PowerPoint attachments > Why?See: http://www.gnu.org/philosophy/no-word-attachments.html > > > > > 5, 50, 500, 5000 - Store N number of mails in your inbox. Click here. > _______________________________________________ > Pw_forum mailing list > Pw_forum at pwscf.org > http://www.democritos.it/mailman/listinfo/pw_forum --- Stefano Baroni - SISSA & DEMOCRITOS National Simulation Center - Trieste [+39] 040 3787 406 (tel) -528 (fax) / stefanobaroni (skype) Please, if possible, don't send me MS Word or PowerPoint attachments Why? See: http://www.gnu.org/philosophy/no-word-attachments.html -------------- next part -------------- An HTML attachment was scrubbed... URL: http://www.democritos.it/pipermail/pw_forum/attachments/20080125/7c871c07/attachment.htm From baroni at sissa.it Fri Jan 25 13:51:56 2008 From: baroni at sissa.it (Stefano Baroni) Date: Fri, 25 Jan 2008 13:51:56 +0100 Subject: [Pw_forum] fermi energy In-Reply-To: <779605.34980.qm@web94102.mail.in2.yahoo.com> References: <779605.34980.qm@web94102.mail.in2.yahoo.com> Message-ID: Hi Rudra: I am a bit embarassed by your question: On Jan 25, 2008, at 1:43 PM, Rudra Banerjee wrote: > when i am plotting band structure for non-metallic molecules, do molecules display any band structure? (I thought the very concept of energy bands would apply to infinite systems, which molecules are not) how would you distinguish a metallic molecule from a non metallic one? (again, being a metal or an insulator is a property of extended systems, not of molecules) > how should i choose fermi energy? for an infinite system, at zero temperature the fermi energy is ill defined and may be chosen anywhere in the energy gap. at small T, it lies at mid gap Please, do not forget to include your affiliation while signing Stefano Baroni > > > -- > Rudra > Have a Nice Time > http://www.bose.res.in/~rudra > Please, if possible, don't send me MS Word or PowerPoint attachments > Why?See: http://www.gnu.org/philosophy/no-word-attachments.html > > > > > 5, 50, 500, 5000 - Store N number of mails in your inbox. Click here. > _______________________________________________ > Pw_forum mailing list > Pw_forum at pwscf.org > http://www.democritos.it/mailman/listinfo/pw_forum --- Stefano Baroni - SISSA & DEMOCRITOS National Simulation Center - Trieste [+39] 040 3787 406 (tel) -528 (fax) / stefanobaroni (skype) Please, if possible, don't send me MS Word or PowerPoint attachments Why? See: http://www.gnu.org/philosophy/no-word-attachments.html -------------- next part -------------- An HTML attachment was scrubbed... URL: http://www.democritos.it/pipermail/pw_forum/attachments/20080125/7c871c07/attachment-0003.htm From calderin at physics.queensu.ca Fri Jan 25 14:30:57 2008 From: calderin at physics.queensu.ca (Lazaro Calderin) Date: Fri, 25 Jan 2008 08:30:57 -0500 (EST) Subject: [Pw_forum] ld1.x problem (not really) In-Reply-To: <21527.82.49.215.221.1201250279.squirrel@webmail.sissa.it> References: <1201239878.26001.57.camel@wolfe> <21527.82.49.215.221.1201250279.squirrel@webmail.sissa.it> Message-ID: <50411.81.172.21.147.1201267857.squirrel@physics.queensu.ca> > You should only put the valence states in test configuration, e.g. > > configts(1) = '6s2 6p3' > Thanks in deep Andrea and Lorenzo. I completely missed that ... > Oh, and you have a ghost in the s component at ~ -4 rydberg and one in the > p channel at ~ 0.8 Ry. It means that your rcloc is probably far too small. Thanks Lorenzo, I am aware of the ghosts. That's the very first try, so there is a lot of run for improvement. > Even if the test works in ld1 this pseudo will never be usable in pw Humm .. I wonder why. Those ghosts are out of the interval of -2.5 to 0 Ry which one would expect to be the important one for the bonding in this case. Besides the log derivative is reproduced very well in that interval. So, all the ground state properties calculated with that psp should be fine except, perhaps, the band gap in systems with one. It should also work for metals without any problems. Bye, L. From paulatto at sissa.it Fri Jan 25 17:02:04 2008 From: paulatto at sissa.it (Lorenzo Paulatto) Date: Fri, 25 Jan 2008 17:02:04 +0100 (CET) Subject: [Pw_forum] ld1.x problem (not really) In-Reply-To: <50411.81.172.21.147.1201267857.squirrel@physics.queensu.ca> References: <1201239878.26001.57.camel@wolfe> <21527.82.49.215.221.1201250279.squirrel@webmail.sissa.it> <50411.81.172.21.147.1201267857.squirrel@physics.queensu.ca> Message-ID: <8601.147.122.5.182.1201276924.squirrel@webmail.sissa.it> On Fri, January 25, 2008 14:30, Lazaro Calderin wrote: > Those ghosts are out of the interval of -2.5 to 0 Ry which one would > expect to be the important one for the bonding in this case. This is an optimistic assumption; in my modest experience I have found that a ghosts at E < E_atomic_state always breaks the potential, at E < 1 Ry usually breaks it, and at E < 5~6 Ry causes erratic and slow convergence. Try to test it in pw in a single atom configuration: it may converge fine as long as you start from atomic wavefunctions. But if you start with some random wavefunctions it converge slowly to a deeper "haunted" configuration. Bye -- Lorenzo Paulatto SISSA & DEMOCRITOS (Trieste) +39 040 3787 511 http://people.sissa.it/~paulatto/ ---------------------------------------------------------------- SISSA Webmail https://webmail.sissa.it/ Powered by SquirrelMail http://www.squirrelmail.org/ From amigliore at cmm.upenn.edu Fri Jan 25 20:10:44 2008 From: amigliore at cmm.upenn.edu (Agostino Migliore) Date: Fri, 25 Jan 2008 14:10:44 -0500 (EST) Subject: [Pw_forum] fermi energy In-Reply-To: <779605.34980.qm@web94102.mail.in2.yahoo.com> References: <779605.34980.qm@web94102.mail.in2.yahoo.com> Message-ID: <3736.130.91.67.162.1201288244.squirrel@cmm.upenn.edu> Hello Rudra With reference to the level structure of a molecule (indeed, only for a very large molecule, with several thousands of atoms, you could speak about a level structure which approximately resembles a "band" structure, although it is still not rigorously the case: it is not an infinite system), a chemical potential can be defined which lies at mid HOMO-LUMO gap for integer occupation numbers. It lies still at mid gap for small brodaning of fractional occupations, when only HOMO and LUMO are fractionally occupied, although it is no more the case for sufficiently high broadening, in every broadening scheme (as you can get from the expression of fractional occupations in the chosen broadening scheme; e.g., a good reference are Phys Rev B 58, 13459 and references therein). Hello Agostino Migliore > when i am plotting band structure for non-metallic molecules, how should i > choose fermi energy? > > -- > Rudra > Have a Nice Time > http://www.bose..res.in/~rudra > Please, if possible, don't send me MS Word or PowerPoint attachments > Why?See: http://www.gnu.org/philosophy/no-word-attachments.html > > > > > > > Download prohibited? No problem. CHAT from any browser, without > download. Go to > http://in.messenger.yahoo.com/webmessengerpromo.php/_______________________________________________ > Pw_forum mailing list > Pw_forum at pwscf.org > http://www.democritos.it/mailman/listinfo/pw_forum > From calderin at physics.queensu.ca Fri Jan 25 21:03:22 2008 From: calderin at physics.queensu.ca (Lazaro Calderin) Date: Fri, 25 Jan 2008 15:03:22 -0500 (EST) Subject: [Pw_forum] ld1.x problem (not really) In-Reply-To: <8601.147.122.5.182.1201276924.squirrel@webmail.sissa.it> References: <1201239878.26001.57.camel@wolfe> <21527.82.49.215.221.1201250279.squirrel@webmail.sissa.it> <50411.81.172.21.147.1201267857.squirrel@physics.queensu.ca> <8601.147.122.5.182.1201276924.squirrel@webmail.sissa.it> Message-ID: <50924.81.172.21.147.1201291402.squirrel@physics.queensu.ca> > > This is an optimistic assumption; in my modest experience I have found > that a ghosts at E < E_atomic_state always breaks the potential, at E < 1 > Ry usually breaks it, and at E < 5~6 Ry causes erratic and slow > convergence. > Interesting .. Have you found this problem only with pw ? I guess that ghost states that introduce large differences between the log derivatives in the bonding energy window, will create spurious features in the potential like more minima, walls, etc, that break the code. But if in the presence of ghosts the log derivatives are the same in the bonding energy window then the ghost should be benign and the code should run fine. > Try to test it in pw in a single atom configuration: it may converge fine > as long as you start from atomic wavefunctions. But if you start with some > random wavefunctions it converge slowly to a deeper "haunted" > configuration. > Well in this case there is plenty of room for improvements. Probably just increasing rcloc as you said and/or including a d projector will bust the ghosts. I was more interested in understanding what may be the reasons for pw and the method in general to break down in the presence of ghosts like those in this example. Thanks for the exchange .. bye, L. From baroni at sissa.it Sat Jan 26 09:43:36 2008 From: baroni at sissa.it (Stefano Baroni) Date: Sat, 26 Jan 2008 09:43:36 +0100 Subject: [Pw_forum] ld1.x problem (not really) In-Reply-To: <8601.147.122.5.182.1201276924.squirrel@webmail.sissa.it> References: <1201239878.26001.57.camel@wolfe> <21527.82.49.215.221.1201250279.squirrel@webmail.sissa.it> <50411.81.172.21.147.1201267857.squirrel@physics.queensu.ca> <8601.147.122.5.182.1201276924.squirrel@webmail.sissa.it> Message-ID: <3EEB65F4-92D2-4306-A8BE-88AFDCA1B1BE@sissa.it> On Jan 25, 2008, at 5:02 PM, Lorenzo Paulatto wrote: > > On Fri, January 25, 2008 14:30, Lazaro Calderin wrote: >> Those ghosts are out of the interval of -2.5 to 0 Ry which one would >> expect to be the important one for the bonding in this case. > > This is an optimistic assumption; in my modest experience I have found > that a ghosts at E < E_atomic_state always breaks the potential, at > E < 1 > Ry usually breaks it, and at E < 5~6 Ry causes erratic and slow > convergence. > > Try to test it in pw in a single atom configuration: it may > converge fine > as long as you start from atomic wavefunctions. But if you start > with some > random wavefunctions it converge slowly to a deeper "haunted" > configuration. that's the fault of the Ritz's variational principle, boys! SB --- Stefano Baroni - SISSA & DEMOCRITOS National Simulation Center - Trieste [+39] 040 3787 406 (tel) -528 (fax) / stefanobaroni (skype) Please, if possible, don't send me MS Word or PowerPoint attachments Why? See: http://www.gnu.org/philosophy/no-word-attachments.html -------------- next part -------------- An HTML attachment was scrubbed... URL: http://www.democritos.it/pipermail/pw_forum/attachments/20080126/cc026399/attachment.htm From calderin at physics.queensu.ca Sat Jan 26 22:20:31 2008 From: calderin at physics.queensu.ca (Lazaro Calderin) Date: Sat, 26 Jan 2008 22:20:31 +0100 Subject: [Pw_forum] [Re:] ld1.x problem (not really) Message-ID: <1201382441.22473.103.camel@wolfe> > that's the fault of the Ritz's variational principle, boys! Stefano .. yes, I agree .. but just in principle. In practice I have came across tested pseudopotentials that give good results for the solid, however, they have ghosts. I think I can give as examples the Pb, Tl and Bi pseudos in the uspp pseudopotentials library. There is a ghost that do not appear in the tests with bessel functions but if you go to deep enough in energy (down to -8 ry) in the calculation of the log derivatives you should see it (I still should have those tests somewhere). So the program some how must be omitting that ghost probably because is out of the range of interest. Any ways, I think it is a very extended practice to keep an eye only on the range of energy of the solid when designing the psps. (If I remember well that is even a recommendation in some tutorials (uspp?)). Thanks, L. -- ============================================================= Lazaro Calderin, Ph.D. Dpt. of Theoretical, Atomic and Optical Physics Univ. of Valladolid, 47005 Valladolid. Spain Phone: +34 983 423891 Fax: +34 983 423013 E-mails: calderin at fta.uva.es or calderin at physics.queensu.ca From calderin at physics.queensu.ca Sat Jan 26 23:11:56 2008 From: calderin at physics.queensu.ca (Lazaro Calderin) Date: Sat, 26 Jan 2008 23:11:56 +0100 Subject: [Pw_forum] ld1.x problem -II Message-ID: <1201385526.22473.125.camel@wolfe> Hi again, I corrected the error in the input file for testing, but still ld1.x is giving the message below in testing mode only. It does not appear when generating and testing at the same time. I am also finishing up programming the kubo formula and do not have time to dig into the ld1 code now, so any info would be much appreciated. Thanks, L. --------------------------------------------------------- The ld1.x message is: from compute phi : info # -1 negative determinant 1 ld= -0.022618 f2ae 1.185735 faenor 0.377893 The input files are: For generation and testing ........................... &input title='Bi', zed=83.0, rel=1, iswitch=3, rlderiv=3.50, eminld=-4.0, emaxld=4.0, deld=0.02, nld=2, config='[Xe] 4f14 5d10 6s2 6p3', dft='LDA', xmin=-7.0, dx=0.01250, rmax=155.0 / &inputp pseudotype=2, tm=.true., file_pseudopw='Bi-lda-cc-tm.UPF', nlcc=.true., rcloc=1.80, / 2 6S 1 0 2.00 0.00 1.8564 1.8564 6P 2 1 3.00 0.00 2.4441 2.4441 For testing only ................ &input title='Bi', zed=83.0, rel=1, iswitch=2, config='[Xe] 4f14 5d10 6s2 6p3' dft='LDA', / &test nconf=1 file_pseudo='Bi-lda-cc-tm.UPF', configts(1)= '6s2 6p3' / -- ============================================================= Lazaro Calderin, Ph.D. Dpt. of Theoretical, Atomic and Optical Physics Univ. of Valladolid, 47005 Valladolid. Spain Phone: +34 983 423891 Fax: +34 983 423013 E-mails: calderin at fta.uva.es or calderin at physics.queensu.ca From calderin at physics.queensu.ca Sun Jan 27 04:19:11 2008 From: calderin at physics.queensu.ca (Lazaro Calderin) Date: Sun, 27 Jan 2008 04:19:11 +0100 Subject: [Pw_forum] [Re:] ld1.x problem (not really) In-Reply-To: <1201382441.22473.103.camel@wolfe> References: <1201382441.22473.103.camel@wolfe> Message-ID: <1201403961.22473.145.camel@wolfe> On Sat, 2008-01-26 at 22:20 +0100, Lazaro Calderin wrote: > > that's the fault of the Ritz's variational principle, boys! > > Stefano .. yes, I agree .. but just in principle. > In practice I have came across tested pseudopotentials that give > good results for the solid, however, they have ghosts. I think I can > give as examples the Pb, Tl and Bi pseudos in the uspp pseudopotentials > library. There is a ghost that do not appear in the tests with bessel > functions but if you go to deep enough in energy (down to -8 ry) in > calculation of the log derivatives you should see it (I still should > have those tests somewhere). So the program some how must be omitting > that ghost probably because is out of the range of interest. No .. I am definitely wrong .. I checked my notes and those are states of the all electron hamiltonian. But je .. I still think I came across that case .. > Any ways, I think it is a very extended practice to keep an eye only on > the range of energy of the solid when designing the psps. (If I remember > well that is even a recommendation in some tutorials (uspp?)). That is the case but it is dangerous practice. Sorry for the monologue .. it's been a long coughing night, L. -- ============================================================= Lazaro Calderin, Ph.D. Dpt. of Theoretical, Atomic and Optical Physics Univ. of Valladolid, 47005 Valladolid. Spain Phone: +34 983 423891 Fax: +34 983 423013 E-mails: calderin at fta.uva.es or calderin at physics.queensu.ca From hashem.yamani at gmail.com Sun Jan 27 16:24:31 2008 From: hashem.yamani at gmail.com (Hashem Al-Yamani) Date: Sun, 27 Jan 2008 17:24:31 +0200 Subject: [Pw_forum] incompatable nr1 or nr2 or nr3 ???!!! Message-ID: <82a593400801270724s4b97bd23jc6cb3e7030ea85b9@mail.gmail.com> Dear All ; While I were trying to get the charge density difference between adsorped O and clean surface i had the following error : Program POST-PROC v.3.2 starts ... Today is 27Jan2008 at 19:57:57 Reading header from file IrO2OtopIrcharge2D Reading data from file IrO2OtopIrcharge2D Reading data from file IrO2sclnsurfcharge2D %%%%%%%%%%%%%%%%%%%%%%%%%%%%%%%%%%%%%%%%%%%%%%%%%%%%%%%%%%%%%%%%%%%%%%%%%%%%%% from chdens : error # 1 incompatible nr1 or nr2 or nr3 %%%%%%%%%%%%%%%%%%%%%%%%%%%%%%%%%%%%%%%%%%%%%%%%%%%%%%%%%%%%%%%%%%%%%%%%%%%%%% stopping ... Any one have an idea what could be the problem ??? my input file was as follow : &inputpp / &plot nfile = 3 filepp(1) = 'IrO2OtopIrcharge2D' filepp(2) = 'IrO2sclnsurfcharge2D' filepp(3) = 'Ocharge2D' weight(1) = 1.0 weight(2) = -1.0 weight(3) = -1.0 iflag = 2 output_format = 2 fileout = 'IrO2chargediff.dat' e1(1) =2.0, e1(2)=2.0, e1(3) = 0.0, e2(1) =0.0, e2(2)=0.0, e2(3) = 2.0, nx=60, ny=60 / any help would be very appreciaple ... Thanks in advance Have a nice day . H. Al-Yamani physics department Jordan University -------------- next part -------------- An HTML attachment was scrubbed... URL: http://www.democritos.it/pipermail/pw_forum/attachments/20080127/c489733c/attachment.htm From eyvaz_isaev at yahoo.com Sun Jan 27 23:51:07 2008 From: eyvaz_isaev at yahoo.com (Eyvaz Isaev) Date: Sun, 27 Jan 2008 14:51:07 -0800 (PST) Subject: [Pw_forum] incompatable nr1 or nr2 or nr3 ???!!! In-Reply-To: <82a593400801270724s4b97bd23jc6cb3e7030ea85b9@mail.gmail.com> Message-ID: <121253.56618.qm@web60321.mail.yahoo.com> Hi, --- Hashem Al-Yamani wrote: > > Program POST-PROC v.3.2 starts ... > Today is 27Jan2008 at 19:57:57 > Reading header from file IrO2OtopIrcharge2D > Reading data from file IrO2OtopIrcharge2D > Reading data from file IrO2sclnsurfcharge2D > %%%%%%%%%%%%%%%%%%%%%%%%%%%%%%%%%%%%%%%%%%%%%%%%%%%%%%%%%%%%%%%%%%%%%%%%%%%%%% > from chdens : error # 1 > incompatible nr1 or nr2 or nr3 > > %%%%%%%%%%%%%%%%%%%%%%%%%%%%%%%%%%%%%%%%%%%%%%%%%%%%%%%%%%%%%%%%%%%%%%%%%%%%%% > > stopping ... > Most likely it is due to different nr1,2,3, generated by default using ecutrho specified (it might be due to different symmetry, supercell size). They are used for FFT for charge density. Please check FFT grids in your output files for sure. You can specify these grids (nr1,nr2,nr3) manually for both types of calculations to avoid incompatibility. Bests, Eyvaz. > > Any one have an idea what could be the problem ??? > > > my input file was as follow : > > &inputpp > / > &plot > nfile = 3 > filepp(1) = 'IrO2OtopIrcharge2D' > filepp(2) = 'IrO2sclnsurfcharge2D' > filepp(3) = 'Ocharge2D' > weight(1) = 1.0 > weight(2) = -1.0 > weight(3) = -1.0 > iflag = 2 > output_format = 2 > fileout = 'IrO2chargediff.dat' > e1(1) =2.0, e1(2)=2.0, e1(3) = 0.0, > e2(1) =0.0, e2(2)=0.0, e2(3) = 2.0, > nx=60, ny=60 > / > > > any help would be very appreciaple ... Thanks in > advance > > Have a nice day . > > H. Al-Yamani > physics department > Jordan University > > _______________________________________________ > Pw_forum mailing list > Pw_forum at pwscf.org > http://www.democritos.it/mailman/listinfo/pw_forum > ------------------------------------------------------------------- Prof. Eyvaz Isaev, Theoretical Physics Department, Moscow State Institute of Steel & Alloys, Russia, IFM, Linkoping University, Sweden Condensed Matter Theory Group, Uppsala University, Sweden Eyvaz.Isaev at fysik.uu.se, eyvaz_isaev at yahoo.com ____________________________________________________________________________________ Be a better friend, newshound, and know-it-all with Yahoo! Mobile. Try it now. http://mobile.yahoo.com/;_ylt=Ahu06i62sR8HDtDypao8Wcj9tAcJ From quantum-espresso at 163.com Mon Jan 28 07:15:45 2008 From: quantum-espresso at 163.com (quantum-espresso at 163.com) Date: Mon, 28 Jan 2008 14:15:45 +0800 (CST) Subject: [Pw_forum] help In-Reply-To: References: Message-ID: <17055110.488611201500945161.JavaMail.coremail@bj163app45.163.com> help ?2008-01-27?pw_forum-request at pwscf.org ??? Send Pw_forum mailing list submissions to pw_forum at pwscf.org To subscribe or unsubscribe via the World Wide Web, visit http://www.democritos.it/mailman/listinfo/pw_forum or, via email, send a message with subject or body 'help' to pw_forum-request at pwscf.org You can reach the person managing the list at pw_forum-owner at pwscf.org When replying, please edit your Subject line so it is more specific than "Re: Contents of Pw_forum digest..." Today's Topics: 1. Re: ld1.x problem (not really) (Stefano Baroni) 2. [Re:] ld1.x problem (not really) (Lazaro Calderin) 3. ld1.x problem -II (Lazaro Calderin) 4. Re: [Re:] ld1.x problem (not really) (Lazaro Calderin) ---------------------------------------------------------------------- Message: 1 Date: Sat, 26 Jan 2008 09:43:36 +0100 From: Stefano Baroni Subject: Re: [Pw_forum] ld1.x problem (not really) To: PWSCF Forum Message-ID: <3EEB65F4-92D2-4306-A8BE-88AFDCA1B1BE at sissa.it> Content-Type: text/plain; charset="us-ascii" On Jan 25, 2008, at 5:02 PM, Lorenzo Paulatto wrote: > > On Fri, January 25, 2008 14:30, Lazaro Calderin wrote: >> Those ghosts are out of the interval of -2.5 to 0 Ry which one would >> expect to be the important one for the bonding in this case. > > This is an optimistic assumption; in my modest experience I have found > that a ghosts at E < E_atomic_state always breaks the potential, at > E < 1 > Ry usually breaks it, and at E < 5~6 Ry causes erratic and slow > convergence. > > Try to test it in pw in a single atom configuration: it may > converge fine > as long as you start from atomic wavefunctions. But if you start > with some > random wavefunctions it converge slowly to a deeper "haunted" > configuration. that's the fault of the Ritz's variational principle, boys! SB --- Stefano Baroni - SISSA & DEMOCRITOS National Simulation Center - Trieste [+39] 040 3787 406 (tel) -528 (fax) / stefanobaroni (skype) Please, if possible, don't send me MS Word or PowerPoint attachments Why? See: http://www.gnu.org/philosophy/no-word-attachments.html -------------- next part -------------- An HTML attachment was scrubbed... URL: http://www.democritos.it/pipermail/pw_forum/attachments/20080126/cc026399/attachment.html ------------------------------ Message: 2 Date: Sat, 26 Jan 2008 22:20:31 +0100 From: Lazaro Calderin Subject: [Pw_forum] [Re:] ld1.x problem (not really) To: pw_forum at pwscf.org Message-ID: <1201382441.22473.103.camel at wolfe> Content-Type: text/plain > that's the fault of the Ritz's variational principle, boys! Stefano .. yes, I agree .. but just in principle. In practice I have came across tested pseudopotentials that give good results for the solid, however, they have ghosts. I think I can give as examples the Pb, Tl and Bi pseudos in the uspp pseudopotentials library. There is a ghost that do not appear in the tests with bessel functions but if you go to deep enough in energy (down to -8 ry) in the calculation of the log derivatives you should see it (I still should have those tests somewhere). So the program some how must be omitting that ghost probably because is out of the range of interest. Any ways, I think it is a very extended practice to keep an eye only on the range of energy of the solid when designing the psps. (If I remember well that is even a recommendation in some tutorials (uspp?)). Thanks, L. -- ============================================================= Lazaro Calderin, Ph.D. Dpt. of Theoretical, Atomic and Optical Physics Univ. of Valladolid, 47005 Valladolid. Spain Phone: +34 983 423891 Fax: +34 983 423013 E-mails: calderin at fta.uva.es or calderin at physics.queensu.ca ------------------------------ Message: 3 Date: Sat, 26 Jan 2008 23:11:56 +0100 From: Lazaro Calderin Subject: [Pw_forum] ld1.x problem -II To: pw_forum at pwscf.org Message-ID: <1201385526.22473.125.camel at wolfe> Content-Type: text/plain Hi again, I corrected the error in the input file for testing, but still ld1.x is giving the message below in testing mode only. It does not appear when generating and testing at the same time. I am also finishing up programming the kubo formula and do not have time to dig into the ld1 code now, so any info would be much appreciated. Thanks, L. --------------------------------------------------------- The ld1.x message is: from compute phi : info # -1 negative determinant 1 ld= -0.022618 f2ae 1.185735 faenor 0.377893 The input files are: For generation and testing ........................... &input title='Bi', zed=83.0, rel=1, iswitch=3, rlderiv=3.50, eminld=-4.0, emaxld=4.0, deld=0.02, nld=2, config='[Xe] 4f14 5d10 6s2 6p3', dft='LDA', xmin=-7.0, dx=0.01250, rmax=155.0 / &inputp pseudotype=2, tm=.true., file_pseudopw='Bi-lda-cc-tm.UPF', nlcc=.true., rcloc=1.80, / 2 6S 1 0 2.00 0.00 1.8564 1.8564 6P 2 1 3.00 0.00 2.4441 2.4441 For testing only ................ &input title='Bi', zed=83.0, rel=1, iswitch=2, config='[Xe] 4f14 5d10 6s2 6p3' dft='LDA', / &test nconf=1 file_pseudo='Bi-lda-cc-tm.UPF', configts(1)= '6s2 6p3' / -- ============================================================= Lazaro Calderin, Ph.D. Dpt. of Theoretical, Atomic and Optical Physics Univ. of Valladolid, 47005 Valladolid. Spain Phone: +34 983 423891 Fax: +34 983 423013 E-mails: calderin at fta.uva.es or calderin at physics.queensu.ca ------------------------------ Message: 4 Date: Sun, 27 Jan 2008 04:19:11 +0100 From: Lazaro Calderin Subject: Re: [Pw_forum] [Re:] ld1.x problem (not really) To: PWSCF Forum Message-ID: <1201403961.22473.145.camel at wolfe> Content-Type: text/plain On Sat, 2008-01-26 at 22:20 +0100, Lazaro Calderin wrote: > > that's the fault of the Ritz's variational principle, boys! > > Stefano .. yes, I agree .. but just in principle. > In practice I have came across tested pseudopotentials that give > good results for the solid, however, they have ghosts. I think I can > give as examples the Pb, Tl and Bi pseudos in the uspp pseudopotentials > library. There is a ghost that do not appear in the tests with bessel > functions but if you go to deep enough in energy (down to -8 ry) in > calculation of the log derivatives you should see it (I still should > have those tests somewhere). So the program some how must be omitting > that ghost probably because is out of the range of interest. No .. I am definitely wrong .. I checked my notes and those are states of the all electron hamiltonian. But je .. I still think I came across that case .. > Any ways, I think it is a very extended practice to keep an eye only on > the range of energy of the solid when designing the psps. (If I remember > well that is even a recommendation in some tutorials (uspp?)). That is the case but it is dangerous practice. Sorry for the monologue .. it's been a long coughing night, L. -- ============================================================= Lazaro Calderin, Ph.D. Dpt. of Theoretical, Atomic and Optical Physics Univ. of Valladolid, 47005 Valladolid. Spain Phone: +34 983 423891 Fax: +34 983 423013 E-mails: calderin at fta.uva.es or calderin at physics.queensu.ca ------------------------------ _______________________________________________ Pw_forum mailing list Pw_forum at pwscf.org http://www.democritos.it/mailman/listinfo/pw_forum End of Pw_forum Digest, Vol 7, Issue 33 *************************************** -------------- next part -------------- An HTML attachment was scrubbed... URL: http://www.democritos.it/pipermail/pw_forum/attachments/20080128/44f7efa7/attachment.htm From giannozz at nest.sns.it Mon Jan 28 17:16:20 2008 From: giannozz at nest.sns.it (Paolo Giannozzi) Date: Mon, 28 Jan 2008 17:16:20 +0100 Subject: [Pw_forum] ld1.x problem -II In-Reply-To: <1201385526.22473.125.camel@wolfe> References: <1201385526.22473.125.camel@wolfe> Message-ID: <492298E8-3FB6-4E58-890D-F44F6E9C631E@nest.sns.it> On Jan 26, 2008, at 23:11 , Lazaro Calderin wrote: > pseudotype=2, > [...] > 6S 1 0 2.00 0.00 1.8564 1.8564 > 6P 2 1 3.00 0.00 2.4441 2.4441 pseudotype=2 means "more than one channel per angular momentum". Since you have one, you should set pseudotype=1 instead. Anyway, it should work also with pseudotype=2 . In fact, it works for me ... for both the 3.2.3 and the cvs version P. --- Paolo Giannozzi, Dept of Physics, University of Udine via delle Scienze 208, 33100 Udine, Italy Phone +39-0432-558216, fax +39-0432-558222 From calderin at physics.queensu.ca Mon Jan 28 18:01:50 2008 From: calderin at physics.queensu.ca (Lazaro Calderin) Date: Mon, 28 Jan 2008 18:01:50 +0100 Subject: [Pw_forum] ld1.x problem -II In-Reply-To: <492298E8-3FB6-4E58-890D-F44F6E9C631E@nest.sns.it> References: <1201385526.22473.125.camel@wolfe> <492298E8-3FB6-4E58-890D-F44F6E9C631E@nest.sns.it> Message-ID: <1201539720.3361.40.camel@wolfe> > pseudotype=2 means "more than one channel per > angular momentum". Since you have one, you should > set pseudotype=1 instead. Anyway, it should work > also with pseudotype=2 . In fact, it works for me ... > for both the 3.2.3 and the cvs version ld1 runs in both modes and the numbers in both outputs are the same. But I get that message only in the testing output (with both pseudotype=1,2). Before I start checking compilation options and so on, I wonder if you could make sure that there is no 'info' message in your testing output. Thanks in deep, L. -- ============================================================= Lazaro Calderin, Ph.D. Dpt. of Theoretical, Atomic and Optical Physics Univ. of Valladolid, 47005 Valladolid. Spain Phone: +34 983 423891 Fax: +34 983 423013 E-mails: calderin at fta.uva.es or calderin at physics.queensu.ca From giannozz at nest.sns.it Tue Jan 29 10:59:47 2008 From: giannozz at nest.sns.it (Paolo Giannozzi) Date: Tue, 29 Jan 2008 10:59:47 +0100 Subject: [Pw_forum] ld1.x problem -II In-Reply-To: <1201539720.3361.40.camel@wolfe> References: <1201385526.22473.125.camel@wolfe> <492298E8-3FB6-4E58-890D-F44F6E9C631E@nest.sns.it> <1201539720.3361.40.camel@wolfe> Message-ID: <479EF913.8060008@nest.sns.it> Lazaro Calderin wrote: > ld1 runs in both modes and the numbers in both outputs are > the same. But I get that message only in the testing > output (with both pseudotype=1,2). Before I start checking > compilation options and so on, I wonder if you could make > sure that there is no 'info' message in your testing output. you are right, the message is there. This has to be fixed, because if one does not notice it, the code will go on giving the impression that everything is ok, while this kind of errors invariably means that there is some nasty problem in the generated pseudopotential Paolo -- Paolo Giannozzi, Democritos and University of Udine, Italy From giannozz at nest.sns.it Tue Jan 29 10:23:07 2008 From: giannozz at nest.sns.it (Paolo Giannozzi) Date: Tue, 29 Jan 2008 10:23:07 +0100 Subject: [Pw_forum] ld1.x problem -II In-Reply-To: <1201539720.3361.40.camel@wolfe> References: <1201385526.22473.125.camel@wolfe> <492298E8-3FB6-4E58-890D-F44F6E9C631E@nest.sns.it> <1201539720.3361.40.camel@wolfe> Message-ID: <479EF07B.80109@nest.sns.it> Lazaro Calderin wrote: > > ld1 runs in both modes and the numbers in both outputs are > the same. But I get that message only in the testing > output (with both pseudotype=1,2). Before I start checking > compilation options and so on, I wonder if you could make > sure that there is no 'info' message in your testing output. you are right, the message is there. This has to be fixed, because if one does not notice it, the code will go on giving the impression that everything is ok, while this kind of errors invariably means that there is some nasty problem in the generated pseudopotential Paolo -- Paolo Giannozzi, Democritos and University of Udine, Italy From calderin at physics.queensu.ca Tue Jan 29 12:06:42 2008 From: calderin at physics.queensu.ca (Lazaro Calderin) Date: Tue, 29 Jan 2008 12:06:42 +0100 Subject: [Pw_forum] ld1.x problem -II In-Reply-To: <479EF913.8060008@nest.sns.it> References: <1201385526.22473.125.camel@wolfe> <492298E8-3FB6-4E58-890D-F44F6E9C631E@nest.sns.it> <1201539720.3361.40.camel@wolfe> <479EF913.8060008@nest.sns.it> Message-ID: <1201604812.3361.74.camel@wolfe> > > ld1 runs in both modes and the numbers in both outputs are > > the same. But I get that message only in the testing > > output (with both pseudotype=1,2). Before I start checking > > compilation options and so on, I wonder if you could make > > sure that there is no 'info' message in your testing output. > > you are right, the message is there. This has to be fixed, > because if one does not notice it, the code will go on giving > the impression that everything is ok, while this kind of > errors invariably means that there is some nasty problem > in the generated pseudopotential I was afraid of some thing like that. But if the pseudopotential is wrong why the message does not appear in the generation mode as well? After all we are using the same electronic configuration in both modes. It seems that there is some sort of inconsistency between the two modes. In addition, the error message seems to replace the output of a couple of messages saying that the log derivatives were calculated. If you run the input file with rcloc=2.44 the log derivatives look fine but you still get the error message in the testing output. So perhaps the 'generation and testing' mode is working fine and the problem is in the 'testing' mode alone. Thanks, L. -- ============================================================= Lazaro Calderin, Ph.D. Dpt. of Theoretical, Atomic and Optical Physics Univ. of Valladolid, 47005 Valladolid. Spain Phone: +34 983 423891 Fax: +34 983 423013 E-mails: calderin at fta.uva.es or calderin at physics.queensu.ca From lanhaiping at gmail.com Tue Jan 29 17:52:58 2008 From: lanhaiping at gmail.com (lan haiping) Date: Tue, 29 Jan 2008 17:52:58 +0100 Subject: [Pw_forum] Problem of bands data In-Reply-To: References: Message-ID: Dear All, I have performed a bands calculation on a carbon nanotube system (7,0). SCF and NSCF calculations are well converged, but the bands data obtained from bands.x is very strange. I then checked the bands data between the output of nscf calculation and the data from bands.x calculation, and found some obvious differences , for example , at kpoint (0.00 0.000 0.8803) , output of nscf calculation gave data below : k = 0.0000 0.0000 0.8803 band energies (ev): -23.2631 -22.8169 -22.8148 -21.5536 -21.5517 -19.6376 -19.6331 -19.2573 -18.8505 -18.8483 -17.7098 -17.7079 -17.3344 -17.3317 -15.8326 -15.7203 -15.7153 -15.5144 -15.5128 -15.0403 -15.0395 -13.5446 -13.5400 -13.5214 -13.5210 -12.6873 -12.6836 -11.6226 -11.3921 -11.3913 -11.0763 -11.0745 -11.0744 -11.0738 -10.9811 -10.9786 -10.0482 -9.7632 -9.7609 -8.5038 -8.5037 -8.3616 -8.3610 -7.6188 -7.6184 -7.3844 -6.9549 -6.9522 -6.9467 -6.9088 -6.3798 -6.3782 -5.0396 -5.0385 -4.8055 -4.8016 -2.9700 -2.3738 -2.3735 -2.1789 -2.1764 -1.8244 -1.8197 -0.1813 0.3902 0.3909 0.4404 0.5333 0.6119 0.6178 0.6886 0.7548 0.7578 0.7763 1.1057 1.1764 1.2288 1.2302 1.3717 1.4616 while, bands data from bands.x is 0.000000 0.000000 0.880282 -23.263 -22.817 -22.815 -21.554 -21.552 -19.638 -19.633 -19.257 -18.850 - 18.848 -17.334 -17.332 -17.710 -15.833 -17.708 -15.514 -15.513 -15.720 -15.715 - 15.040 -15.040 -13.540 -13.545 -12.687 -12.684 -13.521 -13.521 -11.623 -11.075 - 11.076 -11.392 -11.391 -11.074 -11.074 -9.763 -9.761 -8.504 -8.504 -10.048 - 8.362 -8.361 -7.618 -7.619 -7.384 -6.955 -6.947 -6.909 -6.952 -6.380 - 2.970 -2.374 -6.378 -4.805 -4.802 -5.040 -5.038 -2.374 -2.179 -2.176 - 1.824 -1.820 -0.181 0.390 0.391 0.440 0.533 0.612 0.618 0.689 0.755 0.758 0.776 1.176 1.106 1.229 1.230 * *1.372* -10.979 -10.981* 1.462 How to explain this difference ? My input files are attached. -- Hai-Ping Lan Department of Electronics , Peking University , Bejing, 100871 lanhaiping at gmail.com, hplan at pku.edu.cn -- Hai-Ping Lan Department of Electronics , Peking University , Bejing, 100871 lanhaiping at gmail.com, hplan at pku.edu.cn -------------- next part -------------- An HTML attachment was scrubbed... URL: http://www.democritos.it/pipermail/pw_forum/attachments/20080129/b65ff0d8/attachment.htm -------------- next part -------------- A non-text attachment was scrubbed... Name: c70-bnds.in Type: application/octet-stream Size: 80 bytes Desc: not available Url : http://www.democritos.it/pipermail/pw_forum/attachments/20080129/b65ff0d8/attachment.obj -------------- next part -------------- A non-text attachment was scrubbed... Name: c70-scf.in Type: application/octet-stream Size: 2262 bytes Desc: not available Url : http://www.democritos.it/pipermail/pw_forum/attachments/20080129/b65ff0d8/attachment-0001.obj -------------- next part -------------- A non-text attachment was scrubbed... Name: c70-nscf.in Type: application/octet-stream Size: 2727 bytes Desc: not available Url : http://www.democritos.it/pipermail/pw_forum/attachments/20080129/b65ff0d8/attachment-0002.obj From giannozz at nest.sns.it Tue Jan 29 18:26:53 2008 From: giannozz at nest.sns.it (Paolo Giannozzi) Date: Tue, 29 Jan 2008 18:26:53 +0100 Subject: [Pw_forum] ld1.x problem -II In-Reply-To: <1201604812.3361.74.camel@wolfe> References: <1201385526.22473.125.camel@wolfe> <492298E8-3FB6-4E58-890D-F44F6E9C631E@nest.sns.it> <1201539720.3361.40.camel@wolfe> <479EF913.8060008@nest.sns.it> <1201604812.3361.74.camel@wolfe> Message-ID: <479F61DD.3060203@nest.sns.it> Lazaro Calderin wrote: > But if the pseudopotential is wrong why the message does not appear > in the generation mode as well? After all we are using the same > electronic configuration in both modes. but we are not doing the same calculations in the two cases! Maybe the potential is not wrong, we are looking at it. Paolo -- Paolo Giannozzi, Democritos and University of Udine, Italy From lyu7 at ncsu.edu Tue Jan 29 19:50:17 2008 From: lyu7 at ncsu.edu (Liping Yu) Date: Tue, 29 Jan 2008 13:50:17 -0500 Subject: [Pw_forum] Problem of bands data In-Reply-To: References: Message-ID: <479F7569.3000208@ncsu.edu> lan haiping wrote: > > > Dear All, > > I have performed a bands calculation on a carbon nanotube system (7,0). > SCF and NSCF calculations are well converged, but the bands data > obtained from bands.x is very strange. > I then checked the bands data between the output of nscf calculation > and the data from bands.x calculation, and found some obvious > differences , for example , at kpoint (0.00 0.000 0.8803) , output of > nscf calculation gave data below : > k = 0.0000 0.0000 0.8803 band energies (ev): > -23.2631 -22.8169 -22.8148 -21.5536 -21.5517 -19.6376 -19.6331 -19.2573 > -18.8505 -18.8483 -17.7098 -17.7079 -17.3344 -17.3317 -15.8326 -15.7203 > -15.7153 -15.5144 -15.5128 -15.0403 -15.0395 -13.5446 -13.5400 -13.5214 > -13.5210 -12.6873 -12.6836 -11.6226 -11.3921 -11.3913 -11.0763 -11.0745 > -11.0744 -11.0738 -10.9811 -10.9786 -10.0482 -9.7632 -9.7609 -8.5038 > -8.5037 -8.3616 -8.3610 -7.6188 -7.6184 -7.3844 -6.9549 -6.9522 > -6.9467 -6.9088 -6.3798 -6.3782 -5.0396 -5.0385 -4.8055 -4.8016 > -2.9700 -2.3738 -2.3735 -2.1789 -2.1764 -1.8244 -1.8197 -0.1813 > 0.3902 0.3909 0.4404 0.5333 0.6119 0.6178 0.6886 0.7548 > 0.7578 0.7763 1.1057 1.1764 1.2288 1.2302 1.3717 1.4616 > while, bands data from bands.x is > 0.000000 0.000000 0.880282 > -23.263 -22.817 -22.815 -21.554 -21.552 -19.638 -19.633 -19.257 > -18.850 -18.848 > -17.334 -17.332 -17.710 -15.833 -17.708 -15.514 -15.513 -15.720 > -15.715 -15.040 > -15.040 -13.540 -13.545 -12.687 -12.684 -13.521 -13.521 -11.623 > -11.075 -11.076 > -11.392 -11.391 -11.074 -11.074 -9.763 -9.761 -8.504 -8.504 -10.048 -8.362 > -8.361 -7.618 -7.619 -7.384 -6.955 -6.947 -6.909 -6.952 -6.380 -2.970 > -2.374 -6.378 -4.805 -4.802 -5.040 -5.038 -2.374 -2.179 -2.176 -1.824 > -1.820 -0.181 0.390 0.391 0.440 0.533 0.612 0.618 0.689 0.755 > 0.758 0.776 1.176 1.106 1.229 1.230 * *1.372* -10.979 -10.981* 1.462 > > How to explain this difference ? My input files are attached. The difference is in the out-put precision. They are actually same at the same out-put format. > > > > > -- > Hai-Ping Lan > Department of Electronics , > Peking University , Bejing, 100871 > lanhaiping at gmail.com , hplan at pku.edu.cn > > > > > -- > Hai-Ping Lan > Department of Electronics , > Peking University , Bejing, 100871 > lanhaiping at gmail.com , hplan at pku.edu.cn > > ------------------------------------------------------------------------ > > _______________________________________________ > Pw_forum mailing list > Pw_forum at pwscf.org > http://www.democritos.it/mailman/listinfo/pw_forum > -- Liping YU, PhD Candidate Department of Physics & Center for High Performance Simulation North Carolina State University, Raleigh, NC 27695, USA From giannozz at nest.sns.it Wed Jan 30 10:49:10 2008 From: giannozz at nest.sns.it (Paolo Giannozzi) Date: Wed, 30 Jan 2008 10:49:10 +0100 Subject: [Pw_forum] Problem of bands data In-Reply-To: References: Message-ID: <47A04816.3070804@nest.sns.it> lan haiping wrote: > bands data from bands.x is > [..] *1.372 -10.979 -10.981* [..] I can't reproduce your problem. P. -- Paolo Giannozzi, Democritos and University of Udine, Italy From lanhaiping at gmail.com Wed Jan 30 11:19:11 2008 From: lanhaiping at gmail.com (lan haiping) Date: Wed, 30 Jan 2008 11:19:11 +0100 Subject: [Pw_forum] Problem of bands data In-Reply-To: <479F7569.3000208@ncsu.edu> References: <479F7569.3000208@ncsu.edu> Message-ID: Dear Liping, thanks. donot you notice that the data in bands.x output are in some wrong order ? while for data in nscf calculation output, the eigenvalues are sorted ascending . my confusion is about this difference . as far as i know, the bands data should be arranged in ascending or descending styles. Bests H.P On Jan 29, 2008 7:50 PM, Liping Yu wrote: > lan haiping wrote: > > > > > > Dear All, > > > > I have performed a bands calculation on a carbon nanotube system (7,0). > > SCF and NSCF calculations are well converged, but the bands data > > obtained from bands.x is very strange. > > I then checked the bands data between the output of nscf calculation > > and the data from bands.x calculation, and found some obvious > > differences , for example , at kpoint (0.00 0.000 0.8803) , output of > > nscf calculation gave data below : > > k = 0.0000 0.0000 0.8803 band energies (ev): > > -23.2631 -22.8169 -22.8148 -21.5536 -21.5517 -19.6376 -19.6331 -19.2573 > > -18.8505 -18.8483 -17.7098 -17.7079 -17.3344 -17.3317 -15.8326 -15.7203 > > -15.7153 -15.5144 -15.5128 -15.0403 -15.0395 -13.5446 -13.5400 -13.5214 > > -13.5210 -12.6873 -12.6836 -11.6226 -11.3921 -11.3913 -11.0763 -11.0745 > > -11.0744 -11.0738 -10.9811 -10.9786 -10.0482 -9.7632 -9.7609 -8.5038 > > -8.5037 -8.3616 -8.3610 -7.6188 -7.6184 -7.3844 -6.9549 -6.9522 > > -6.9467 -6.9088 -6.3798 -6.3782 -5.0396 -5.0385 -4.8055 -4.8016 > > -2.9700 -2.3738 -2.3735 -2.1789 -2.1764 -1.8244 -1.8197 -0.1813 > > 0.3902 0.3909 0.4404 0.5333 0.6119 0.6178 0.6886 0.7548 > > 0.7578 0.7763 1.1057 1.1764 1.2288 1.2302 1.3717 1.4616 > > while, bands data from bands.x is > > 0.000000 0.000000 0.880282 > > -23.263 -22.817 -22.815 -21.554 -21.552 -19.638 -19.633 -19.257 > > -18.850 -18.848 > > -17.334 -17.332 -17.710 -15.833 -17.708 -15.514 -15.513 -15.720 > > -15.715 -15.040 > > -15.040 -13.540 -13.545 -12.687 -12.684 -13.521 -13.521 -11.623 > > -11.075 -11.076 > > -11.392 -11.391 -11.074 -11.074 -9.763 -9.761 -8.504 -8.504 -10.048 - > 8.362 > > -8.361 -7.618 -7.619 -7.384 -6.955 -6.947 -6.909 -6.952 -6.380 -2.970 > > -2.374 -6.378 -4.805 -4.802 -5.040 -5.038 -2.374 -2.179 -2.176 -1.824 > > -1.820 -0.181 0.390 0.391 0.440 0.533 0.612 0.618 0.689 0.755 > > 0.758 0.776 1.176 1.106 1.229 1.230 * *1.372* -10.979 -10.981* 1.462 > > > > How to explain this difference ? My input files are attached. > The difference is in the out-put precision. They are actually same at > the same out-put format. > > > > > > > > > > -- > > Hai-Ping Lan > > Department of Electronics , > > Peking University , Bejing, 100871 > > lanhaiping at gmail.com , hplan at pku.edu.cn > > > > > > > > > > -- > > Hai-Ping Lan > > Department of Electronics , > > Peking University , Bejing, 100871 > > lanhaiping at gmail.com , hplan at pku.edu.cn > > > > ------------------------------------------------------------------------ > > > > _______________________________________________ > > Pw_forum mailing list > > Pw_forum at pwscf.org > > http://www.democritos.it/mailman/listinfo/pw_forum > > > > > -- > Liping YU, PhD Candidate > Department of Physics & Center for High Performance Simulation > North Carolina State University, Raleigh, NC 27695, USA > > _______________________________________________ > Pw_forum mailing list > Pw_forum at pwscf.org > http://www.democritos.it/mailman/listinfo/pw_forum > -- Hai-Ping Lan Department of Electronics , Peking University , Bejing, 100871 lanhaiping at gmail.com, hplan at pku.edu.cn -------------- next part -------------- An HTML attachment was scrubbed... URL: http://www.democritos.it/pipermail/pw_forum/attachments/20080130/fc04188f/attachment.htm From lanhaiping at gmail.com Wed Jan 30 11:22:08 2008 From: lanhaiping at gmail.com (lan haiping) Date: Wed, 30 Jan 2008 11:22:08 +0100 Subject: [Pw_forum] Problem of bands data In-Reply-To: <47A04816.3070804@nest.sns.it> References: <47A04816.3070804@nest.sns.it> Message-ID: Dear Paolo i donot know what wrong with this problem. i can properly obtain (16,0) nanotube's bands , but cannot produce proper bands of (7,0) nanotube. Can i just plot bands with the value supplied in nscf calculation output ? Bests, H.P On Jan 30, 2008 10:49 AM, Paolo Giannozzi wrote: > lan haiping wrote: > > > bands data from bands.x is > > [..] *1.372 -10.979 -10.981* [..] > > I can't reproduce your problem. > > P. > -- > Paolo Giannozzi, Democritos and University of Udine, Italy > _______________________________________________ > Pw_forum mailing list > Pw_forum at pwscf.org > http://www.democritos.it/mailman/listinfo/pw_forum > -- Hai-Ping Lan Department of Electronics , Peking University , Bejing, 100871 lanhaiping at gmail.com, hplan at pku.edu.cn -------------- next part -------------- An HTML attachment was scrubbed... URL: http://www.democritos.it/pipermail/pw_forum/attachments/20080130/40a924fa/attachment.htm From giannozz at nest.sns.it Wed Jan 30 11:38:29 2008 From: giannozz at nest.sns.it (Paolo Giannozzi) Date: Wed, 30 Jan 2008 11:38:29 +0100 Subject: [Pw_forum] Problem of bands data In-Reply-To: References: <479F7569.3000208@ncsu.edu> Message-ID: <47A053A5.3030304@nest.sns.it> lan haiping wrote: > as far as i know, the bands data should > be arranged in ascending or descending styles. if you want to plot bands, it is convenient to list them in such a way that the N-th band is in the N-th position for all k-points. The code "bands" tries to follow the bands and to re-order the Kohn-Sham eigenvalues in such a way that a simple plot displays the correct crossings > Can i just plot bands with the value supplied in > nscf calculation output ? of course you can P. -- Paolo Giannozzi, Democritos and University of Udine, Italy From hashem.yamani at gmail.com Thu Jan 31 16:09:33 2008 From: hashem.yamani at gmail.com (Hashem Al-Yamani) Date: Thu, 31 Jan 2008 17:09:33 +0200 Subject: [Pw_forum] Incompatable nr1 or nr2 or nr3 Message-ID: <82a593400801310709r63dd0ae1l5da121ea61165f26@mail.gmail.com> Thanx alot Eyvaz and all .... In fact am still working on it .... please tell me if am wrong but I think the problem is due to different - or lets say - incompatible data between the system input file and the adsorbed oxygen ..... am sending the two input files data as it might help ..... *The oxygen input file was : * &control calculation='scf', restart_mode='from_scratch', pseudo_dir='/home/new_generation/espresso-3.2/pseudo/' outdir='/home/new_generation/espresso-3.2/tmp/' prefix = 'O' / &system ibrav=1, celldm(1)=14.0, nat=1, ntyp=1, nbnd=6, nosym=.true., ecutwfc=27.0, ecutrho=216.0, occupations='from_input', nspin=2, starting_magnetization(1)=0.5d0, / &electrons mixing_beta=0.25, conv_thr=1.0E-8, / ATOMIC_SPECIES O 15.99994 O.pbe-rrkjus.UPF ATOMIC_POSITIONS O 0.000000000 0.000000000 0.000000000 K_POINTS (gamma) OCCUPATIONS 1.0 1.0 1.0 1.0 0.0 0.0 1.0 0.33333333333 0.33333333333 0.33333333333 0.0 0.0 ------------------------------------------------------------------------------------------ *And the whole system input file was : * &CONTROL calculation = "scf", restart_mode = 'from_scratch' , prefix = 'IrO2-OtopIr', outdir = '/home/new_generation/espresso-3.2-2/tmp', pseudo_dir = '/home/new_generation/espresso-3.2-2/pseudo' / &SYSTEM ibrav = 6, celldm(1) = 8.6714, celldm(3) = 6.923880978, nat = 29, ntyp = 2, ecutwfc = 55.D0,ecutrho=600.0, occupations = "smearing", smearing = "methfessel-paxton", degauss = 0.03D0, / &ELECTRONS electron_maxstep =300, conv_thr = 1.D-8, mixing_beta = 0.3D0, startingpot = 'atomic' startingwfc = 'atomic' / &IONS ion_dynamics = 'bfgs', / ATOMIC_SPECIES Ir 192.217 Ir.pbe-n-rrkjus.UPF.txt O 15.999 O.pbe-rrkjus.UPF ATOMIC_POSITIONS (bohr) Ir 0.000000000 0.000000000 -12.223315777 Ir -4.335700000 4.335700000 -9.151253653 Ir 0.000000000 0.000000000 -6.022199397 Ir -4.335700000 4.335700000 -3.039010085 Ir 0.000000000 0.000000000 0.000000000 Ir -4.335700000 4.335700000 3.039010085 Ir 0.000000000 0.000000000 6.022199397 Ir -4.335700000 4.335700000 9.151253653 Ir 0.000000000 0.000000000 12.223315777 O 0.000000000 0.000000000 15.557622468 O 0.000000000 0.000000000 -15.557622468 O 2.674536021 2.674536021 0.000000000 O -2.674536021 -2.674536021 0.000000000 O 1.669820608 -1.669820608 -3.018450895 O 1.669820608 -1.669820608 3.018450895 O -1.669820608 1.669820608 -3.018450895 O -1.669820608 1.669820608 3.018450895 O 2.671662729 2.671662729 -6.079652442 O 2.671662729 2.671662729 6.079652442 O -2.671662729 -2.671662729 -6.079652442 O -2.671662729 -2.671662729 6.079652442 O 1.644951111 -1.644951111 -9.029413066 O 1.644951111 -1.644951111 9.029413066 O -1.644951111 1.644951111 -9.029413066 O -1.644951111 1.644951111 9.029413066 O 2.558767457 2.558767457 -11.893600003 O 2.558767457 2.558767457 11.893600003 O -2.558767457 -2.558767457 -11.893600003 O -2.558767457 -2.558767457 11.893600003 K_POINTS (automatic) 12 12 1 0 0 0 Please tell me if you can see the problem ..... thanks again Have a nice day . Hashem Al-Yamani Computational physics department Jordan Univ. -------------- next part -------------- An HTML attachment was scrubbed... URL: http://www.democritos.it/pipermail/pw_forum/attachments/20080131/6764422f/attachment.htm From giannozz at nest.sns.it Thu Jan 31 16:40:12 2008 From: giannozz at nest.sns.it (Paolo Giannozzi) Date: Thu, 31 Jan 2008 16:40:12 +0100 Subject: [Pw_forum] Incompatable nr1 or nr2 or nr3 In-Reply-To: <82a593400801310709r63dd0ae1l5da121ea61165f26@mail.gmail.com> References: <82a593400801310709r63dd0ae1l5da121ea61165f26@mail.gmail.com> Message-ID: <40BD07E7-AA91-4A28-A1A9-02C0DD2D85CE@nest.sns.it> On Jan 31, 2008, at 16:09 , Hashem Al-Yamani wrote: > > The oxygen input file was : > [...] > ibrav=1, > celldm(1)=14.0, > [...] > And the whole system input file was : > [...] > ibrav = 6, > celldm(1) = 8.6714, > celldm(3) = 6.923880978 you cannot plot differences between charge densities calculated for different unit cells Paolo --- Paolo Giannozzi, Dept of Physics, University of Udine via delle Scienze 208, 33100 Udine, Italy Phone +39-0432-558216, fax +39-0432-558222 From eyvaz_isaev at yahoo.com Thu Jan 31 19:31:53 2008 From: eyvaz_isaev at yahoo.com (Eyvaz Isaev) Date: Thu, 31 Jan 2008 10:31:53 -0800 (PST) Subject: [Pw_forum] Incompatable nr1 or nr2 or nr3 In-Reply-To: <82a593400801310709r63dd0ae1l5da121ea61165f26@mail.gmail.com> Message-ID: <190698.33465.qm@web60317.mail.yahoo.com> Please have a look at: 1) with oxygen: > &system > ibrav=1, > celldm(1)=14.0, > nat=1, > ntyp=1, > nbnd=6, > nosym=.true., > ecutwfc=27.0, > ecutrho=216.0, and then > &SYSTEM > ibrav = 6, > celldm(1) = 8.6714, > celldm(3) = 6.923880978, > nat = 29, > ntyp = 2, > ecutwfc = 55.D0,ecutrho=600.0, According to your input files there are lots of reasons to get incompatible nr1,nr2,nr3. At least, two of them I suggested turned out to be correct: different symmetry and lattice vectors. One more reason is different ecutwfc. To me it is unclear, how you are going to contract the charge density for crystals of two different symmetry? Bests, Eyvaz. --- Hashem Al-Yamani wrote: > Thanx alot Eyvaz and all .... In fact am still > working on it .... please > tell me if am wrong but I think the problem is due > to different - or lets > say - incompatible data between the system input > file and the adsorbed > oxygen ..... am sending the two input files data as > it might help ..... > > > *The oxygen input file was : * > > &control > calculation='scf', > restart_mode='from_scratch', > > pseudo_dir='/home/new_generation/espresso-3.2/pseudo/' > outdir='/home/new_generation/espresso-3.2/tmp/' > prefix = 'O' > / > &system > ibrav=1, > celldm(1)=14.0, > nat=1, > ntyp=1, > nbnd=6, > nosym=.true., > ecutwfc=27.0, > ecutrho=216.0, > occupations='from_input', > nspin=2, > starting_magnetization(1)=0.5d0, > / > &electrons > mixing_beta=0.25, > conv_thr=1.0E-8, > / > ATOMIC_SPECIES > O 15.99994 O.pbe-rrkjus.UPF > ATOMIC_POSITIONS > O 0.000000000 0.000000000 0.000000000 > K_POINTS (gamma) > OCCUPATIONS > 1.0 1.0 1.0 1.0 0.0 0.0 > 1.0 0.33333333333 0.33333333333 0.33333333333 0.0 > 0.0 > > ------------------------------------------------------------------------------------------ > > > *And the whole system input file was : * > > &CONTROL > calculation = "scf", > restart_mode = 'from_scratch' , > prefix = 'IrO2-OtopIr', > outdir = > '/home/new_generation/espresso-3.2-2/tmp', > pseudo_dir = > '/home/new_generation/espresso-3.2-2/pseudo' > / > &SYSTEM > ibrav = 6, > celldm(1) = 8.6714, > celldm(3) = 6.923880978, > nat = 29, > ntyp = 2, > ecutwfc = 55.D0,ecutrho=600.0, > occupations = "smearing", > smearing = "methfessel-paxton", > degauss = 0.03D0, > / > &ELECTRONS > electron_maxstep =300, > conv_thr = 1.D-8, > mixing_beta = 0.3D0, > startingpot = 'atomic' > startingwfc = 'atomic' > / > &IONS > ion_dynamics = 'bfgs', > / > ATOMIC_SPECIES > Ir 192.217 Ir.pbe-n-rrkjus.UPF.txt > O 15.999 O.pbe-rrkjus.UPF > ATOMIC_POSITIONS (bohr) > Ir 0.000000000 0.000000000 -12.223315777 > Ir -4.335700000 4.335700000 -9.151253653 > Ir 0.000000000 0.000000000 -6.022199397 > Ir -4.335700000 4.335700000 -3.039010085 > Ir 0.000000000 0.000000000 0.000000000 > Ir -4.335700000 4.335700000 3.039010085 > Ir 0.000000000 0.000000000 6.022199397 > Ir -4.335700000 4.335700000 9.151253653 > Ir 0.000000000 0.000000000 12.223315777 > O 0.000000000 0.000000000 15.557622468 > O 0.000000000 0.000000000 -15.557622468 > O 2.674536021 2.674536021 0.000000000 > O -2.674536021 -2.674536021 0.000000000 > O 1.669820608 -1.669820608 -3.018450895 > O 1.669820608 -1.669820608 3.018450895 > O -1.669820608 1.669820608 -3.018450895 > O -1.669820608 1.669820608 3.018450895 > O 2.671662729 2.671662729 -6.079652442 > O 2.671662729 2.671662729 6.079652442 > O -2.671662729 -2.671662729 -6.079652442 > O -2.671662729 -2.671662729 6.079652442 > O 1.644951111 -1.644951111 -9.029413066 > O 1.644951111 -1.644951111 9.029413066 > O -1.644951111 1.644951111 -9.029413066 > O -1.644951111 1.644951111 9.029413066 > O 2.558767457 2.558767457 -11.893600003 > O 2.558767457 2.558767457 11.893600003 > O -2.558767457 -2.558767457 -11.893600003 > O -2.558767457 -2.558767457 11.893600003 > K_POINTS (automatic) > 12 12 1 0 0 0 > > > Please tell me if you can see the problem ..... > thanks again > > Have a nice day . > > Hashem Al-Yamani > Computational physics department > Jordan Univ. > > _______________________________________________ > Pw_forum mailing list > Pw_forum at pwscf.org > http://www.democritos.it/mailman/listinfo/pw_forum > ------------------------------------------------------------------- Prof. Eyvaz Isaev, Theoretical Physics Department, Moscow State Institute of Steel & Alloys, Russia, IFM, Linkoping University, Sweden Condensed Matter Theory Group, Uppsala University, Sweden Eyvaz.Isaev at fysik.uu.se, eyvaz_isaev at yahoo.com ____________________________________________________________________________________ Be a better friend, newshound, and know-it-all with Yahoo! Mobile. Try it now. http://mobile.yahoo.com/;_ylt=Ahu06i62sR8HDtDypao8Wcj9tAcJ From eyvaz_isaev at yahoo.com Thu Jan 31 20:11:08 2008 From: eyvaz_isaev at yahoo.com (Eyvaz Isaev) Date: Thu, 31 Jan 2008 11:11:08 -0800 (PST) Subject: [Pw_forum] Incompatable nr1 or nr2 or nr3 In-Reply-To: <190698.33465.qm@web60317.mail.yahoo.com> Message-ID: <752063.71143.qm@web60319.mail.yahoo.com> > different symmetry and lattice vectors Sorry, I have mistaken: lattice parameters, but not lattice vectors. Bests, Eyvaz. --- Eyvaz Isaev wrote: > Please have a look at: > > 1) with oxygen: > > &system > > ibrav=1, > > celldm(1)=14.0, > > nat=1, > > ntyp=1, > > nbnd=6, > > nosym=.true., > > ecutwfc=27.0, > > ecutrho=216.0, > > and then > > > &SYSTEM > > ibrav = 6, > > celldm(1) = 8.6714, > > celldm(3) = 6.923880978, > > nat = 29, > > ntyp = 2, > > ecutwfc = 55.D0,ecutrho=600.0, > > According to your input files there are lots of > reasons to get incompatible nr1,nr2,nr3. At least, > two > of them I suggested turned out to be correct: > different symmetry and lattice vectors. One more > reason is different ecutwfc. > > To me it is unclear, how you are going to contract > the > charge density for crystals of two different > symmetry? > > Bests, > Eyvaz. > > --- Hashem Al-Yamani > wrote: > > > Thanx alot Eyvaz and all .... In fact am still > > working on it .... please > > tell me if am wrong but I think the problem is due > > to different - or lets > > say - incompatible data between the system > input > > file and the adsorbed > > oxygen ..... am sending the two input files data > as > > it might help ..... > > > > > > *The oxygen input file was : * > > > > &control > > calculation='scf', > > restart_mode='from_scratch', > > > > > pseudo_dir='/home/new_generation/espresso-3.2/pseudo/' > > > outdir='/home/new_generation/espresso-3.2/tmp/' > > prefix = 'O' > > / > > &system > > ibrav=1, > > celldm(1)=14.0, > > nat=1, > > ntyp=1, > > nbnd=6, > > nosym=.true., > > ecutwfc=27.0, > > ecutrho=216.0, > > occupations='from_input', > > nspin=2, > > starting_magnetization(1)=0.5d0, > > / > > &electrons > > mixing_beta=0.25, > > conv_thr=1.0E-8, > > / > > ATOMIC_SPECIES > > O 15.99994 O.pbe-rrkjus.UPF > > ATOMIC_POSITIONS > > O 0.000000000 0.000000000 0.000000000 > > K_POINTS (gamma) > > OCCUPATIONS > > 1.0 1.0 1.0 1.0 0.0 0.0 > > 1.0 0.33333333333 0.33333333333 0.33333333333 0.0 > > 0.0 > > > > > ------------------------------------------------------------------------------------------ > > > > > > *And the whole system input file was : * > > > > &CONTROL > > calculation = "scf", > > restart_mode = 'from_scratch' , > > prefix = 'IrO2-OtopIr', > > outdir = > > '/home/new_generation/espresso-3.2-2/tmp', > > pseudo_dir = > > '/home/new_generation/espresso-3.2-2/pseudo' > > / > > &SYSTEM > > ibrav = 6, > > celldm(1) = 8.6714, > > celldm(3) = 6.923880978, > > nat = 29, > > ntyp = 2, > > ecutwfc = 55.D0,ecutrho=600.0, > > occupations = "smearing", > > smearing = "methfessel-paxton", > > degauss = 0.03D0, > > / > > &ELECTRONS > > electron_maxstep =300, > > conv_thr = 1.D-8, > > mixing_beta = 0.3D0, > > startingpot = 'atomic' > > startingwfc = 'atomic' > > / > > &IONS > > ion_dynamics = 'bfgs', > > / > > ATOMIC_SPECIES > > Ir 192.217 Ir.pbe-n-rrkjus.UPF.txt > > O 15.999 O.pbe-rrkjus.UPF > > ATOMIC_POSITIONS (bohr) > > Ir 0.000000000 0.000000000 -12.223315777 > > Ir -4.335700000 4.335700000 -9.151253653 > > Ir 0.000000000 0.000000000 -6.022199397 > > Ir -4.335700000 4.335700000 -3.039010085 > > Ir 0.000000000 0.000000000 0.000000000 > > Ir -4.335700000 4.335700000 3.039010085 > > Ir 0.000000000 0.000000000 6.022199397 > > Ir -4.335700000 4.335700000 9.151253653 > > Ir 0.000000000 0.000000000 12.223315777 > > O 0.000000000 0.000000000 15.557622468 > > O 0.000000000 0.000000000 -15.557622468 > > O 2.674536021 2.674536021 0.000000000 > > O -2.674536021 -2.674536021 0.000000000 > > O 1.669820608 -1.669820608 -3.018450895 > > O 1.669820608 -1.669820608 3.018450895 > > O -1.669820608 1.669820608 -3.018450895 > > O -1.669820608 1.669820608 3.018450895 > > O 2.671662729 2.671662729 -6.079652442 > > O 2.671662729 2.671662729 6.079652442 > > O -2.671662729 -2.671662729 -6.079652442 > > O -2.671662729 -2.671662729 6.079652442 > > O 1.644951111 -1.644951111 -9.029413066 > > O 1.644951111 -1.644951111 9.029413066 > > O -1.644951111 1.644951111 -9.029413066 > > O -1.644951111 1.644951111 9.029413066 > > O 2.558767457 2.558767457 -11.893600003 > > O 2.558767457 2.558767457 11.893600003 > > O -2.558767457 -2.558767457 -11.893600003 > > O -2.558767457 -2.558767457 11.893600003 > > K_POINTS (automatic) > > 12 12 1 0 0 0 > > > > > > Please tell me if you can see the problem ..... > > thanks again > > > > Have a nice day . > > > > Hashem Al-Yamani > > Computational physics department > > Jordan Univ. > > > _______________________________________________ > > Pw_forum mailing list > > Pw_forum at pwscf.org > > http://www.democritos.it/mailman/listinfo/pw_forum > > > > > ------------------------------------------------------------------- > Prof. Eyvaz Isaev, > Theoretical Physics Department, Moscow State > Institute of Steel & Alloys, Russia, > IFM, Linkoping University, Sweden > Condensed Matter Theory Group, Uppsala University, > Sweden > Eyvaz.Isaev at fysik.uu.se, eyvaz_isaev at yahoo.com > > > > ____________________________________________________________________________________ > Be a better friend, newshound, and > know-it-all with Yahoo! Mobile. Try it now. > http://mobile.yahoo.com/;_ylt=Ahu06i62sR8HDtDypao8Wcj9tAcJ > > > _______________________________________________ > Pw_forum mailing list > Pw_forum at pwscf.org > http://www.democritos.it/mailman/listinfo/pw_forum > === message truncated === ------------------------------------------------------------------- Prof. Eyvaz Isaev, Theoretical Physics Department, Moscow State Institute of Steel & Alloys, Russia, IFM, Linkoping University, Sweden Condensed Matter Theory Group, Uppsala University, Sweden Eyvaz.Isaev at fysik.uu.se, eyvaz_isaev at yahoo.com ____________________________________________________________________________________ Never miss a thing. Make Yahoo your home page. http://www.yahoo.com/r/hs